Sie sind auf Seite 1von 356

Precalculus

Learner’s Material

This learning resource was collaboratively developed and


reviewed by educators from public and private schools, colleges, and/or
universities. We encourage teachers and other education stakeholders
to email their feedback, comments and recommendations to the
Department of Education at action@deped.gov.ph.
We value your feedback and recommendations.

Department of Education
Republic of the Philippines
Precalculus
Learner’s Material
First Edition 2016

Republic Act 8293. Section 176 states that: No copyright shall subsist in any work
of the Government of the Philippines. However, prior approval of the government agency or
office wherein the work is created shall be necessary for exploitation of such work for profit.
Such agency or office may, among other things, impose as a condition the payment of
royalties.

Borrowed materials (i.e., songs, stories, poems, pictures, photos, brand names,
trademarks, etc.) included in this learning resource are owned by their respective copyright
holders. DepEd is represented by the Filipinas Copyright Licensing Society (FILCOLS), Inc.
in seeking permission to use these materials from their respective copyright owners. All
means have been exhausted in seeking permission to use these materials. The publisher
and authors do not represent nor claim ownership over them.

Only institutions and companies which have entered an agreement with FILCOLS
and only within the agreed framework may copy from this Manual. Those who have not
entered in an agreement with FILCOLS must, if they wish to copy, contact the publishers
and authors directly.

Authors and publishers may email or contact FILCOLS at filcols@gmail.com or (02)


435-5258, respectively.

Published by the Department of Education


Secretary: Br. Armin A. Luistro FSC
Undersecretary: Dina S. Ocampo, PhD

Development Team of the Precalculus Learner’s Material


Joy P. Ascano Jesus Lemuel L. Martin Jr.
Arnel D. Olofernes Mark Anthony C. Tolentino, Ph.D
Reviewers:
Jerico B. Bacani, Ph.D Richard B. Eden, Ph.D
Dr. Flordeliza F. Francisco Mark Anthony J. Vidallo
Carly Mae Casteloy Angela Dianne Agustin
Cover Art Illustrator: Quincy D. Gonzales
Team Leader: Ian June L. Garces, Ph.D.

Management Team of the Precalculus Learner’s Material Bureau of Curriculum Development Bureau
of Learning Resources

Printed in the Philippines by Sunshine Interlinks Publishing House, Inc.


3F Maine City Tower, 236 Tomas Morato Avenue,
Brgy. South Triangle, Quezon City

Department of Education-Bureau of Learning Resources (DepEd-BLR)


Office Address: Ground Floor Bonifacio Building, DepEd Complex
Meralco Avenue, Pasig City, Philippines 1600
Telefax: (02) 634-1054, 634-1072, 631-4985
E-mail Address: blr.lrqad@deped.gov.ph / blr.lrpd@deped.gov.ph
Table of Contents

To the Precalculus Learners 1

DepEd Curriculum Guide for Precalculus 2

Unit 1: Analytic Geometry 6

Lesson 1.1: Introduction to Conic Sections and Circles . . . . . . . . 7


1.1.1: An Overview of Conic Sections . . . . . . . . . . . . . . . . . . . . . . . . . . . 7
1.1.2: De nition and Equation of a Circle . . . . . . . . . . . . . . . . . . . . . . . 8
1.1.3: More Properties of Circles . . . . . . . . . . . . . . . . . . . . . . . . . . . . . . . . 10
1.1.4: Situational Problems Involving Circles. . . . . . . . . . . . . . . . . . . . 12

Lesson 1.2: Parabolas. . . . . . . . . . . . . . . . . . . . . . . . . . . . . . . . . . . . . . . . . 19


1.2.1: De nition and Equation of a Parabola . . . . . . . . . . . . . . . . . . . . 19
1.2.2: More Properties of Parabolas . . . . . . . . . . . . . . . . . . . . . . . . . . . . . 23
1.2.3: Situational Problems Involving Parabolas . . . . . . . . . . . . . . . . 26

Lesson 1.3: Ellipses . . . . . . . . . . . . . . . . . . . . . . . . . . . . . . . . . . . . . . . . . . . 33


1.3.1: De nition and Equation of an Ellipse . . . . . . . . . . . . . . . . . . . . . 33
1.3.2: More Properties of Ellipses . . . . . . . . . . . . . . . . . . . . . . . . . . . . . . . 36
1.3.3: Situational Problems Involving Ellipses . . . . . . . . . . . . . . . . . . . 40

Lesson 1.4: Hyperbolas . . . . . . . . . . . . . . . . . . . . . . . . . . . . . . . . . . . . . . . 46


1.4.1: De nition and Equation of a Hyperbola . . . . . . . . . . . . . . . . . . 46
1.4.2: More Properties of Hyperbolas . . . . . . . . . . . . . . . . . . . . . . . . . . . 50
1.4.3: Situational Problems Involving Hyperbolas . . . . . . . . . . . . . . . 54

Lesson 1.5: More Problems on Conic Sections . . . . . . . . . . . . . . . . 60


1.5.1: Identifying the Conic Section by Inspection . . . . . . . . . . . . . . . 60
1.5.2: Problems Involving Di erent Conic Sections . . . . . . . . . . . . . . 62

iii
Lesson 1.6: Systems of Nonlinear Equations . . . . . . . . . . . . . . . . . . 67
1.6.1: Review of Techniques in Solving Systems of Linear
Equations . . . . . . . . . . . . . . . . . . . . . . . . . . . . . . . . . . . . . . . . . . . . . . . . 68

1.6.2: Solving Systems of Equations Using Substitution . . . . . . . . . 69


1.6.3: Solving Systems of Equations Using Elimination . . . . . . . . . . 70
1.6.4: Applications of Systems of Nonlinear Equations . . . . . . . . . . 73

Unit 2: Mathematical Induction 80

Lesson 2.1: Review of Sequences and Series . . . . . . . . . . . . . . . . . . . 81

Lesson 2.2: Sigma Notation . . . . . . . . . . . . . . . . . . . . . . . . . . . . . . . . . . . 86


2.2.1: Writing and Evaluating Sums in Sigma Notation . . . . . . . . . 87
2.2.2: Properties of Sigma Notation . . . . . . . . . . . . . . . . . . . . . . . . . . . . . 89

Lesson 2.3: Principle of Mathematical Induction . . . . . . . . . . . . . . 96


2.3.1: Proving Summation Identities . . . . . . . . . . . . . . . . . . . . . . . . . . . . 97
2.3.2: Proving Divisibility Statements . . . . . . . . . . . . . . . . . . . . . . . . . . . 101
?
2.3.3: Proving Inequalities . . . . . . . . . . . . . . . . . . . . . . . . . . . . . . . . . . . . . . 102
Lesson 2.4: The Binomial Theorem . . . . . . . . . . . . . . . . . . . . . . . . . . . 108
2.4.1: Pascal’s Triangle and the Concept of Combination . . . . . . . . 109
2.4.2: The Binomial Theorem . . . . . . . . . . . . . . . . . . . . . . . . . . . . . . . . . . . 111
2.4.3: Terms of a Binomial Expansion . . . . . . . . . . . . . . . . . . . . . . . . . . 114
?
2.4.4: Approximation and Combination Identities . . . . . . . . . . . . . . . 116

Unit 3: Trigonometry 123

Lesson 3.1: Angles in a Unit Circle . . . . . . . . . . . . . . . . . . . . . . . . . . . 124


3.1.1: Angle Measure . . . . . . . . . . . . . . . . . . . . . . . . . . . . . . . . . . . . . . . . . . . 124
3.1.2: Coterminal Angles . . . . . . . . . . . . . . . . . . . . . . . . . . . . . . . . . . . . . . . 128
3.1.3: Arc Length and Area of a Sector . . . . . . . . . . . . . . . . . . . . . . . . . 129

Lesson 3.2: Circular Functions . . . . . . . . . . . . . . . . . . . . . . . . . . . . . . . . 135


3.2.1: Circular Functions on Real Numbers . . . . . . . . . . . . . . . . . . . . . 136
3.2.2: Reference Angle . . . . . . . . . . . . . . . . . . . . . . . . . . . . . . . . . . . . . . . . . . 139
Lesson 3.3: Graphs of Circular Functions and Situational
Problems . . . . . . . . . . . . . . . . . . . . . . . . . . . . . . . . . . . . . . . . . 144
3.3.1: Graphs of y = sin x and y = cos x . . . . . . . . . . . . . . . . . . . . . . . . 145
3.3.2: Graphs of y = a sin bx and y = a cos bx . . . . . . . . . . . . . . . . . . . 147
3.3.3: Graphs of y = a sin b(x c) + d and
y = a cos b(x c) + d . . . . . . . . . . . . . . . . . . . . . . . . . . . . . . . . . . . . . 151
3.3.4: Graphs of Cosecant and Secant Functions . . . . . . . . . . . . . . . . 154
3.3.5: Graphs of Tangent and Cotangent Functions . . . . . . . . . . . . . 158
3.3.6: Simple Harmonic Motion . . . . . . . . . . . . . . . . . . . . . . . . . . . . . . . . . 160

Lesson 3.4: Fundamental Trigonometric Identities. . . . . . . . . . . . . 171


3.4.1: Domain of an Expression or Equation . . . . . . . . . . . . . . . . . . . . 171
3.4.2: Identity and Conditional Equation . . . . . . . . . . . . . . . . . . . . . . . 173
3.4.3: The Fundamental Trigonometric Identities . . . . . . . . . . . . . . . 174
3.4.4: Proving Trigonometric Identities . . . . . . . . . . . . . . . . . . . . . . . . . 176

Lesson 3.5: Sum and Di erence Identities . . . . . . . . . . . . . . . . . . . . . 181


3.5.1: The Cosine Di erence and Sum Identities . . . . . . . . . . . . . . . . 181
3.5.2: The Cofunction Identities and the Sine Sum and
Di erence Identities . . . . . . . . . . . . . . . . . . . . . . . . . . . . . . . . . . . . . . 183
3.5.3: The Tangent Sum and Di erence Identities . . . . . . . . . . . . . . . 186

Lesson 3.6: Double-Angle and Half-Angle Identities . . . . . . . . . . . 192


3.6.1: Double-Angle Identities . . . . . . . . . . . . . . . . . . . . . . . . . . . . . . . . . . 192
3.6.2: Half-Angle Identities . . . . . . . . . . . . . . . . . . . . . . . . . . . . . . . . . . . . . 195

Lesson 3.7: Inverse Trigonometric Functions . . . . . . . . . . . . . . . . . . 201


3.7.1: Inverse Sine Function . . . . . . . . . . . . . . . . . . . . . . . . . . . . . . . . . . . . 202
3.7.2: Inverse Cosine Function . . . . . . . . . . . . . . . . . . . . . . . . . . . . . . . . . . 205
3.7.3: Inverse Tangent Function and the Remaining Inverse
Trigonometric Functions . . . . . . . . . . . . . . . . . . . . . . . . . . . . . . . . . 208

Lesson 3.8: Trigonometric Equations . . . . . . . . . . . . . . . . . . . . . . . . . . 220


3.8.1: Solutions of a Trigonometric Equation . . . . . . . . . . . . . . . . . . . . 221
3.8.2: Equations with One Term . . . . . . . . . . . . . . . . . . . . . . . . . . . . . . . . 224
3.8.3: Equations with Two or More Terms . . . . . . . . . . . . . . . . . . . . . . 227
Lesson 3.9: Polar Coordinate System . . . . . . . . . . . . . . . . . . . . . . . . . 236
3.9.1: Polar Coordinates of a Point . . . . . . . . . . . . . . . . . . . . . . . . . . . . . 237
3.9.2: From Polar to Rectangular, and Vice Versa . . . . . . . . . . . . . . . 241
3.9.3: Basic Polar Graphs and Applications . . . . . . . . . . . . . . . . . . . . . 244

Answers to Odd-Numbered Exercises in Supplementary Problems


and All Exercises in Topic Tests 255

References 290
To the Precalculus Learners

The Precalculus course bridges basic mathematics and calculus. This course
completes your foundational knowledge on algebra, geometry, and trigonometry.
It provides you with conceptual understanding and computational skills that are
prerequisites for Basic Calculus and future STEM courses.
Based on the Curriculum Guide for Precalculus of the Department of Edu-
cation (see pages 2-5), the primary aim of this Learning Manual is to give
you an adequate stand-alone material that can be used for the Grade 11
Precalculus course.
The Manual is divided into three units: analytic geometry, summation no-
tation and mathematical induction, and trigonometry. Each unit is composed
of lessons that bring together related learning competencies in the unit. Each
lesson is further divided into sub-lessons that focus on one or two
competencies for e ective learning.
At the end of each lesson, more examples are given in Solved Examples
to reinforce the ideas and skills being developed in the lesson. You have the
oppor-tunity to check your understanding of the lesson by solving the
Supplementary Problems. Finally, two sets of Topic Test are included to
prepare you for the exam.
Answers, solutions, or hints to odd-numbered items in the Supplementary
Problems and all items in the Topic Tests are provided at the end of the
Manual to guide you while solving them. We hope that you will use this
feature of the Manual responsibly.
Some items are marked with a star. A starred sub-lesson means the
discussion and accomplishment of the sub-lesson are optional. This will be
decided by your teacher. On the other hand, a starred example or exercise
means the use of calculator is required.
We hope that you will nd this Learning Manual helpful and convenient to
use. We encourage you to carefully study this Manual and solve the
exercises yourselves with the guidance of your teacher. Although great e ort
has been put into this Manual for technical correctness and precision, any
mistake found and reported to the Team is a gain for other students. Thank
you for your cooperation.

The Precalculus LM Team

All rights reserved. No part of this material may be reproduced or transmitted in any form or by any means -
electronic or mechanical including photocopying – without written permission from the DepEd Central Office. First Edition, 2016.
without–photocopyingincludingmechanicalorelectronic

K to 12 BASIC EDUCATION CURRICULUM


SENIOR HIGH SCHOOL – SCIENCE, TECHNOLOGY, ENGINEERING AND MATHEMATICS (STEM) SPECIALIZED SUBJECT

Grade: 11 Semester: First Semester


maymaterialthisofpartNo.reservedrightsAll

Core Subject Title: Pre-Calculus No. of Hours/ Semester: 80 hours/ semester


Pre-requisite (if needed):

Subject Description: At the end of the course, the students must be able to apply concepts and solve problems involving conic sections, systems of nonlinear
equations, series and mathematical induction, circular and trigonometric functions, trigonometric identities, and polar coordinate system.

CONTENT PERFORMANCE
CONTENT LEARNING COMPETENCIES CODE
STANDARDS STANDARDS
Analytic The learners The learners shall be able The learners...
Geometry demonstrate an to...
understanding STEM_PC11AG-Ia-1
1. illustrate the different types of conic sections: parabola, ellipse,
of...
circle, hyperbola, and degenerate cases.***
anyintransmittedorreproducedbe
DepEdthefrompermissionwritten Central .Office Edition,First .2016

model situations 2. define a circle. STEM_PC11AG-Ia-2


appropriately and solve 3. determine the standard form of equation of a circle STEM_PC11AG-Ia-3
key concepts of problems accurately using
conic sections and conic sections and systems 4. graph a circle in a rectangular coordinate system STEM_PC11AG-Ia-4
systems of of nonlinear equations 5. define a parabola STEM_PC11AG-Ia-5
nonlinear 6. determine the standard form of equation of a parabola STEM_PC11AG-Ib-1
equations 7. graph a parabola in a rectangular coordinate system STEM_PC11AG-Ib-2
8. define an ellipse STEM_PC11AG-Ic-1
9. determine the standard form of equation of an ellipse STEM_PC11AG-Ic-2
10. graph an ellipse in a rectangular coordinate system STEM_PC11AG-Ic-3
11. define a hyperbola STEM_PC11AG-Id-1
2 orform anyby

12. determine the standard form of equation of a hyperbola STEM_PC11AG-Id-2


-means

K to 12 Senior High School STEM Specialized Subject – Pre-Calculus December 2013 Page 1 of 4
without–photocopyingincludingmechanicalorelectronic

K to 12 BASIC EDUCATION CURRICULUM


SENIOR HIGH SCHOOL – SCIENCE, TECHNOLOGY, ENGINEERING AND MATHEMATICS (STEM) SPECIALIZED SUBJECT
maymaterialthisofpartNo.reservedrightsAll

CONTENT PERFORMANCE
CONTENT LEARNING COMPETENCIES CODE
STANDARDS STANDARDS
13. graph a hyperbola in a rectangular coordinate system STEM_PC11AG-Id-3
14. recognize the equation and important characteristics of the
STEM_PC11AG-Ie-1
different types of conic sections
15. solves situational problems involving conic sections STEM_PC11AG-Ie-2

16. illustrate systems of nonlinear equations STEM_PC11AG-If-1


17. determine the solutions of systems of nonlinear equations using
STEM_PC11AG-If-g-1
techniques such as substitution, elimination, and graphing***
18. solve situational problems involving systems
STEM_PC11AG-Ig-2
of nonlinear equations
Series and key concepts of keenly observe and STEM_PC11SMI-Ih-1
1. illustrate a series
Mathematical series and investigate patterns, and
Induction mathematical formulate appropriate
anyintransmittedorreproducedbe

2. differentiate a series from a sequence STEM_PC11SMI-Ih-2


DepEdthefrompermissionwritten Central .Office Edition,First .2016

induction and the mathematical statements 3. use the sigma notation to represent a series STEM_PC11SMI-Ih-3
Binomial and prove them using 4. illustrate the Principle of Mathematical Induction STEM_PC11SMI-Ih-4
Theorem. mathematical induction 5. apply mathematical induction in proving identities STEM_PC11SMI-Ih-i-1
and/or Binomial Theorem. 6. illustrate Pascal’s Triangle in the expansion of + for small STEM_PC11SMI-Ii-2
positive integral values of
7. prove the Binomial Theorem STEM_PC11SMI-Ii-3
8. determine any term of STEM_PC11SMI-Ij-1
+ , where is a positive integer,

without expanding
9. solve problems using mathematical induction and the Binomial
STEM_PC11SMI-Ij-2
Theorem
3 orform anyby -means

K to 12 Senior High School STEM Specialized Subject – Pre-Calculus December 2013 Page 2 of 4
without–photocopyingincludingmechanicalorelectronic

K to 12 BASIC EDUCATION CURRICULUM


SENIOR HIGH SCHOOL – SCIENCE, TECHNOLOGY, ENGINEERING AND MATHEMATICS (STEM) SPECIALIZED SUBJECT

CONTENT PERFORMANCE
CONTENT LEARNING COMPETENCIES CODE
STANDARDS STANDARDS
maymaterialthisofpartNo.reservedrightsAll

Trigonometry key concepts of 1. formulate and solve 1. illustrate the unit circle and the relationship between the linear
circular functions, accurately situational and angular measures of a central angle in a unit circle STEM_PC11T-IIa-1
trigonometric problems involving 2. convert degree measure to radian measure and vice versa STEM_PC11T-IIa-2
identities, inverse circular functions
trigonometric 3. illustrate angles in standard position and coterminal angles STEM_PC11T-IIa-3
functions, and 4. illustrate the different circular functions STEM_PC11T-IIb-1
the polar
coordinate 5. uses reference angles to find exact values of circular functions STEM_PC11T-IIb-2
system 6. determine the domain and range of the different circular functions STEM_PC11T-IIc-1
7. graph the six circular functions (a) amplitude, (b) period, and (c)
STEM_PC11T-IIc-d-1
phase shift
8. solve problems involving circular functions STEM_PC11T-IId-2
2. apply appropriate 9. determine whether an equation is an identity or a conditional
STEM_PC11T-IIe-1
trigonometric identities in equation
solving situational 10. derive the fundamental trigonometric identities STEM_PC11T-IIe-2
problems 11. derive trigonometric identities involving sum and difference of
STEM_PC11T-IIe-3
anyintransmittedorreproducedbe

angles
DepEdthefrompermissionwritten Central .Office Edition,First .2016

12. derive the double and half-angle formulas STEM_PC11T-IIf-1


13. simplify trigonometric expressions STEM_PC11T-IIf-2
14. prove other trigonometric identities STEM_PC11T-IIf-g-1
15. solve situational problems involving trigonometric identities STEM_PC11T-IIg-2
3. formulate and solve 16. illustrate the domain and range of the inverse trigonometric
STEM_PC11T-IIh-1
accurately situational functions.
problems involving 17. evaluate an inverse trigonometric expression. STEM_PC11T-IIh-2
appropriate trigonometric 18. solve trigonometric equations. STEM_PC11T-IIh-i-1
functions 19. solve situational problems involving inverse trigonometric
STEM_PC11T-IIi-2
functions and trigonometric equations
4 orform anyby

4. formulate and solve 20. locate points in polar coordinate system STEM_PC11T-IIj-1
accurately situational 21. convert the coordinates of a point from rectangular to polar
STEM_PC11T-IIj-2
problems involving the systems and vice versa
polar coordinate system 22. solve situational problems involving polar coordinate system STEM_PC11T-IIj-3

***Suggestion for ICT-enhanced lesson when available and where appropriate


-means

K to 12 Senior High School STEM Specialized Subject – Pre-Calculus December 2013 Page 3 of 4
K to 12 BASIC EDUCATION CURRICULUM
SENIOR HIGH SCHOOL – SCIENCE, TECHNOLOGY, ENGINEERING AND MATHEMATICS (STEM) SPECIALIZED SUBJECT
Allrightsreserved.No

Code Book Legend


electronic or mechanical including

Sample: STEM_PC11AG-Ia-1

LEGEND SAMPLE DOMAIN/ COMPONENT CODE


without–photocopying
maymaterialthisofpart

Learning Area and Science, Technology,


Strand/ Subject or Engineering and Mathematics Analytic Geometry AG
Specialization Pre-Calculus
First Entry
Series and Mathematical Induction SMI
Grade Level Grade 11
STEM_PC11AG
anyintransmittedorreproducedbe

Trigonometry T
DepEdthefrompermissionwritten Central .Office First

Uppercase Domain/Content/
Analytic Geometry
Letter/s Component/ Topic

-
Roman Numeral
*Zero if no specific Quarter First Quarter I
quarter
Lowercase
Letter/s
*Put a hyphen (-) in
between letters to Week Week one a
5 orform anyby s

indicate more than a


specific week
-
illustrate the different types
of conic sections: parabola,
Arabic Number Competency ellipse, circle, hyperbola,
1
mean

and degenerate cases


K to 12 Senior High School STEM Specialized Subject – Pre-Calculus December 2013 Page 4 of 4
-
Edition, 2016.
Unit 1

Analytic Geometry

San Juanico Bridge, by Morten N rb e, 21 June 2009,

https://commons.wikimedia.org/wiki/File%3ASan Juanico Bridge 2.JPG. Public Domain.

Stretching from Samar to Leyte with a total length of more than two kilome-
ters, the San Juanico Bridge has been serving as one of the main thoroughfares
of economic and social development in the country since its completion in 1973.
Adding picturesque e ect on the whole architecture, geometric structures are
subtly built to serve other purposes. The arch-shaped support on the main span
of the bridge helps maximize its strength to withstand mechanical resonance
and aeroelastic utter brought about by heavy vehicles and passing winds.

6
All rights reserved. No part of this material may be reproduced or transmitted in any form or by any means -
electronic or mechanical including photocopying – without written permission from the DepEd Central Office. First Edition, 2016.
Lesson 1.1. Introduction to Conic Sections and Circles

Learning Outcomes of the Lesson


At the end of the lesson, the student is able to:
illustrate the di erent types of conic sections: parabola, ellipse, circle, hyper-
bola, and degenerate cases;
de ne a circle;
determine the standard form of equation of a circle;
graph a circle in a rectangular coordinate system; and
solve situational problems involving conic sections (circles).

Lesson Outline
Introduction of the four conic sections, along with the degenerate conics
De nition of a circle
Derivation of the standard equation of a circle
Graphing circles
Solving situational problems involving circles

Introduction
We present the conic sections, a particular class of curves which
sometimes appear in nature and which have applications in other elds. In this
lesson, we rst illustrate how each of these curves is obtained from the
intersection of a plane and a cone, and then discuss the rst of their kind,
circles. The other conic sections will be covered in the next lessons.

1.1.1. An Overview of Conic Sections

We introduce the conic sections (or conics), a particular class of curves


which oftentimes appear in nature and which have applications in other elds.
One of the rst shapes we learned, a circle, is a conic. When you throw a ball,
the trajectory it takes is a parabola. The orbit taken by each planet around
the sun is an ellipse. Properties of hyperbolas have been used in the design
of certain telescopes and navigation systems. We will discuss circles in this
lesson, leaving parabolas, ellipses, and hyperbolas for subsequent lessons.
Circle (Figure 1.1) - when the plane is horizontal
Ellipse (Figure 1.1) - when the (tilted) plane intersects only one cone to
form a bounded curve

7
All rights reserved. No part of this material may be reproduced or transmitted in any form or by any means -
electronic or mechanical including photocopying – without written permission from the DepEd Central Office. First Edition, 2016.
Parabola (Figure 1.2) - when the plane intersects only one cone to form an
unbounded curve
Hyperbola (Figure 1.3) - when the plane (not necessarily vertical)
intersects both cones to form two unbounded curves (each called a branch
of the hyper-bola)

Figure 1.1 Figure 1.2 Figure 1.3


We can draw these conic sections (also called conics) on a rectangular
co-ordinate plane and nd their equations. To be able to do this, we will
present equivalent de nitions of these conic sections in subsequent sections,
and use these to nd the equations.
There are other ways for a plane and the cones to intersect, to form what
are referred to as degenerate conics: a point, one line, and two lines. See
Figures 1.4, 1.5 and 1.6.

Figure 1.4 Figure 1.5 Figure 1.6

1.1.2. De nition and Equation of a Circle

A circle may also be considered a special kind of ellipse (for the special case
when the tilted plane is horizontal). As we get to know more about a circle,
we will also be able to distinguish more between these two conics.

8
All rights reserved. No part of this material may be reproduced or transmitted in any form or by any means -
electronic or mechanical including photocopying – without written permission from the DepEd Central Office. First Edition, 2016.
p
See Figure 1.7, with the point C(3; 1) shown. From the gure, the distance
of A(2; 1) from C is AC = 5. By the distance formula, the distance of B(6; 5)
2 2
from C is BC = (6 3) + (5 1) = 5. There are other points P such that
P C = 5. The collection of all such points which are 5 units away from C,
forms a circle.

Figure 1.7 Figure 1.8

Let C be a given point. The set of all points P having the same
distance from C is called a circle. The point C is called the center
of the circle, and the common distance its radius.

The term radius is both used to refer to a segment from the center C to a
point P on the circle, and the length of this segment.
See Figure 1.8, where a circle is drawn. It has center C(h; k) and radius r
> 0. A point P (x; y) is on the circle if and only if P C = r. For any such point
then, its coordinates should satisfy the following.

P C = r2
p (x 2
+ (y 2
=r
2 2
h) k)

(x h) + (y k) =r

This is the standard equation of the circle with center C(h; k) and radius r. If
the center is the origin, then h = 0 and k = 0. The standard equation is then
2 2 2
x +y =r .

9
All rights reserved. No part of this material may be reproduced or transmitted in any form or by any means -
electronic or mechanical including photocopying – without written permission from the DepEd Central Office. First Edition, 2016.
Example 1.1.1. In each item, give
the standard equation of the circle
satisfy-ing the given conditions.
5888 center
at the
origin,
radius
4p
5889 center (4; 3), radius 7
5890 circle in Figure 1.7
5891 circle A in Figure 1.9
5892 circle B in Figure 1.9
(6) center (5; 6), tangent to the y-
axis Figure 1.9
(7) center (5; 6), tangent to the x-axis
(8) It has a diameter with endpoints A(1; 4) and B(4; 2).

2 2
Solution. (1) x + y = 16
2 2
(2) (x + 4) + (y 3) = 7
2 2
(3) The center is (3; 1) and the radius is 5, so the equation is (x 3) + (y 1) =
25.
23 By inspection, the center is (2; 1) and the radius is 4. The equation is (x
2 2
+ 2) + (y + 1) = 16.
2 2
(5) Similarly by inspection, we have (x 3) + (y 2) = 9.
(6) The center is 5 units away from the y-axis, so the radius is r = 5 (you can
2 2
make a sketch to see why). The equation is (x 5) + (y + 6) = 25.
23 Similarly, since the center is 6 units away from the x-axis, the equation is
2 2
(x 5) + (y + 6) = 36.
1+4 4+2 3
(8) The center C is the midpoint of A and B: C = 2 ; 2= 2 ; 3 . The
q
q
3 2 2 29
radius is then r = AC = 1 2 + (4 3) = 4 . The circle has
3 2 29

2
equation x 2 + (y 3) = 4 . 2
1.1.3. More Properties of Circles

After expanding, the standard equation


3 2 29
2
x 2 + (y 3) = 4
can be rewritten as
2 2
x +y 3x 6y + 4 = 0;
an equation of the circle in general form.

10
All rights reserved. No part of this material may be reproduced or transmitted in any form or by any means -
electronic or mechanical including photocopying – without written permission from the DepEd Central Office. First Edition, 2016.
If the equation of a circle is given in the general form
2 2
Ax + Ay + Cx + Dy + E = 0; A 6= ;0

or
2 2
x + y + Cx + Dy + E = 0;
we can determine the standard form by completing the square in both variables.
2
Completing the square in an expression like x + 14x means determining the
term to be added that will produce a perfect polynomial square. Since the coe
2
cient of x is already 1, we take half the coe cient of x and square it, and we get
2 2
49. Indeed, x + 14x + 49 = (x + 7) is a perfect square. To complete the square
2 2 2
in, say, 3x + 18x, we factor the coe cient of x from the expression: 3(x + 6x),
then add 9 inside. When completing a square in an equation, any extra term
introduced on one side should also be added to the other side.

Example 1.1.2. Identify the center and radius of the circle with the given
equa-tion in each item. Sketch its graph, and indicate the center.
2 2
(1) x + y 6x = 7
2 2
(2) x + y 14x + 2y = 14
2 2
(3) 16x + 16y + 96x 40y = 315

Solution. The rst step is to rewrite each equation in standard form by


complet-ing the square in x and in y. From the standard equation, we can
determine the center and radius.
(1)
2 2
x 6x + y = 7
2 2
x 6x + 9 + y = 7 + 9
(x 3)2 + y2 = 16

Center (3; 0), r = 4, Figure 1.10


(2)
2
x 14x + y2 + 2y = 14
2
x 14x + 49 + y2 + 2y + 1 = 14 + 49 + 1
2 2
(x 7) + (y + 1) = 36

Center (7; 1), r = 6, Figure 1.11


(3)
2 2
16x + 96x + 16y 40y = 315

11
All rights reserved. No part of this material may be reproduced or transmitted in any form or by any means -
electronic or mechanical including photocopying – without written permission from the DepEd Central Office.
First Edition, 2016.
5
2 2
16(x + 6x) + 16 y 2y = 315
16(x2 + 6x + 9) + 16 y2 2y + 16 = 315 + 16(9) + 16 16
5 25 25

5 2
2
16(x + 3) + 16 y 4 = 484
2
(x + 3) + y 4 = 16
=
4 = 2
5 2 484 121 11 2

Center 3; 4 , r = 5:5, Figure 1.12. 2

Figure 1.10 Figure 1.11 Figure 1.12


2 2 2
In the standard equation (x h) + (y k) = r , both the two squared terms
on the left side have coe cient 1. This is the reason why in the preceding
example, we divided by 16 at the last equation.

1.1.4. Situational Problems Involving Circles

Let us now take a look at some situational problems involving circles.


?
Example 1.1.3. A street with two lanes, each 10 ft wide, goes through a
semicircular tunnel with radius 12 ft. How high is the tunnel at the edge of
each lane? Round o to 2 decimal places.

12
All rights reserved. No part of this material may be reproduced or transmitted in any form or by any means -
electronic or mechanical including photocopying – without written permission from the DepEd Central Office. First Edition, 2016.
Solution. We draw a coordinate system with origin at the middle of the highway,
as shown. Because of the given radius, the tunnel’s boundary is on the circle x 2
+ y2 = 122. Point P is the point on the arc just above the edge of a lane, so
2 2 2
its x-coordinate is 10. We need its y-coordinate. We then solve 10 +y = 12
p

for y > 0, giving us y = 2 11 6:63 ft. 2


Example 1.1.4. A piece of a broken plate was dug up in an archaeological
site. It was put on top of a grid, as shown in Figure 1.13, with the arc of the
plate passing through A(7; 0), B(1; 4) and C(7; 2). Find its center, and the
standard equation of the circle describing the boundary of the plate.

Figure 1.13

Figure 1.14

Solution. We rst determine the center. It is the intersection of the perpendicular bisectors of AB and
BC (see Figure 1.14). Recall that, in a circle, the perpen-dicular bisector of any chord passes through
the center. Since the midpoint M
of AB is 7+1 ; 0+4
2 2
= (3; 2), and mAB = 40 = 1 , the perpendicular bisector
1+7 2

of AB has equation y 2 = 2(x + 3), or equivalently, y = 2x 4.


13
All rights reserved. No part of this material may be reproduced or transmitted in any form or by any means -
electronic or mechanical including photocopying – without written permission from the DepEd Central Office.
First Edition, 2016.
Since the midpoint N of BC is 1+7 ; 4+2 = (4; 3), and mBC = 24 = 1,
BC 2 2 71 3
the perpendicular bisector of has
y = 3x 9.
The intersection of the two lines y = 2x 4 and y = 3x 9 is (1; 6) (by
solving a system of linear equations). We can take the radius as the distance of this point from any of
A, B or C (it’s most convenient to use B in this case). We
2 2
then get r = 10. The standard equation is thus (x 1) + (y + 6) = 100. 2

More Solved Examples


23 In each item, give the standard equation of the circle satisying the given
con-ditions.
0 center at the origin, contains (0; 3)

1 center (1; 5), diameter 8

2 circle A in Figure 1.15

3 circle B in Figure 1.15

4 circle C in Figure 1.15

(f) center (2; 3), tangent to the y-


axis

(g) center (2; 3), tangent to the x-


axis
Figure 1.15
(h) contains the points (2; 0) and
(8; 0), radius 5
Solution:
2 2
(a) The radius is 3, so the equation is x + y = 9.
2 2
(b) The radius is 8=2 = 4, so the equation is (x 1) + (y 5) = 16.
(c) The center is (2; 2) and the radius is 2,
2 2
so the equation is (x + 2) + (y 2) = 4.
(d) The center is (2; 3) and the radius is 1,
2 2
so the equation is (x 2) + (y 3) = 1.
(e) The center is (1; 1) and by the
Pythagorean Theorem, the radius (see
2 2 p
Figure 1.16) is p 2 +2 = 8, so the
2 2
equation is (x 1) + (x + 1) = 8. Figure 1.16

14
All rights reserved. No part of this material may be reproduced or transmitted in any form or by any means -
electronic or mechanical including photocopying – without written permission from the DepEd Central Office. First Edition, 2016.
2 2
23 The radius is 3, so the equation is (x + 2) + (y + 3) = 9.
2 2
24 The radius is 2, so the equation is (x + 2) + (y + 3) = 4.
25 The distance between (2; 0) and (8; 0) is 10; since the radius is 5, these
two points are endpoints of a diameter. Then the circle has center at
(3; 0) and radius 5, so its equation is (x 3)2 + y2 = 25.
23 Identify the center and radius of the circle with the given equation in each
item. Sketch its graph, and indicate the center.
2 2
(a) x + y + 8y = 33
2 2
(b) 4x + 4y 16x + 40y + 67 = 0
2 2
(c) 4x + 12x + 4y + 16y 11=0
Solution:
(a)
2 2
x + y + 8y = 33
2 2
x + y + 8y + 16 = 33 + 16
2 2
x + (y + 4) = 49
Center (0; 4), radius 7, see Figure 1.17.
(b)
2 2
4x + 4y 16x + 40y + 67 = 0
67 2
x2 4x + y + 10y =
4
2 2 67
x 4x + 4 + y + 10y + 25 = +4+25
4
49 7 2

2 2
(x 2) + (y + 5) = 4 = 2
Center (2; 5), radius 3.5, see Figure 1.18.
(c)
2 2
4x + 12x + 4y + 16y 11 = 0
2 11 2
x + 3x + y + 4y =
4
2 9 2 11
x + 3x + + y + 4y + 4 = +9 +4
4 4 4
3 2

x+2 + (y + 2)2 = 9
Center 3 ; 2 , radius 3, see Figure 1.19.
2

15
All rights reserved. No part of this material may be reproduced or transmitted in any form or by any means -
electronic or mechanical including photocopying – without written permission from the DepEd Central Office. First Edition, 2016.
Figure 1.17 Figure 1.18 Figure 1.19
3. A circular play area with radius 3 m is
to be partitioned into two sections using
a straight fence as shown in Figure 1.20.
How long should the fence be?
Solution: To determine the length of the
fence, we need to determine the coordi-
nates of its endpoints. From Figure 1.20,
the endpoints have x coordinate 1 and
2 2
are on the circle x + y = 9. Then
1 + y = 9, or y = 2 2.
2 p Therefore,
p Figure 1.20
the length of the fence is 4 2 5:66 m.
0 A Cartesian coordinate system was used to identify locations on a circu-
lar track. As shown in Figure 1.21, the circular track contains the points
A(2; 4), B(2; 3), C(5; 2). Find the total length of the track.

Figure 1.21 Figure 1.22


Solution: The segment AB is vertical and has midpoint (2; 0:5), so its
perpendicular bisector has equation y = 0:5. On the other hand, the segment
BC has slope 1=7 and midpoint (1 :5; 2:5), so its perpendicular bisector has
equation y 2:5 = 7(x 1:5), or 7x y 8 = 0.
The center of the circle is the intersection of y = 0:5 and 7 x y 8 = 0; that
15 1
is, the center is at 14 ; 2 .
The radius of the circle is the distance from the center to any of the points A, B,

r2125 5p
or C; by the distance formula, the radius is 98 = 14 170. Therefore,

16
All rights reserved. No part of this material may be reproduced or transmitted in any form or by any means -
electronic or mechanical including photocopying – without written permission from the DepEd Central Office. First Edition, 2016.
the total length of the track (its circumference), is
5p
2 14 170 29:26 units:

Supplementary Problems 1.1


Identify the center and radius of the circle with the given equation in each item.
Sketch its graph, and indicate the center.

2 2 1
0x +y = 4
2 2
1 5x + 5y = 125
3 2

2
3. (x + 4) + y 4 =1
2 2
4. x 4x + y 4y 8 = 0
2 2
5. x + y 14x + 12y = 36
2 2
6. x + 10x + y 16y 11 = 0
2 2
7. 9x + 36x + 9y + 72y + 155 = 0
2 2
8. 9x + 9y 6x + 24y = 19
2 2
9. 16x + 80x + 16y 112y + 247 = 0

Find the standard equation of the circle which satis es the given
conditions. p
0 center at the origin, radius 5 3

1 center at (17; 5), radius 12

2 center at (8; 4), contains (4; 2)

13. center at (15; 7), tangent to the x-axis

14. center at (15; 7), tangent to the y-axis

15. center at (15; 7), tangent to the line y = 10

16. center at (15; 7), tangent to the line x=8

17. has a diameter with endpoints (3; 1) and (7; 6)

17
All rights reserved. No part of this material may be reproduced or transmitted in any form or by any means -
electronic or mechanical including photocopying – without written permission from the DepEd Central Office.
First Edition, 2016.
9 3

18. has a diameter with endpoints 2 ; 4 and 2 ;2


2 2
19. concentric with x + 20x + y 14y + 145 = 0, diameter 12
2 2
20. concentric with x 2x + y 2y 23 = 0 and has 1=5 the area
2 2
21. concentric with x + 4x + y 6y + 9 = 0 and has the same circumference as
2 2
x + 14x + y + 10y + 62 = 0
22. contains the points (3; 3), (7; 1), (0; 2)

23. contains the points (1; 4), (1; 2), (4; 3)

24. center at (3; 2) and tangent to the line 2x 3y = 1

25. center at (5; 1) and tangent to the line x + y + 10 = 0

26. has center with x-coordinate 4 and tangent to the line x + 3y = 9 at (3; 4)

23 A stadium is shaped as in Figure 1.23, where its left and right ends are
circular arcs both with center at C. What is the length of the stadium 50 m
from one of the straight sides?

Figure 1.23
5888 A waterway in a theme park
has a semicircular cross section
with di-ameter 11 ft. The boats that
are going to be used in this
waterway have rectangular cross
sections and are found to submerge
1 ft into the water. If the waterway is
to be lled with water 4.5 ft deep,
what is
the maximum possible width of the Figure 1.24
boats?

18
All rights reserved. No part of this material may be reproduced or transmitted in any form or by any means -
electronic or mechanical including photocopying – without written permission from the DepEd Central Office. First Edition, 2016.
Lesson 1.2. Parabolas

Learning Outcomes of the Lesson


At the end of the lesson, the student is able to:
0 de ne a parabola;
1 determine the standard form of equation of a parabola;
2 graph a parabola in a rectangular coordinate system; and
3 solve situational problems involving conic sections (parabolas).

Lesson Outline
0 De nition of a parabola
1 Derivation of the standard equation of a parabola
2 Graphing parabolas
3 Solving situational problems involving parabolas

Introduction
A parabola is one of the conic sections. We have already seen parabolas
which open upward or downward, as graphs of quadratic functions. Here, we
will see parabolas opening to the left or right. Applications of parabolas are
presented at the end.

1.2.1. De nition and Equation of a Parabola

Consider the point F (0; 2) and the line ‘ having equation y = 2, as shown in
Figure 1.25. What are the distances of A(4; 2) from F and from ‘? (The latter
is taken as the distance of A from A‘, the point on ‘ closest to A). How about
the distances of B(8; 8) from F and from ‘ (from B ‘)?

AF =4 andAA‘ = 4
p 2
BF = (80) + (8 2)2 = 10andBB‘ = 10
There are other points P such that P F = P P‘ (where P‘ is the closest point
on line ‘). The collection of all such points forms a shape called a parabola.

Let F be a given point, and ‘ a given line not containing F . The set of
all points P such that its distances from F and from ‘ are the same, is
called a parabola. The point F is its focus and the line ‘ its directrix.

19
All rights reserved. No part of this material may be reproduced or transmitted in any form or by any means -
electronic or mechanical including photocopying – without written permission from the DepEd Central Office. First Edition, 2016.
Figure 1.25

Figure 1.26
Consider a parabola with focus F (0; c) and directrix ‘ having equation y = c. See
Figure 1.26. The focus and directrix are c units above and below, respectively, the
origin. Let P (x; y) be a point on the parabola so P F = P P‘, where P‘ is the point on ‘
closest to P . The point P has to be on the same side of the directrix as the focus (if
P was below, it would be closer to ‘ than it is from F ).

PF = PP‘
p
2 2
x + (y c) = y (c) = y + c
2 2 2 2 2
x +y 2cy + c = y + 2cy + c
2
x = 4cy

The vertex V is the point midway between the focus and the directrix. This
2
equation, x = 4cy, is then the standard equation of a parabola opening
upward with vertex V (0; 0).
Suppose the focus is F (0; c) and the directrix is y = c. In this case, a point P
on the resulting parabola would be below the directrix (just like the focus).
Instead of opening upward, it will open downward. Consequently, P F =
p

2 2
x + (y + c) and P P‘ = c y (you may draw a version of Figure 1.26 for this
case). Computations similar to the one done above will lead to the equation

20
All rights reserved. No part of this material may be reproduced or transmitted in any form or by any means -
electronic or mechanical including photocopying – without written permission from the DepEd Central Office.
First Edition, 2016.
2
x = 4cy .
We collect here the features of the graph of a parabola with standard equation
2 2
x = 4cy or x = 4cy , where c > 0.

(1) vertex : origin V (0; 0)


If the parabola opens upward, the vertex is the lowest point. If the
parabola opens downward, the vertex is the highest point.
(2) directrix : the line y = c or y = c
The directrix is c units below or above the vertex.
0 focus: F (0; c) or F (0; c)
The focus is c units above or below the vertex.
Any point on the parabola has the same distance from the focus as
it has from the directrix.
1 axis of symmetry: x = 0 (the y-axis)
This line divides the parabola into two parts which are mirror images
of each other.
Example 1.2.1. Determine the focus and directrix of the parabola with the
given equation. Sketch the graph, and indicate the focus, directrix, vertex,
and axis of symmetry.

21
All rights reserved. No part of this material may be reproduced or transmitted in any form or by any means -
electronic or mechanical including photocopying – without written permission from the DepEd Central Office. First Edition, 2016.
2 2
(1) x = 12y (2) x = 6y

Solution. (1) The vertex is V (0; 0) and the parabola opens upward. From 4c
= 12, c = 3. The focus, c = 3 units above the vertex, is F (0; 3). The
directrix, 3 units below the vertex, is y = 3. The axis of symmetry is x = 0.

(2) The vertex is V (0; 0) and the parabola opens downward. From 4c = 6, c
3 3 3 3
= 2 . The focus, c = 2 units below the vertex, is F 0; 2 . The directrix, 2
3
units above the vertex, is y = 2 . The axis of symmetry is x = 0.

Example 1.2.2. What is the standard equation of the parabola in Figure 1.25?

2
Solution. From the gure, we deduce that c = 2. The equation is thus x =
8y. 2

22
All rights reserved. No part of this material may be reproduced or transmitted in any form or by any means -
electronic or mechanical including photocopying – without written permission from the DepEd Central Office. First Edition, 2016.
1.2.2. More Properties of Parabolas

The parabolas we considered so far are \vertical" and have their vertices at
the origin. Some parabolas open instead horizontally (to the left or right), and
some have vertices not at the origin. Their standard equations and properties
are given in the box. The corresponding computations are more involved, but
are similar to the one above, and so are not shown anymore.
In all four cases below, we assume that c > 0. The vertex is V (h; k), and
it lies between the focus F and the directrix ‘. The focus F is c units away
from the vertex V , and the directrix is c units away from the vertex. Recall
that, for any point on the parabola, its distance from the focus is the same as
its distance from the directrix.

2 2
(x h) = 4c(y k) (y k) = 4c(x h)

2 2
(x h) = 4c(y k) (y k) = 4c(x h)

directrix ‘: horizontal directrix ‘: vertical


axis of symmetry: x=h, vertical axis of symmetry: y=k, horizontal

23
All rights reserved. No part of this material may be reproduced or transmitted in any form or by any means -
electronic or mechanical including photocopying – without written permission from the DepEd Central Office. First Edition, 2016.
Note the following observations:

The equations are in terms of x h and y k: the vertex coordinates are


subtracted from the corresponding variable. Thus, replacing both h and k
with 0 would yield the case where the vertex is the origin. For instance,
2
this replacement applied to (x h) = 4c(y k) (parabola opening upward)
2
would yield x = 4cy, the rst standard equation we encountered (parabola
opening upward, vertex at the origin).
If the x-part is squared, the parabola is \vertical"; if the y-part is squared,
the parabola is \horizontal." In a horizontal parabola, the focus is on the
left or right of the vertex, and the directrix is vertical.
If the coe cient of the linear (non-squared) part is positive, the parabola
opens upward or to the right; if negative, downward or to the left.
Example 1.2.3. Figure 1.27 shows the graph of parabola, with only its focus
and vertex indicated. Find its standard equation. What is its directrix and its
axis of symmetry?

Solution. The vertex is V (5; 4) and the focus is F (3; 4). From these, we
deduce the following: h = 5, k = 4, c = 2 (the distance of the focus from the
2
vertex). Since the parabola opens to the left, we use the template (y k) =
4c(x h). Our equation is
2
(y + 4) = 8(x 5):
Its directrix is c = 2 units to the right of V , which is x = 7. Its axis is the
horizontal line through V : y = 4.

Figure 1.27

24
All rights reserved. No part of this material may be reproduced or transmitted in any form or by any means -
electronic or mechanical including photocopying – without written permission from the DepEd Central Office. First Edition, 2016.
2
The standard equation (y + 4) = 8(x 5) from the preceding example can
2
be rewritten as y + 8x + 8y 24 = 0, an equation of the parabola in general
form.
2
If the equation is given in the general form Ax + Cx + Dy + E = 0 (A and
2
C are nonzero) or By +Cx+Dy +E = 0 (B and C are nonzero), we can
determine the standard form by completing the square in both variables.

Example 1.2.4. Determine the vertex, focus, directrix, and axis of symmetry
of the parabola with the given equation. Sketch the parabola, and include
these points and lines.
2
(1) y 5x + 12y = 16
2
(2) 5x + 30x + 24y = 51

Solution. (1) We complete the square on y, and move x to the other side.
2
y + 12y = 5x 16
2
y + 12y + 36 = 5x 16 + 36 = 5x + 20
2
(y + 6) = 5(x + 4)
The parabola opens to the right. It has vertex V (4; 6). From 4c = 5, we
get c = 5 = 1:25. The focus is c = 1:25 units to the right of V : F (2:75; 6).
4
The (vertical) directrix is c = 1:25 units to the left of V : x = 5:25. The
(horizontal) axis is through V : y = 6.

25
All rights reserved. No part of this material may be reproduced or transmitted in any form or by any means -
electronic or mechanical including photocopying – without written permission from the DepEd Central Office. First Edition, 2016.
(2) We complete the square on x, and move y to the other side.
2
5x + 30x = 24y + 51
2
5(x + 6x + 9) = 24y + 51 + 5(9)
2
5(x + 3) = 24y + 96 = 24(y4)
2 24 (y 4)
(x + 3) =
5
In the last line, we divided by 5 for the squared part not to have any coe -
cient. The parabola opens downward. It has vertex V (3; 4).
24 6
From 4c = 5 , we get c = 5 = 1:2. The focus is c = 1:2 units below V :
F (3; 2:8). The (horizontal) directrix is c = 1:2 units above V : y = 5:2. The
(vertical) axis is through V : x = 3.

Example 1.2.5. A parabola has focus F (7; 9) and directrix y = 3. Find its
standard equation.

Solution. The directrix is horizontal, and the focus is above it. The parabola
then opens upward and its standard equation has the form (x h)2 = 4c(y k).
Since the distance from the focus to the directrix is 2c = 9 3 = 6, then c = 3.
Thus, the vertex is V (7; 6), the point 3 units below F . The standard equation is
2
then (x 7) = 12(y 6). 2

1.2.3. Situational Problems Involving Parabolas

Let us now solve some situational problems involving parabolas.

Example 1.2.6. A satellite dish has a shape called a paraboloid, where each
cross-section is a parabola. Since radio signals (parallel to the axis) will bounce

26
All rights reserved. No part of this material may be reproduced or transmitted in any form or by any means -
electronic or mechanical including photocopying – without written permission from the DepEd Central Office. First Edition, 2016.
o the surface of the dish to the focus, the receiver should be placed at the
focus. How far should the receiver be from the vertex, if the dish is 12 ft
across, and 4:5 ft deep at the vertex?

Solution. The second gure above shows a cross-section of the satellite dish
drawn on a rectangular coordinate system, with the vertex at the origin. From
the problem, we deduce that (6; 4:5) is a point on the parabola. We need the
2
distance of the focus from the vertex, i.e., the value of c in x = 4cy.
2
x = 4cy
2
6 = 4c(4:5)
2
6
c= =2
4 4:5
Thus, the receiver should be 2 ft away from the vertex. 2

Example 1.2.7. The cable of a suspension bridge hangs in the shape of a


parabola. The towers supporting the cable are 400 ft apart and 150 ft high. If
the cable, at its lowest, is 30 ft above the bridge at its midpoint, how high is
the cable 50 ft away (horizontally) from either tower?

27
All rights reserved. No part of this material may be reproduced or transmitted in any form or by any means -
electronic or mechanical including photocopying – without written permission from the DepEd Central Office. First Edition, 2016.
Solution. Refer to the gure above, where the parabolic cable is drawn with its
vertex on the y-axis 30 ft above the origin. We may write its equation as (x
2
0) = a(y 30); since we don’t need the focal distance, we use the simpler
variable a in place of 4c. Since the towers are 150 ft high and 400 ft apart,
we deduce from the gure that (200; 150) is a point on the parabola.
2
x = a(y 30)
2
200 = a(150 30)
2
a = 200 = 1000
120 3
1000 2
The parabola has equation x = 3 (y 30), or equivalently,
2
y = 0:003x + 30. For the two points on the parabola 50 ft away from the
towers, x = 150 or x = 150. If x = 150, then
2
0 = 0:003(150 ) + 30 = 97:5:

Thus, the cable is 97:5 ft high 50 ft away from either tower. (As expected, we
get the same answer from x = 150.) 2

More Solved Examples


For Examples 1 and 2, determine the focus and directrix of the parabola with the
given equation. Sketch the graph, and indicate the focus, directrix, and vertex.
2
2. 3x = 12y
2
1. y = 20x
Solution: 2 2
Solution: 3x = 12y , x = 4y
Vertex: V (0; 0), opens to the
Vertex: V (0; 0), opens
right 4c = 20 ) c = 5
downward 4c = 4 ) c = 1
Focus: F (5; 0), Directrix: x = 5 Focus: F (0; 1), Directrix: y = 1
See Figure 1.28. See Figure 1.29.

Figure 1.28 Figure 1.29

28
All rights reserved. No part of this material may be reproduced or transmitted in any form or by any means -
electronic or mechanical including photocopying – without written permission from the DepEd Central Office. First Edition, 2016.
0 Determine the standard equation of
the parabola in Figure 1.30 given only 3
its vertex and focus. Then determine V ;4 ;F(4; 4
2
its di-rectix and axis of symmetry. 5
Solution: c = 2 ) 4c = 10
Parabola opens to th
2
Equation: (y 4) =
Direc
trix: x
= 1,
Axis:
y=4

23 D
e
t
e
r
m
i
n
e

t
h
e

s
t
a
n
d
a
r
d

e
q
u
a
t
i
o
n

o
f
t
h
e
parabola in Figure 1.31 given only its
vertex and diretrix. Then determine its
focus and axis of symmetry.
Solution:
13 15
V 5; 2 , directrix: y = 2
c = 1 ) 4c = 4
Parabola opens downward
13 2
Equation: y 2 = 4( x 5)

Figure 1.30

Figure 1.31

11
Focus: 5; 2 , Axis: x = 5
For Examples 5 and 6, determine the vertex, focus, directrix, and axis of
sym-metry of the parabola with the given equation. Sketch the parabola, and
include these points and lines.
2
5. x 6x 2y + 9 = 0
Solution:
2
x 6x = 2y 9
2
x 6x + 9 = 2y
2
(x 3) = 2y

V (3; 0), parabola opens upward


1 1
4c = 2 ) c = 2 ,F 3; 2 ,
directrix: y = 1 , axis: x = 3
2 Figure 1.32
See Figure 1.32.

29
All rights reserved. No part of this material may be reproduced or transmitted in any form or by any means -
electronic or mechanical including photocopying – without written permission from the DepEd Central Office. First Edition, 2016.
2
0 3y + 8x + 24y + 40 =
0 Solution:
2
3y + 24y = 8x 40
2
3(y + 8y) = 8x 40
2
3(y + 8y + 16) = 8x 40+48
2
3(y + 4) = 8x + 8
8
2
(y + 4) = 3 (x 1)
V (1; 4), parabola opens to the left
8 2 1
4c = 3 ) c =3, F 3 ; 4 , Figure 1.33
5
directrix: x = , axis: y = 4
3
See Figure 1.33.
0 A parabola has focus F (11; 8) and directrix x = 17. Find its standard
equation.
Solution: Since the focus is 6 units to the right of the directrix, the parabola
opens to the right with 2c = 6. Then c = 3 and V (14; 8). Hence, the
2
equation is (y 8) = 12(x + 14).
0 A ashlight is shaped like a
paraboloid and the light source
is placed at the focus so that
the light bounces o parallel to
the axis of symmetry; this is
done to maximize illumination.
A particular ashlight has its light
source located 1 cm from the
base and is 6 cm deep; see
Figure 1.34. What is the width
Figure 1.34
of the ashlight’s opening?
Solution: Let the base (the vertex) of the ashlight be the point V (0; 0).
Then the light source (the focus) is at F (0; 1); so c = 1. Hence, the
2
parabola’s equation is x = 4y. To get the width of the opening, we need
the x coordinates of the points on the parabola with y coordinate 6.
2
x = 4(6) ) x = 2 p6
p p

Therefore, the width of the opening is 2 2 6 = 4 6 9:8 cm.

30
All rights reserved. No part of this material may be reproduced or transmitted in any form or by any means -
electronic or mechanical including photocopying – without written permission from the DepEd Central Office. First Edition, 2016.
9. An object thrown from a height of 2 m
above the ground follows a parabolic path
until the object falls to the ground; see
Figure 1.35. If the object reaches a
maximum height (measured from the
ground) of 7 m after travelling a hor-izontal
distance of 4 m, determine the horizontal
distance between the object’s initial and nal
positions.

Figure 1.35
Solution: Let V (0; 7) be the parabola’s vertex, which corresponds to the high-est point reached
by the object. Then the parabola’s equation is of the form
2
x = 4c(y 7) and the object’s starting point is at (4; 2). Then
16 4
2
(4) = 4c(2 7) ) c = 20 = 5 :

2 16
Hence, the equation of the parabola is x =
5 (y 7). When the object hits
the ground, the y coordinate is 0 and
2 r
x = 5(0 7)= 5 )x= 4 : 5

16 112 7

7
r
Since this point is to the right of the vertex, we choose x = +4 5 . Therefore,
7

r
the total distance travelled is 4 5 (4) 8:73 m.

Supplementary Problems 1.2


Determine the vertex, focus, directrix, and axis of symmetry of the parabola with
the given equation. Sketch the graph, and include these points and lines.

2
0 y = 36x
2
1 5x = 100y
2
3. y + 4x 14y = 53
2
4. y 2x + 2y 1 = 0
2
5. 2x 12x + 28y = 38
2
6. (3x 2) = 84y 112

31
All rights reserved. No part of this material may be reproduced or transmitted in any form or by any means -
electronic or mechanical including photocopying – without written permission from the DepEd Central Office. First Edition, 2016.
Find the standard equation of the parabola which satis es the given conditions.

7. vertex (7; 11), focus (16; 11)

8. vertex (10; 5), directrix y = 1

23 , directrix y = 11
9. focus 10;
2 2
3 37
10. focus ; 3 , directrix x =
2 2
11. axis of symmetry y = 9, directrix x = 24, vertex on the line 3y 5x = 7

12. vertex (0; 7), vertical axis of symmetry, through the point P (4; 5)

13. vertex (3; 8), horizontal axis of symmetry, through the point P (5; 12)

5888 A satellite dish shaped like a paraboloid has its receiver located at
the focus. How far is the receiver from the vertex if the dish is 10 ft across
and 3 ft deep at the center?

5889 A ashlight shaped like a paraboloid has its light source at the focus
located 1.5 cm from the base and is 10 cm wide at its opening. How deep
is the ashlight at its center?

5890 The ends of a rope are held in place at the top of two posts, 9 m
apart and each one 8 m high. If the rope assumes a parabolic shape and
touches the ground midway between the two posts, how high is the rope 2
m from one of the posts?

5891 Radiation is focused to an unhealthy area in a patient’s body using


a parabolic re ector, positioned in such a way that the target area is at the
focus. If the re ector is 30 cm wide and 15 cm deep at the center, how far
should the base of the re ector be from the target area?

5892 A rectangular object 25 m wide is to pass under a parabolic arch


that has a width of 32 m at the base and a height of 24 m at the center. If
the vertex of the parabola is at the top of the arch, what maximum height
should the rectangular object have?

32
All rights reserved. No part of this material may be reproduced or transmitted in any form or by any means -
electronic or mechanical including photocopying – without written permission from the DepEd Central Office. First Edition, 2016.
Lesson 1.3. Ellipses

Learning Outcomes of the Lesson


At the end of the lesson, the student is able to:
0 de ne an ellipse;
1 determine the standard form of equation of an ellipse;
2 graph an ellipse in a rectangular coordinate system; and
3 solve situational problems involving conic sections (ellipses).

Lesson Outline
0 De nition of an ellipse
1 Derivation of the standard equation of an ellipse
2 Graphing ellipses
3 Solving situational problems involving ellipses

Introduction
Unlike circle and parabola, an ellipse is one of the conic sections that most
stu-dents have not encountered formally before. Its shape is a bounded curve
which looks like a attened circle. The orbits of the planets in our solar system
around the sun happen to be elliptical in shape. Also, just like parabolas, ellipses
have re ective properties that have been used in the construction of certain
structures. These applications and more will be encountered in this lesson.

1.3.1. De nition and Equation of an Ellipse

Consider the points F1(3; 0) and F2(3; 0), as shown in Figure 1.36. What is
the sum of the distances of A(4; 2:4) from F 1 and from F2? How about the
sum of the distances of B (and C(0; 4)) from F 1 and from F2?

AF1 + AF2 = 7:4 + 2:6 = 10


BF1 + BF2 = 3:8 + 6:2 = 10
CF1+CF2 =5+5=10

There are other points P such that P F 1 + P F2 = 10. The collection of all
such points forms a shape called an ellipse.

33
All rights reserved. No part of this material may be reproduced or transmitted in any form or by any means -
electronic or mechanical including photocopying – without written permission from the DepEd Central Office. First Edition, 2016.
Figure 1.36
Figure 1.37

Let F1 and F2 be two distinct points. The set of all points P , whose
distances from F1 and from F2 add up to a certain constant, is called
an ellipse. The points F1 and F2 are called the foci of the ellipse.

Given are two points on the x-axis, F 1 (c; 0) and F2(c; 0), the foci, both c
units away from their center (0; 0). See Figure 1.37. Let P (x; y) be a point on
the ellipse. Let the common sum of the distances be 2a (the coe cient 2 will
make computations simpler). Thus, we have P F1 + P F2 = 2a.

P F1 = 2a P F2
p p
2 2 2 2
(x + c) + y = 2a (x c) + y
2 2 2 p
a (x c) +y =a cx
2 2
2
x + 2cx + c 2
+ y = 4a2 2
4a (x c)2 + y2 + x2 2cx + c + y
p

2 2 4 2 2 2
a2 x2 2cx + c + y = a 2a cx + c x
2 2 2 2 2 2 2 2 2
(a c )x + a y2 = a4 a c = a (a c )

2 2 2 2 2 2
p 2
b x +a y =a b by letting b = a2 c , so a > b
x2 y2

a2 + b 2 = 1
p 2 2
When we let b = a c , we assumed a > c. To see why this is true, look at
4P F1F2 in Figure 1.37. By the Triangle Inequality, P F 1 + P F2 > F1F2, which
implies 2a > 2c, so a > c.
We collect here the features of the graph of an ellipse with standard equation
2 2
x y p 2 2
2 + 2 = 1, where a > b. Let c = a b .
a b

34
All rights reserved. No part of this material may be reproduced or transmitted in any form or by any means -
electronic or mechanical including photocopying – without written permission from the DepEd Central Office. First Edition, 2016.
0 center: origin (0; 0)
1 foci: F1(c; 0) and F2(c; 0)

Each focus is c units away from the center.


For any point on the ellipse, the sum of its distances from the foci is 2a.

(3) vertices: V1(a; 0) and V2(a; 0)

The vertices are points on the ellipse, collinear with the center and
foci. If y = 0, then x = a. Each vertex is a units away from the center.
The segment V1V2 is called the major axis. Its length is 2a. It divides
the ellipse into two congruent parts.

0 covertices: W1(0; b) and W2(0; b)

The segment through the center, perpendicular to the major axis, is


the minor axis. It meets the ellipse at the covertices. It divides the
ellipse into two congruent parts.
If x = 0, then y = b. Each covertex is b units away from the center.
The minor axis W1W2 is 2b units long. Since a > b, the major axis is
longer than the minor axis.

Example 1.3.1. Give the coordinates of the foci, vertices, and covertices of
the ellipse with equation
2 2
x + y = 1:
25 9
Sketch the graph, and include these points.

35
All rights reserved. No part of this material may be reproduced or transmitted in any form or by any means -
electronic or mechanical including photocopying – without written permission from the DepEd Central Office. First Edition, 2016.
p
2 2 2 2
Solution. With a = 25 and b = 9, we have a = 5, b = 3, and c = a b = 4.

foci: F1(4; 0); F2(4; 0) vertices: V1(5; 0); V2(5; 0)

covertices: W1(0; 3); W 2(0; 3)

Example 1.3.2. Find the (standard) equation of the ellipse whose foci are
F1(3; 0) and F2(3; 0), such that for any point on it, the sum of its distances
from the foci is 10. See Figure 1.36.
p 2
Solution. We have 2a = 10 and c = 3, so a = 5 and b = a2 c = 4. The
equation is 2 2
x y
+ = 1: 2
25 16

1.3.2. More Properties of Ellipses

The ellipses we have considered so far are \horizontal" and have the origin
as their centers. Some ellipses have their foci aligned vertically, and some
have centers not at the origin. Their standard equations and properties are
given in the box. The derivations are more involved, but are similar to the
one above, and so are not shown anymore.
p
2 2
In all four cases below, a > b and c = a b . The foci F1 and F2 are c units
away from the center. The vertices V 1 and V2 are a units away from the
center, the major axis has length 2a, the covertices W 1 and W2 are b units
away from the center, and the minor axis has length 2b. Recall that, for any
point on the ellipse, the sum of its distances from the foci is 2a.

36
All rights reserved. No part of this material may be reproduced or transmitted in any form or by any means -
electronic or mechanical including photocopying – without written permission from the DepEd Central Office. First Edition, 2016.
Center Corresponding Graphs

(0; 0)

2 2 2 2
x y x y
2 2 2 2
a +b = 1, a > b b +a = 1, b > a

(h; k)

2 2 2 2
(x h) + (y k) =1 (x h) + (y k) =1
a2 b2 b2 a2
a>b b>a
major axis: horizontal major axis: vertical
minor axis: vertical minor axis: horizontal

In the standard equation, if the x-part has the bigger denominator, the ellipse
is horizontal. If the y-part has the bigger denominator, the ellipse is vertical.

Example 1.3.3. Give the coordinates of the center, foci, vertices, and covertices of
the ellipse with the given equation. Sketch the graph, and include these points.

37
All rights reserved. No part of this material may be reproduced or transmitted in any form or by any means -
electronic or mechanical including photocopying – without written permission from the DepEd Central Office.
First Edition, 2016.
2 2
(1) (x + 3) + (y 5) =1
24 49
2 2
(2) 9x + 16y 126x + 64y = 71
Solution. (1) From a 2 = 49 and b 2 p
= 24, we have a = 7, b = 2 6 4:9, and

p
c= a2 b2 = 5. The ellipse is vertical.
center: (3; 5)
foci: F1(3; 0), F2(3; 10)
vertices: V1(3; 2), V2(3; 12)
p
covertices: W1(3 2 6; 5) (7:9; 5)
p

W2(3 + 2 6; 5) (1:9; 5)

(2) We rst change the given equation to standard form.


2 2
9(x 14x) + 16(y + 4y) = 71
2 2
9(x 14x + 49) + 16(y + 4y + 4) = 71 + 9(49) + 16(4)
2 2
9(x 7) + 16(y + 2) = 576
2 2
(x 7) + (y + 2) =1
64 36
38
All rights reserved. No part of this material may be reproduced or transmitted in any form or by any means -
electronic or mechanical including photocopying – without written permission from the DepEd Central Office.
First Edition, 2016.
p 2 2 p
We have a = 8 and b = 6. Thus, c = a b = 2 7 5:3. The ellipse is
horizontal.

center: (7; 2)
p
foci: F1(7 2 7; 2) (1:7; 2)
p
F2(7 + 2 7; 2) (12:3; 2)
vertices: V1(1; 2), V2(15; 2)

covertices: W1(7; 8), W2(7; 4)

Example 1.3.4. The foci of an ellipse are (3; 6) and (3; 2). For any point on
the ellipse, the sum of its distances from the foci is 14. Find the standard
equation of the ellipse.

Solution. The midpoint (3; 2) of the foci is the center of the ellipse. The
ellipse is vertical (because the foci are vertically aligned) and c = 4. From the
given sum, 2a = 14 so a = 7. Also, b = p a2 c2 = p 33. The equation is
2 2
(x + 3) + (y + 2) = 1. 2
33 49
p
Example 1.3.5. An ellipse has vertices (2 p 61; 5) and (2 + 61; 5), and
its minor axis is 12 units long. Find its standard equation and its foci.

39
All rights reserved. No part of this material may be reproduced or transmitted in any form or by any means -
electronic or mechanical including photocopying – without written permission from the DepEd Central Office. First Edition, 2016.
Solution. The midpoint (2; 5) of the vertices is the center of the ellipse, which is p
horizontal. Each vertex is a = 61 units away from the center. From the length of
2 2
the minor axis, 2b = 12 so b = 6. The standard equation is (x 2) + (y + 5) =
p 2 2 61 36
1. Each focus is c = a b = 5 units away from (2; 5), so their coordinates
are (3; 5) and (7; 5). 2

1.3.3. Situational Problems Involving Ellipses

Let us now apply the concept of ellipse to some situational problems.


?
Example 1.3.6. A tunnel has the shape of a semiellipse that is 15 ft high at
the center, and 36 ft across at the base. At most how high should a passing
truck be, if it is 12 ft wide, for it to be able to t through the tunnel? Round o
your answer to two decimal places.

Solution. Refer to the gure above. If we draw the semiellipse on a rectangular coordinate system,
with its center at the origin, an equation of the ellipse which contains it, is
x2 y2
2 = 1: 2 +
18 15
To maximize its height, the corners of the truck, as shown in the gure, would
have to just touch the ellipse. Since the truck is 12 ft wide, let the point (6; n)
be the corner of the truck in the rst quadrant, where n > 0, is the (maximum)
height of the truck. Since this point is on the ellipse, it should t the equation.
Thus, we have
2 2
6 n
2 + 2 =1
18 15 p

n = 10 2 14:14 ft 2

40
All rights reserved. No part of this material may be reproduced or transmitted in any form or by any means -
electronic or mechanical including photocopying – without written permission from the DepEd Central Office. First Edition, 2016.
Example 1.3.7. The orbit of a planet has the shape of an ellipse, and on one
of the foci is the star around which it revolves. The planet is closest to the
star when it is at one vertex. It is farthest from the star when it is at the other
vertex. Suppose the closest and farthest distances of the planet from this
star, are 420 million kilometers and 580 million kilometers, respectively. Find
the equation of the ellipse, in standard form, with center at the origin and the
star at the x-axis. Assume all units are in millions of kilometers.

Solution. In the gure above, the orbit is drawn as a horizontal ellipse with
center at the origin. From the planet’s distances from the star, at its closest
and farthest points, it follows that the major axis is 2a = 420 + 580 = 1000
(million kilometers), so a = 500. If we place the star at the positive x-axis,
2
then it is c = 500 420 = 80 units away from the center. Therefore, we get b =
2 2 2 2
a c = 500 80 = 243600. The equation then is
2 2
x y
+ = 1:
250000 243600
The star could have been placed on the negative x-axis, and the answer would
still be the same. 2

41
All rights reserved. No part of this material may be reproduced or transmitted in any form or by any means -
electronic or mechanical including photocopying – without written permission from the DepEd Central Office. First Edition, 2016.
More Solved Examples
ᜀĀᜀĀᜀĀᜀĀᜀĀᜀĀᜀĀᜀȀȀ̀⠀⤀ĀᜀĀᜀĀᜀĀᜀĀᜀĀᜀĀ
ᜀĀᜀ0 Give the coordinates of the foci,
vertices, and covertices of the ellipse with
x2 y2
equa-tion + = 1. Then sketch the
169 144
graph and include these points.
Solution: The ellipse is horizontal.
a2 = 169 ) a = 13, b2 = 144 ) b = 12,
p
c= 169 144=5
Foci: F1(5; 0), F2(5; 0)
Vertices: V1(13; 0), V2(13; 0)
Covertices: W1(0; 12), W2 (0; 12)
See Figure 1.38. Figure 1.38

2. Find the standard equation of the ellipse whose foci are F 1(0; 8) and F2(0;
8), such that for any point on it, the sum of its distances from the foci is 34.
Solution: The ellipse is vertical and has center at (0; 0).

2a = 34 ) a = 17
p 2 2
c=8)b= 17 8 = 15
x2 y2
The equation is + = 1.
225 289

For Examples 3 and 4, give the coordinates of the center, foci, vertices,
and covertices of the ellipse with the given equation. Sketch the graph, and
include these points.
0 (x 7) 2
+ (y + 2) = 1
2

6425
Solution: The ellipse is
2 2
horizontal. a =p64 ) a = 8,pb =
25 ) b = 5 c = 64 25 = 39 6:24
center: (7; 2)
p
F
foci: 1 (7 39; 2) (0:76; 2)
p
F2 (7 + 39; 2) (13:24; 2)
vertices: V1(1; 2); V 2(15; 2)
Figure 1.39
covertices: W1(7; 7); W 2(7; 3)
See Figure 1.39.

42
All rights reserved. No part of this material may be reproduced or transmitted in any form or by any means -
electronic or mechanical including photocopying – without written permission from the DepEd Central Office. First Edition, 2016.
2 2
0 16x + 96x + 7y + 14y + 39 = 0
Solution:
2 2
16x + 96x + 7y + 14y = 39
2 2
16(x + 6x + 9) + 7(y + 2y + 1) = 39 + 151
2 2
16(x + 3) + 7(y + 1) = 112
2 2
(x + 3) (y + 1)
+ =1
7 16
The ellipse is vertical.
p
a2 = 16 ) a = 4, b2 = 7 ) b = 7
2:65
p
c= 16 7=3
center: (3; 1)
foci: F1(3; 4), F2(3; 2)
vertices: V1(3; 5); V 2(3; 3)
p Figure 1.40
covertices: W1(3 7; 1) (5:65; 1)
p
W2(3 + 7; 1) (0:35; 1)

23 The covertices of an ellipse are (5; 6) and (5; 8). For any point on the
ellipse, the sum of its distances from the foci is 12. Find the standard
equation of the ellipse.
Solution: The ellipse is horizontal with center at the midpoint (5; 7) of the
2
covertices. Also, 2a = 12 so a = 6 while b = 1. The equation is (x 5) +
2
36
(y 7) = 1.
1
6. An ellipse has foci (4 p 15; 3) and (4 + p 15; 3), and its major axis is 10
units long. Find its standard equation and its vertices.
Solution: The ellipse is horizontal with center at the midpoint (4; 3) of the
p
foci; also c = 15. Since the length of the major axis is 10, 2a = 10 and
p 2
p
a = 5. Thus b = 5 15 = 10. Therefore, the equation of the ellipse is
(y 3)2
= 1 and its vertices are (9; 3) and (1; 3).
2510
0 A whispering gallery is an enclosure or room where whispers can be clearly
heard in some parts of the gallery. Such a gallery can be constructed by making
its ceiling in the shape of a semi-ellipse; in this case, a whisper from one focus
(x+4)2
can be clearly heard at the other focus. If an elliptical whispering

43
All rights reserved. No part of this material may be reproduced or transmitted in any form or by any means -
electronic or mechanical including photocopying – without written permission from the DepEd Central Office. First Edition, 2016.
gallery is 90 feet long and the foci are 50 feet apart, how high is the
gallery at its center?

Solution: We set up a Cartesian coordinate system by assigning the center of the


semiellipse as the origin. The point on the ceiling right above the center is a
covertex of the ellipse. Since 2a = 90 and 2c = 50; then b 2 = 452 252 = 1400.
p
The height is given by b = 1400 37:4 ft.

0 A spheroid (or oblate spheroid) is the surface obtained by rotating an ellipse


around its minor axis. The bowl in Figure 1.41 is in the shape of the lower half
of a spheroid; that is, its horizontal cross sections are circles while its vertical
cross sections that pass through the center are semiellipses. If this bowl is 10 p
in wide at the opening and 10 in deep at the center, how deep does a
circular cover with diameter 9 in go into the bowl?

Figure 1.41

Solution: We set up a Cartesian coordinate system by assigning the center p


of the semiellipse as the origin. Then a = 5, b = 10, and the equation of
x y
the ellipse is 252 + 102 = 1. We want the y-coordinate of the points on the
q
2
x
ellipse that has x = 4:5. This coordinate is y = 10 1 25 1:38.

Therefore, the cover will go 1.38 inches into the bowl.

44
All rights reserved. No part of this material may be reproduced or transmitted in any form or by any means -
electronic or mechanical including photocopying – without written permission from the DepEd Central Office. First Edition, 2016.
Supplementary Problems 1.3
Give the coordinates of the center, foci, vertices, and covertices of the ellipse
with the given equation. Sketch the graph, and include these points.

x2 + y2 = 1 8 4
2 2
x (y 2)
2. + =1
16 25
2 2
3. (x 1) + (2y 2) =4
2 2
4. (x + 5) + (y 2) =1
49 121
2 2
5. 16x 224x + 25y + 250y 191 = 0
2 2
6. 25x 200x + 16y 160y = 800

Find the standard equation of the ellipse which satis es the given conditions.
p
7. foci (2 p 33; 8) and (2 + 33; 8), the sum of the distances of any point from
the foci is 14

8. center (3; 7), vertical major axis of length 20, minor axis of length 12

9. foci (21; 10) and (3; 10), contains the point (9; 15)

0 a vertex at (3; 18) and a covertex at ( 12; 7), major axis is either hori-
zontal or vertical

1 a focus at (9; 15) and a covertex at (1; 10), with vertical major axis

2 A 40-ft wide tunnel has the shape of a semiellipse that is 5 ft high a distance
of 2 ft from either end. How high is the tunnel at its center?

3 The moon’s orbit is an ellipse with Earth as one focus. If the maximum
distance from the moon to Earth is 405 500 km and the minimum distance
is 363 300 km, nd the equation of the ellipse in a Cartesian coordinate
system where Earth is at the origin. Assume that the ellipse has horizontal
major axis and that the minimum distance is achieved when the moon is
to the right of Earth. Use 100 km as one unit.

4 Two friends visit a whispering gallery (in the shape of a semiellipsoid) where
they stand 100 m apart to be at the foci. If one of them is 6 m from the
nearest wall, how high is the gallery at its center?

45
All rights reserved. No part of this material may be reproduced or transmitted in any form or by any means -
electronic or mechanical including photocopying – without written permission from the DepEd Central Office. First Edition, 2016.
0 A jogging path is in the shape of an ellipse. If it is 120 ft long and 40 ft
wide, what is the width of the track 15 ft from either vertex?

1 Radiation is focused to an unhealthy area in a patient’s body using a


semiel-liptic re ector, positioned in such a way that the target area is at
one focus while the source of radiation is at the other. If the re ector is 100
cm wide and 30 cm high at the center, how far should the radiation source
and the target area be from the ends of the re ector?

Lesson 1.4. Hyperbolas

Learning Outcomes of the Lesson


At the end of the lesson, the student is able to:
0 de ne a hyperbola;
1 determine the standard form of equation of a hyperbola;
2 graph a hyperbola in a rectangular coordinate system; and
3 solve situational problems involving conic sections (hyperbolas).

Lesson Outline
0 De nition of a hyperbola
1 Derivation of the standard equation of a hyperbola
2 Graphing hyperbolas
3 Solving situational problems involving hyperbolas

Introduction
Just like ellipse, a hyperbola is one of the conic sections that most students
have not encountered formally before. Its graph consists of two unbounded
branches which extend in opposite directions. It is a misconception that each
branch is a parabola. This is not true, as parabolas and hyperbolas have very di
erent features. An application of hyperbolas in basic location and navigation
schemes are presented in an example and some exercises.

1.4.1. De nition and Equation of a Hyperbola

Consider the points F1(5; 0) and F2(5; 0) as shown in Figure 1.42. What is the
absolute value of the di erence of the distances of A(3:75; 3) from F 1 and from

46
All rights reserved. No part of this material may be reproduced or transmitted in any form or by any means -
electronic or mechanical including photocopying – without written permission from the DepEd Central Office.
First Edition, 2016.
from F1 and from F2? 3
16
F2? How about the absolute value of the di erence of the distances of B 5;

jAF1 AF2j = j9:25 3:25j = 6


jBF1 BF2j = 3 3 =6
16 34

There are other points P such that jP F 1 P F2j = 6. The collection of all such
points forms a shape called a hyperbola, which consists of two disjoint
branches. For points P on the left branch, P F 2 P F1 = 6; for those on the
right branch, PF1 PF2 = 6.

Figure 1.42

Figure 1.43

47
All rights reserved. No part of this material may be reproduced or transmitted in any form or by any means -
electronic or mechanical including photocopying – without written permission from the DepEd Central Office. First Edition, 2016.
Let F1 and F2 be two distinct points. The set of all points P , whose
distances from F1 and from F2 di er by a certain constant, is called a
hyperbola. The points F1 and F2 are called the foci of the hyperbola.

In Figure 1.43, given are two points on the x-axis, F 1(c; 0) and F2(c; 0), the
foci, both c units away from their midpoint (0; 0). This midpoint is the center of
the hyperbola. Let P (x; y) be a point on the hyperbola, and let the absolute
value of the di erence of the distances of P from F 1 and F2, be 2a (the coe cient
2 will make computations simpler). Thus, jP F1 P F2j = 2a, and so
p p = 2a: (x + c)2 + y2 (x c)2 + y2

Algebraic manipulations allow us to rewrite this into the much simpler


2 2 p
x y = 1; where b = c
2
a :
2

2 2
a b
p
When we let b = c2 a2, we assumed c > a. To see why this is true, suppose
that P is closer to F 2, so P F1 P F2 = 2a. Refer to Figure 1.43. Suppose also that
P is not on the x-axis, so 4P F1 F2 is formed. From the triangle inequality,
F1F2 + P F2 > P F1. Thus, 2c > P F1 P F2 = 2a, so c > a.
Now we present a derivation. For now, assume P is closer to F2 so P F1 > P F2,
and P F1 P F2 = 2a.
P F1 = 2a + P F2
p
p =2 + 2
x c a (x c 2
+
( +2 ) 2 2 ) 2 2

2 y2 2 +y

p
(x + c) + y + (x c) + y
p 2 = 2a 2 2 2 2 2
cx 2

a
2 =
a p
x
(
c) + y

2 2 (2 2)2 = 2 2( 2 )+
(c a )x ay =a ( c
p
a )
cx a a x c y

2 2 2 2 2 2
p
bx a y =a b by letting b = c2 a2 > 0
x2 y2

a2 b2 = 1
We collect here the features of the graph of a hyperbola with standard equa-tion
2 2
x y
2 2
a b = 1:
p 2 2
Let c = a +b .
48
All rights reserved. No part of this material may be reproduced or transmitted in any form or by any means -
electronic or mechanical including photocopying – without written permission from the DepEd Central Office. First Edition, 2016.
Figure 1.44 Figure 1.45

0 center: origin (0; 0)


1 foci: F1(c; 0) and F2(c; 0)

Each focus is c units away from the center.


For any point on the hyperbola, the absolute value of the di erence
of its distances from the foci is 2a.

(3) vertices: V1(a; 0) and V2(a; 0)

The vertices are points on the hyperbola, collinear with the center
and foci.
If y = 0, then x = a. Each vertex is a units away from the center.
The segment V1V2 is called the transverse axis. Its length is 2a.
(4) asymptotes: y = bx and y = b x, the lines ‘1 and ‘2 in Figure 1.45
a a

The asymptotes of the hyperbola are two lines passing through the
cen-ter which serve as a guide in graphing the hyperbola: each
branch of the hyperbola gets closer and closer to the asymptotes, in
the direction towards which the branch extends. (We need the
concept of limits from calculus to explain this.)
An aid in determining the equations of the asymptotes: in the standard
2
equation, replace 1 by 0, and in the resulting equation x2
2
y
2
= 0, solve
a b
for y.
To help us sketch the asymptotes, we point out that the asymptotes ‘1 and
‘2 are the extended diagonals of the auxiliary rectangle drawn in Figure
1.45. This rectangle has sides 2a and 2b with its diagonals intersecting at
the center C. Two sides are congruent and parallel to the transverse axis
V1V2. The other two sides are congruent and parallel

49
All rights reserved. No part of this material may be reproduced or transmitted in any form or by any means -
electronic or mechanical including photocopying – without written permission from the DepEd Central Office. First Edition, 2016.
to the conjugate axis, the segment shown which is perpendicular to
the transverse axis at the center, and has length 2b.

Example 1.4.1. Determine the foci, vertices, and asymptotes of the hyperbola
with equation
2
x y2 = 1:
7
Sketch the graph, and include these points and lines, the transverse and
conjugate axes, and the auxiliary rectangle.
2 2
Solution. With a = 9 and b = 7, we have
p p
2 2
a = 3, b = 7, and c = a + b = 4.
foci: F1(4; 0) and F2(4; 0)
vertices: V1(3; 0) and V2(3; 0)
p p
7
asymptotes: y = 3 x and y = 3
7
x
The graph is shown at the right. The conju-
p

gate axis drawn has its endpoints b = 7


2:7 units above and below the center. 2
Example 1.4.2. Find the (standard) equation of the hyperbola whose foci are
F1(5; 0) and F2(5; 0), such that for any point on it, the absolute value of the di
erence of its distances from the foci is 6. See Figure 1.42.
p
Solution. We have 2a = 6 and c = 5, so a = 3 and b = c
2 2
a = 4. The
2 2
x y
hyperbola then has equation = 1. 2
9 16

1.4.2. More Properties of Hyperbolas

The hyperbolas we considered so far are \horizontal" and have the origin as
their centers. Some hyperbolas have their foci aligned vertically, and some
have centers not at the origin. Their standard equations and properties are
given in the box. The derivations are more involved, but are similar to the
one above, and so are not shown anymore.
p
2 2
In all four cases below, we let c = a + b . The foci F1 and F2 are c units away
from the center C. The vertices V 1 and V2 are a units away from the center. The
transverse axis V1V2 has length 2a. The conjugate axis has length 2b and is
perpendicular to the transverse axis. The transverse and conjugate axes bisect
each other at their intersection point, C. Each branch of a hyperbola gets closer and
closer to the asymptotes, in the direction towards which the branch extends. The
equations of the asymptotes can be determined by replacing 1 in the standard
equation by 0. The asymptotes can be drawn as the extended diagonals of the
auxiliary rectangle determined by the transverse and conjugate axes. Recall that,

50
All rights reserved. No part of this material may be reproduced or transmitted in any form or by any means -
electronic or mechanical including photocopying – without written permission from the DepEd Central Office. First Edition, 2016.
for any point on the hyperbola, the absolute value of the di erence of its
distances from the foci is 2a.

Center Corresponding Hyperbola

(0; 0)

x2 y2 y2 x2
2 2 2 2
a b =1 a b =1

(h; k)

2 2 2 2
(x h) (y k) =1 (y k) (x h) =1
2 2 2 2
a b a b

transverse axis: horizontal transverse axis: vertical


conjugate axis: vertical conjugate axis: horizontal

In the standard equation, aside from being positive, there are no other re-
strictions on a and b. In fact, a and b can even be equal. The orientation of the
hyperbola is determined by the variable appearing in the rst term (the positive
term): the corresponding axis is where the two branches will open. For example,
if the variable in the rst term is x, the hyperbola is \horizontal": the transverse

51
All rights reserved. No part of this material may be reproduced or transmitted in any form or by any means -
electronic or mechanical including photocopying – without written permission from the DepEd Central Office. First Edition, 2016.
axis is horizontal, and the branches open to the left and right in the direction
of the x-axis.

Example 1.4.3. Give the coordinates of the center, foci, vertices, and asymp-
totes of the hyperbola with the given equation. Sketch the graph, and include
these points and lines, the transverse and conjugate axes, and the auxiliary
rect-angle.
2 2
(1) (y + 2) (x 7) =1
25 9
2 2
(2) 4x 5y + 32x + 30y = 1
2 2
Solution. (1) From a = 25 and b = 9, we have a = 5, b = 3, and c =
p p
2 2
a + b = 34 52:8. The 2hyperbola is vertical. To determine the asymp-
(y+2)(x7) 5
totes, we write 25 9 = 0, which is equivalent to y + 2 = 3 (x 7).
We can then solve this for y.
center: C(7; 2) p p
foci: F1(7; 2 34) (7; 7:8) and F2(7; 2 + 34) (7; 3:8)
vertices: V1(7; 7) and V2 (7; 3)
asymptotes: y = 5 x 41 and y = 5 x + 29
3 3 3 3

The conjugate axis drawn has its endpoints b = 3 units to the left and
right of the center.

52
All rights reserved. No part of this material may be reproduced or transmitted in any form or by any means -
electronic or mechanical including photocopying – without written permission from the DepEd Central Office. First Edition, 2016.
(2) We rst change the given equation to standard form.
2 2
4(x + 8x) 5(y 6y) = 1
2 2
4(x + 8x + 16) 5(y 6y + 9) = 1 + 4(16) 5(9)
2 2
4(x + 4) 5(y 3) = 20
2 2
(x + 4) (y 3) = 1
0 4
a p : b c p a2 b2 3. The hyperbola
=
We have = 5 2 2 and = 2. Thus, = + (x+4)2 (y3) 2
is horizontal. To determine the asymptotes, we write 5 4 =0
2

which is equivalent to y 3 = p 5 (x + 4), and solve for y.


center: C(4; 3)
foci: F1(7; 3) and F2(1; 3)
p p
vertices: V1(4 5; 3) (6:2; 3) and V2(4 + 5; 3) (1:8; 3)
2 8 2 8
asymptotes: y = p 5x + p 5 + 3 and y = p 5 x p5 +3
The conjugate axis drawn has its endpoints b = 2 units above and below
the center.

Example 1.4.4. The foci of a hyperbola are (5; 3) and (9; 3). For any point on
the hyperbola, the absolute value of the di erence of its of its distances from
the foci is 10. Find the standard equation of the hyperbola.

53
All rights reserved. No part of this material may be reproduced or transmitted in any form or by any means -
electronic or mechanical including photocopying – without written permission from the DepEd Central Office. First Edition, 2016.
Solution. The midpoint (2; 3) of the foci is the center of the hyperbola. Each
focus is c = 7 units away from the center. From the given di erence, 2a = 10
2 2 2
so a = 5. Also, b = c a = 24. The hyperbola is horizontal (because the foci
are horizontally aligned), so the equation is
2 2
(x 2) (y + 3)
= 1: 2
25 24
Example 1.4.5. A hyperbola has vertices (4; 5) and (4; 9), and one of its
foci is (4; 2 p 65). Find its standard equation.

Solution. The midpoint (4; 2) of the vertices is the center of the hyperbola,
which is vertical (because the vertices are vertically aligned). Each vertex is p
a = 7 units away from the center. The given focus is c = 65 units away from
2 2 2
the center. Thus, b = c a = 16, and the standard equation is
2 2
(y 2) (x + 4)
= 1: 2
49 16

1.4.3. Situational Problems Involving Hyperbolas

Let us now give an example on an application of hyperbolas.


Example 1.4.6. An explosion was heard by two stations 1200 m apart,
located at F1(600; 0) and F2(600; 0). If the explosion was heard in F 1 two
seconds before it was heard in F 2, identify the possible locations of the
explosion. Use 340 m/s as the speed of sound.

Solution. Using the given speed of sound, we can deduce that the sound
traveled 340(2) = 680 m farther in reaching F 2 than in reaching F1. This is then
the di erence of the distances of the explosion from the two stations. Thus, the
explosion is on a hyperbola with foci are F1 and F2, on the branch closer to F1.

54
All rights reserved. No part of this material may be reproduced or transmitted in any form or by any means -
electronic or mechanical including photocopying – without written permission from the DepEd Central Office. First Edition, 2016.
We have c = 600 and 2a = 680, so a = 340 and b2 = c2 a2 = 244400. The
explosion could therefore be anywhere on the left branch of the hyperbola
2 2
x y
115600 244400 = 1. 2

More Solved Examples


1. Determine the foci, vertices, and asymptotes of the hyperbola with equation
2 2
x y

16 33 = 1. Sketch the graph, and include these points and lines, the
transverse and conjugate axes, and the auxiliary rectangle.
Solution: The hyperbola is horizontal.
2
a = 16 ) a = 4,
2
b = 33 ) b = p 33,
c = p 16+33 = 7
center: (0; 0)
foci: F1(7; 0), F2(7; 0)
V (4;
vertices: 1 p 0), V2(4; 0)p
asymptotes: y = 33 x, y = 33 x
4 4
The conjugate axis has endpoints
p
(0; p 33) and (0; 33). See Figure Figure 1.46
1.46.
ᜀĀᜀĀᜀĀᜀĀᜀĀᜀĀᜀĀᜀȀȀ̀⠀⤀ĀᜀĀᜀĀᜀĀᜀĀᜀĀᜀĀᜀĀᜀ0 Find the standard
equation of the hyperbola whose foci are F 1(0; 10) and F2(0; 10), such
that for any point on it, the absolute value of the di erence of its distances
from the foci is 12.
Solution: The hyperbola is vertical and has center at (0; 0). We have 2a = 12,
2 2
p 2 2 y x
so a = 6; also, c = 10. Then b = 10 6 = 8. The equation is = 1.

36 64
For Examples 3 and 4, give the coordinates of the center, foci, vertices, and asymptotes of the
hyperbola with the given equation. Sketch the graph, and in-clude these points and lines, the
transverse and conjugate axes, and the auxiliary
rectangle.

2 2
(y + 6) (x 4)
3. =1
25 39
Solution: The hyperbola is vertical.

2 2 p p
a = 25 ) a = 5, b = 39 ) b = 39, c = 25+39=8
55
All rights reserved. No part of this material may be reproduced or transmitted in any form or by any means -
electronic or mechanical including photocopying – without written permission from the DepEd Central Office. First Edition, 2016.
center: (4; 6)
foci: F1(4; 14), F2(4; 2)
vertices: V1(4; 11), V2(4; 1)
2 2
(y + 6) (x 4)
asymptotes: =0
25 39
5
,y+6= p 39 (x 4)
p
The conjugate axis has endpoints b = 39 units to the left and to the right of
the center. See Figure 1.47.

Figure 1.47

2 2
4. 9x + 126x 16y 96y + 153 = 0
Solution:
2 2
9x + 126x 16y 96y = 153
2 2
9(x + 14x + 49) 16(y + 6y + 9) = 153 + 9(49)16(9)
2 2
9(x + 7) 16(y + 3) = 144
2 2
(x + 7) (y + 3) = 1
0 9
The hyperbola is horizontal.
p
2 2
a = 16 ) a = 4; b = 9 ) b = 3; c= 16 + 9 = 5

56
All rights reserved. No part of this material may be reproduced or transmitted in any form or by any means -
electronic or mechanical including photocopying – without written permission from the DepEd Central Office. First Edition, 2016.
center: (7; 3)
foci: F1(12; 3), F2(2; 3)
vertices: V1(11; 3), V2(3; 3)
2 2
(x + 7) (y + 3) 3
asymptotes: 16 9 = 0 , y + 3 = 4 (x + 7)

The conjugate axis have endpoints (7; 6) and ( 7; 0). See Figure 1.48.

Figure 1.48

0 The foci of a hyperbola are (17; 3) and (3 ; 3). For any point on the
hyperbola, the absolute value of the di erence of its distances from the foci
is 14. Find the standard equation of the hyperbola.
Solution: The hyperbola is horizontal with center at the midpoint (7; 3) of
2 2 2
the foci. Also, 2a = 14 so a = 7 while c = 10. Then b = 10 7 = 51. The
2 2
equation is (x + 7) (y + 3) = 1.
49 51
6. The auxiliary rectangle of a hyperbola has vertices (24; 15), (24; 9), (10; 9),
and (10; 15). Find the equation of the hyperbola if its conjugate axis is
hor-izontal.
Solution: The hyperbola is vertical. Using the auxiliary rectangle’s dimen-
sions, we see that the length of the transverse axis is 2a = 24 while the
length of the conjugate axis is 2b = 34. Thus, a = 12 and b = 17. The
hyperbola’s vertices are the midpoints (7; 15) and ( 7; 9) of the bottom

57
All rights reserved. No part of this material may be reproduced or transmitted in any form or by any means -
electronic or mechanical including photocopying – without written permission from the DepEd Central Office. First Edition, 2016.
and top sides, respectively, of the auxiliary rectangle. Then the hyperbola’s
center is (7; 3), which is the midpoint of the vertices. The equation is (y +
2 2
3) (x + 7)
0.0.0 1.
144289
1 Two LORAN (long range navigation) stations A and B are situated along a
straight shore, where A is 200 miles west of B. These stations transmit
radio signals at a speed 186 miles per millisecond. The captain of a ship
travelling on the open sea intends to enter a harbor that is located 40
miles east of station A.
Due to the its location, the harbor experiences a time di erence in
receiving the signals from both stations. The captain navigates the ship
into the harbor by following a path where the ship experiences the same
time di erence as the harbor.

0 What time di erence between station signals should the captain be look-ing
for in order the ship to make a successful entry into the harbor?
1 If the desired time di erence is achieved, determine the location of the
ship if it is 75 miles o shore.

Solution:

(a) Let H represent the harbor on the shoreline. Note that BH AH = 160
40 = 120. The time di erence on the harbor is given by 120 186 0:645
milliseconds. This is the time di erence needed to be maintained in order
to for the ship to enter the harbor.
(b) Situate the stations A and B on the Cartesian plane so that A (100; 0)
and B (100; 0). Let P represent the ship on the sea, which has coordinates
(h; 75). Since P B P A = 120, then it should follow that h < 0. More-
over, P should lie on the left branch of the hyperbola whose equation is
given by 2 2
x y
a2 b2 =1
p 2 2
p 2 2
where 2a = 120 ) a = 60, and b = c a = 100 60 = 80.
Therefore,
2 2
h 75
2 2 = 1
60 80 s
h = 82:24
2
1 + 802 602

75

This means that the ship is around 17.76 miles to the east of station A.

58
All rights reserved. No part of this material may be reproduced or transmitted in any form or by any means -
electronic or mechanical including photocopying – without written permission from the DepEd Central Office. First Edition, 2016.
Supplementary Problems 1.4
Give the center, foci, vertices, and asymptotes of the hyperbola with the
given equation. Sketch the graph and the auxiliary rectangle, then include
these points and lines.
2 2
x y
2 2
1. 100 81 = 1 3. 4x 15(y 5) = 60
1 2 2
(y 6) (x 8)
2. y x = 4. =1
y+x 64 36
2 2
5. 9y + 54y 6x 36x 27 = 0
2 2
6. 16x + 64x 105y + 840y 3296 = 0

Find the standard equation of the hyperbola which satis es the given conditions.

7. foci (7; 17) and (7; 17), the absolute value of the di erence of the distances
of any point from the foci is 24

8. foci (3; 2) and (15; 2), a vertex at (9; 2)

9. center (10; 4), one corner of auxiliary rectangle at (1; 12), with horizontal
transverse axis
10. asymptotes y = 71 4x and y = 4 x 17 and a vertex at (17; 9)
3 3 3 3
11. asymptotes y = 5 x+ 19 and y = 5x + 29 and a focus at (4; 5)
12 3 12 3

0 horizontal conjugate axis, one corner of auxiliary rectangle at (3; 8), and
an asymptote 4x + 3y = 12

1 two corners of auxiliary rectangle at (2; 3) and (16; 1), and horizontal
trans-verse axis

2 Two radio stations are located 150 miles apart, where station A is west of
sta-tion B. Radio signals are being transmitted simultaneously by both
stations, travelling at a rate of 0.2 miles/ sec. A plane travelling at 60 miles
above ground level has just passed by station B and is headed towards
the other station. If the signal from B arrives at the plane 480 sec before
the signal sent from A, determine the location of the plane.

59
All rights reserved. No part of this material may be reproduced or transmitted in any form or by any means -
electronic or mechanical including photocopying – without written permission from the DepEd Central Office. First Edition, 2016.
Lesson 1.5. More Problems on Conic Sections

Learning Outcomes of the Lesson


At the end of the lesson, the student is able to:
0 recognize the equation and important characteristics of the di erent types of
conic sections; and
1 solve situational problems involving conic sections.

Lesson Outline
0 Conic sections with associated equations in general form
1 Problems involving characteristics of various conic sections
2 Solving situational problems involving conic sections

Introduction
In this lesson, we will identify the conic section from a given equation. We
will analyze the properties of the identi ed conic section. We will also look at
problems that use the properties of the di erent conic sections. This will allow
us to synthesize what has been covered so far.

1.5.1. Identifying the Conic Section by Inspection

The equation of a circle may be written in standard form


2 2
Ax + Ay + Cx + Dy + E = 0;
2 2
that is, the coe cients of x and y are the same. However, it does not follow
2 2
that if the coe cients of x and y are the same, the graph is a circle.
General Equation Standard Equation graph
2 2
(A)
(B)
2x + 2y
x2 + y2
2x + 6y + 5 = 0
6x 8y + 50 = 0
x 1 2+y+ 3
(x 3)2 + (y 4)2
2
=0
= 25
point
empty set

2 2

2 2 2 2
For a circle with equation (x h) + (y k) = r , we have r > 0. This is
not the case for the standard equations of (A) and (B).
In (A), because the sum of two squares can only be 0 if and only if each square
is 0, it follows that x 1 = 0 and y + 3 = 0. The graph is thus the single point
1 3 2 2

2 ; 2 .
60
All rights reserved. No part of this material may be reproduced or transmitted in any form or by any means -
electronic or mechanical including photocopying – without written permission from the DepEd Central Office. First Edition, 2016.
In (B), no real values of x and y can make the nonnegative left side equal
to the negative right side. The graph is then the empty set.
Let us recall the general form of the equations of the other conic sections.
We may write the equations of conic sections we discussed in the general form
2 2
Ax + By + Cx + Dy + E = 0:

Some terms may vanish, depending on the kind of conic section.


2 2
0 Circle: both x and y appear, and their coe cients are the same
2 2
Ax + Ay + Cx + Dy + E = 0
2 2
Example: 18x + 18y 24x + 48y 5 = 0
Degenerate cases: a point, and the empty set
2 2
(2) Parabola: exactly one of x or y appears
2
Ax + Cx + Dy + E = 0 (D 6= 0, opens upward or downward)
2
By + Cx + Dy + E = 0 (C 6= 0, opens to the right or left)
2
Examples: 3x 12x + 2y + 26 = 0 (opens downward)
2
2y + 3x + 12y 15 = 0 (opens to the right)
2 2
0 Ellipse: both x and y appear, and their coe cients A and B have the
same sign and are unequal
2 2
Examples: 2x + 5y + 8x 10y 7 = 0 (horizontal major axis)
2 2
4x + y 16x 6y + 21 = 0 (vertical major axis)
If A = B, we will classify the conic as a circle, instead of an ellipse.
Degenerate cases: a point, and the empty set
2 2
5888 Hyperbola: both x and y appear, and their coe cients A and B
have dif-ferent signs
2 2
Examples: 5x 3y 20x 18y 22 = 0 (horizontal transverse axis)
2 2
4x + y + 24x + 4y 36 = 0 (vertical transverse axis)
Degenerate case: two intersecting lines
The following examples will show the possible degenerate conic (a point, two intersecting lines,
or the empty set) as the graph of an equation following a similar
pattern as the non-degenerate cases.
2 2
(x 2) (y + 1)
2 2
(1) 4x + 9y 16x + 18y + 25 = 0 =) 32 + 22 =0
=) one point: (2; 1)

61
All rights reserved. No part of this material may be reproduced or transmitted in any form or by any means -
electronic or mechanical including photocopying – without written permission from the DepEd Central Office.
First Edition, 2016.
2 2
(x 2) (y + 1)
2 2 2 2
(2) 4x + 9y 16x + 18y + 61 = 0 =) 3 + 2 =1
=) empty set
2 2
(x 2) (y + 1)
2 2
(3) 4x 9y 16x 18y + 7 = 0 =) 32 22 =0
2
=) two lines: y + 1 = 3 (x 2)

A Note on Identifying a Conic Section


by Its General Equation
It is only after transforming a given general equation to standard
form that we can identify its graph either as one of the degenerate
conic sections (a point, two intersecting lines, or the empty set) or
as one of the non-degenerate conic sections (circle, parabola,
ellipse, or hyperbola).

1.5.2. Problems Involving Di erent Conic Sections

The following examples require us to use the properties of di erent conic


sections at the same time.
2
Example 1.5.1. A circle has center at the focus of the parabola y + 16x + 4y =
44, and is tangent to the directrix of this parabola. Find its standard equation.

2
Solution. The standard equation of the parabola is (y + 2) = 16(x 3). Its
vertex is V (3; 2). Since 4c = 16 or c = 4, its focus is F (1; 2) and its directrix
is x = 7. The circle has center at (1; 2) and radius 8, which is the distance
from F to the directrix. Thus, the equation of the circle is
2 2
(x + 1) + (y + 2) = 64: 2
2 2
Example 1.5.2. The vertices and foci of 5x 4y + 50x + 16y + 29 = 0 are,
respectively, the foci and vertices of an Find the standard equation of
ellipse. this ellipse.

Solution. We rst write the equation of the hyperbola in standard form:


2 2
(x + 5) (y 2)
= 1:
16 20
For this hyperbola, using the notations ah, bh, and ch to refer to a, b, and c of
p
the standard equation of the hyperbola, respectively, we have a h = 4, bh = 2
p
5,
2 2
ch = a h +b h = 6, so we have the following points:

62
All rights reserved. No part of this material may be reproduced or transmitted in any form or by any means -
electronic or mechanical including photocopying – without written permission from the DepEd Central Office. First Edition, 2016.
center: (5; 2)
vertices: (9; 2) and (1; 2)
foci: (11; 2) and (1; 2).
It means that, for the ellipse, we have these points:
center: (5; 2)
vertices: (11; 2) and (1; 2)
foci: (9; 2) and (1; 2). p
In this case, ce = 4 and ae = 6, so that be = a 2 2
c = 20. The standard

equation of the ellipse is pe e


2 2
(x + 5) (y 2)
+ = 1: 2
36 20

More Solved Examples


1. Identify the graph of each of the following equations.
2 2 2 2
(a) 4x 8x 49y + 196y 388 = 0 (e) 36x +360x+64y 512y+1924 =
2 2
(b) x + 5x + y y+7=0 0
2 2 2
(c) y 48x + 6y = 729 (f) x +y 18y 19 = 0
2 2 2 2
(d) 49x + 196x + 100y + 1400y + (g) 5x + 60x + 7y + 84y + 72 = 0
2
196=0 (h) x 16x + 20y = 136

2 2
0 Since the coe cients of x and y have opposite signs, the graph is a
hyperbola or a pair of intersecting lines. Completing the squares, we get
2 2
4x 8x 49y + 196y 388 = 0
2 2
4(x 2x) 49(y 4y) = 388
2 2
4(x 2x + 1) 49(y 4y + 4) = 388 + 4(1) 49(4)
2 2
(x 1) (y 2)
= 1:
49 4

2 2
Since x and y have equal coe cients, the graph is a circle, a point, or
the empty set. Completing the squares, we get
2 2
x + 5x + y y+7=0

63
All rights reserved. No part of this material may be reproduced or transmitted in any form or by any means -
electronic or mechanical including photocopying – without written permission from the DepEd Central Office.
First Edition, 2016.
2 2
x + 5x + 25 + y y+ 1=7+ 25 + 1
4 4 4 4
5 2 1 2 1

x+ 2 +y 2 = 2 :

0 By inspection, the graph is a parabola.


2 2
1 Since the coe cients of x and y are not equal but have the same sign,
the graph is an ellipse, a point, or the empty set. Completing the
squares, we get
2 2
49x + 196x + 100y + 1400y + 196 = 0
2 2
49(x + 4x) + 100(y + 14y) = 196
2 2
49(x + 4x + 4) + 100(y + 14y + 49) = 196 + 49(4) + 100(49)
2 2
(x + 2) + (y + 7) = 1:
100 49
Thus, the graph is an ellipse.
2 2
23 Since the coe cients of x and y are not equal but have the same
sign, the graph is an ellipse, a point, or the empty set. Completing the
squares, we get
2 2
36x + 360x + 64y 512y + 1924 = 0
2 2
36(x + 10x) + 64(y 8y) = 1924
2 2
36(x + 10x + 25) + 64(y 8y + 16) = 1924 + 36(25) + 64(16)
2 2
(x + 5) + (y 4) = 0:
64 36
Since the right-hand side is 0, the graph is a single point (the point is
(5; 4)).
2 2
0 Since x and y have equal coe cients, the graph is a circle, a point,
or the empty set. Completing the squares, we get
2 2
x +y 18y 19 = 0
2 2
x +y 18y + 81 = 19 + 81
2 2
x + (y 9) = 100:

Thus, the graph is a circle.


2 2
0 Since the coe cients of x and y have opposite signs, the graph is a
hyperbola or a pair of intersecting lines. Completing the squares, we get
2 2
5x + 60x + 7y + 84y + 72 = 0

64
All rights reserved. No part of this material may be reproduced or transmitted in any form or by any means -
electronic or mechanical including photocopying – without written permission from the DepEd Central Office.
First Edition, 2016.
2 2
5(x 12x) + 7(y + 12y) = 72
2 2
5(x 12x + 36) + 7(y + 12y + 36) = 72 5(36) + 7(36)
2 2
(x 6) (y + 6)
= 0:
7 5
Since the right hand side is 0, the graph is a pair of intersecting lines;
r
these are y + 6 = 7(x 6).

(h) By inspection, the graph is a parabola.


2
2. The center of a circle is the vertex of the parabola y + 24x 12y + 132 = 0.
If the circle intersects the parabola’s directrix at a point where y = 11, nd
the equation of the circle.
Solution:

y2 12y = 24x 132


2
y 12y + 36 = 24x 132 + 36
2
(y 6) = 24x 96
2
(y 6) = 24(x + 4)

The vertex of the parabola is (4; 6) and its directrix is x = 2. Thus, the
circle has center (4; 6) and contains the point (2; 11). Then its radius is
2 p
(4 2) + (6 11)2 = 61. Therefore, the equation of the circle is (x +
2 2
4) + (y 6) = 61.
2 2
3. The vertices of the hyperbola with equation 9x 72x 16y 128y 256 = 0
are the foci of an ellipse that contains the point (8; 10). Find the standard
equation of the ellipse.
Solution:
2 2
9x 72x 16y
128y 256 = 0
2 2
9(x 8x) 16(y + 8y) = 256
2 2
9(x 8x + 16) 16(y + 8y + 16) = 256 + 9(16) 16(16)
2 2
(x 4) (y + 4)
=1
16 9
The vertices of the hyperbola are (0; 4) and (8; 4). Since these are the foci
of the ellipse, the ellipse is horizontal with center C(4; 4); also, the focal
p

distance of the ellipse is c = 4. The sum of the distances of the point (8;
10) from the foci is
p p
2 2 2 2
(8 0) + (10 (4)) + (8 8) + (10 (4)) = 16:
65
All rights reserved. No part of this material may be reproduced or transmitted in any form or by any means -
electronic or mechanical including photocopying – without written permission from the DepEd Central Office. First Edition, 2016.
This sum is constant for any point on the ellipse; so 2a = 16 and a = 8.
2 2 2
Then b = 8 4 = 48. Therefore, the equation of the ellipse is

(x 4)2 + (y + 4)2 = 1:
64 48

Supplementary Problems 1.5


For items 1 to 8, identify the graph of each of the following equations.

2 2
1. 9x + 72x 64y + 128y + 80 = 0
2 2
2. 49x 490x + 36y + 504y + 1225 = 0
2
3. y + 56x 18y + 417 = 0
2 2
4. x + 20x + y 20y + 200 = 0
2
5. x 10x 48y + 265 = 0
2 2
6. 144x 1152x + 25y 150y 5679 = 0
2 2
7. x + 4x + 16y 128y + 292 = 0
2 2
8. x 6x + y + 14y + 38 = 0

2 2
9. An ellipse has equation 100x 1000x + 36y 144y 956 = 0. Find the
standard equations of all circles whose center is a focus of the ellipse and
which contains at least one of the ellipse’s vertices.
2 2
10. Find all parabolas whose focus is a focus of the hyperbola x 2x 3y 2 = 0
and whose directrix contains the top side of the hyperbola’s auxiliary rectangle.

11. Find the equation of the circle that contains all corners of the auxiliary rect-
2 2
angle of the hyperbola x 18x + y + 10y 81=0.
12. Find the equations of all horizontal parabolas whose focus is the center of
2 2
the ellipse 9x + 17y 170y + 272 = 0 and whose directrix is tangent to the
same ellipse.

13. Find all values of r 6= 1 so that the graph of


2 2
(r 1)x + 14(r 1)x + (r 1)y 6(r 1)y = 60 57r

is

(a) a circle,

66
All rights reserved. No part of this material may be reproduced or transmitted in any form or by any means -
electronic or mechanical including photocopying – without written permission from the DepEd Central Office. First Edition, 2016.
23 a point,
24 the empty set.

14. Find all values of m 6=7; 0 so that the graph of


2 2 2 2
2mx 16mx + my + 7y = 2m 18m

is

(a) a circle.
(b) a horizontal ellipse.
(c) a vertical ellipse.
(d) a hyperbola.
(e) the empty set.

Lesson 1.6. Systems of Nonlinear Equations

Learning Outcomes of the Lesson


At the end of the lesson, the student is able to:
23 illustrate systems of nonlinear equations;
24 determine the solutions of systems of nonlinear equations using
techniques such as substitution, elimination, and graphing; and
25 solve situational problems involving systems of nonlinear equations.

Lesson Outline
23 Review systems of linear equations
24 Solving a system involving one linear and one quadratic equation
25 Solving a system involving two quadratic equations
26 Applications of systems of nonlinear equations

Introduction
After recalling the techniques used in solving systems of linear equations in
Grade 8, we extend these methods to solving a system of equations to systems
in which the equations are not necessarily linear. In this lesson, the equations
are restricted to linear and quadratic types, although it is possible to adapt the

67
All rights reserved. No part of this material may be reproduced or transmitted in any form or by any means -
electronic or mechanical including photocopying – without written permission from the DepEd Central Office. First Edition, 2016.
methodology to systems with other types of equations. We focus on quadratic
equations for two reasons: to include a graphical representation of the solution
and to ensure that either a solution is obtained or it is determined that there is no
solution. The latter is possible because of the quadratic formula.

1.6.1. Review of Techniques in Solving Systems of Linear


Equations

Recall the methods we used to solve systems of linear equations.There were


three methods used: substitution, elimination, and graphical.

Example 1.6.1. Use the substitution method to solve the system, and sketch
the graphs in one Cartesian plane showing the point of intersection.
8
23 4x + y = 6
23 5x + 3y = 4

Solution. Isolate the variable y in the rst equation, and then substitute into the
second equation.

4x + y = 6
=) y = 6 4x

5x + 3y = 4
5x + 3(6 4x) = 4
7x + 18 = 4
x=2
y = 6 4(2) = 2

Example 1.6.2. Use the elimination method to solve the system, and sketch
the graphs in one Cartesian plane showing the point of intersection.
8
0 2x + 7 = 3y
0 4x + 7y = 12

Solution. We eliminate rst the variable x. Rewrite the rst equation wherein
only the constant term is on the right-hand side of the equation, then multiply
it by 2, and then add the resulting equation to the second equation.

68
All rights reserved. No part of this material may be reproduced or transmitted in any form or by any means -
electronic or mechanical including photocopying – without written permission from the DepEd Central Office. First Edition, 2016.
2x
3y = 7
(2)(2x 3y) = (2)(7)
4x + 6y = 14

4x + 6y = 14
4x + 7y = 12
13y = 26
y=2
1
0 =
2

1.6.2. Solving Systems of Equations Using Substitution

We begin our extension with a system involving one linear equation and one
quadratic equation. In this case, it is always possible to use substitution by
solving the linear equation for one of the variables.

Example 1.6.3. Solve the following system, and sketch the graphs in one
Carte-sian plane.
8
0 x y+2=0
2
23 y 1 = x

Solution. We solve for y in terms of x in the rst equation, and substitute this
expression to the second equation.

x y + 2 = 0 =) y = x + 2

2
y 1=x p p p
2
(x + 2) 1 = x 1+ 5 1+ 5 5+ 5
2
x x 1=0 x= 2p =) y = 2p +2= 2p
p 1 5 1 5 5 5
1 5 x= +2=
x= 2 =) y = 2 2
2
p p ! and !
1+ 5 5+ 5 1 p5 5 p5
Solutions: 2 ; 2 2 ; 2

69
All rights reserved. No part of this material may be reproduced or transmitted in any form or by any means -
electronic or mechanical including photocopying – without written permission from the DepEd Central Office. First Edition, 2016.
The rst equation represents a line with x-intercept 2 and y-intercept 2,
while the second equation represents a parabola with vertex at (0; 1) and
which opens upward.

1.6.3. Solving Systems of Equations Using Elimination

Elimination method is also useful in systems of nonlinear equations. Sometimes,


some systems need both techniques (substitution and elimination) to solve them.

Example 1.6.4. Solve the following system:


8 y2 4x 6y = 11
<

2
: 4(3 x) = (y 3) :
Solution 1. We expand the second equation, and eliminate the variable x by
adding the equations.
2 2 2
4(3 x) = (y 3) =) 12 4x = y 6y + 9 =) y + 4x 6y = 3
8 y2 4x 6y = 11
2

get
< y + 4x 6y = 3

Adding these equations, we :


2 2
2y 12y = 14 =) y 6y
7 = 0 =) (y 7)(y+1) = 0 =) y = 7 or y = 1:
Solving for x in the second equation, we have
2
(y 3)
x=3 :
4

70
All rights reserved. No part of this material may be reproduced or transmitted in any form or by any means -
electronic or mechanical including photocopying – without written permission from the DepEd Central Office. First Edition, 2016.
y = 7 =) x = 1 and y=1=)x=1
Solutions: (1; 7) and (1; 1) 2

The graphs of the equations in the preceding example with the points of
intersection are shown below.

Usually, the general form is more convenient to use in solving systems of


equations. However, sometimes the solution can be simpli ed by writing the
equations in standard form. Moreover, the standard form is best for graphing.
Let us again solve the previous example in a di erent way.

Solution 2. By completing the square, we can change the rst equation into
stan-dard form:
y2 4x 6y = 11 =) 4(x + 5) = (y 3)2:
2
8 4(x + 5) = (y 3)
<
2
23 4(3 x) = (y 3)
Using substitution or the transitive property of equality, we get

4(x + 5) = 4(3 x) =) x = 1:
Substituting this value of x into the second equation, we have

2 2
4[3 (1)] = (y 3) =) 16 = (y 3) =) y = 7 or y = 1:
The solutions are (1; 7) and (1; 1), same as Solution 1. 2

71
All rights reserved. No part of this material may be reproduced or transmitted in any form or by any means -
electronic or mechanical including photocopying – without written permission from the DepEd Central Office. First Edition, 2016.
Example 1.6.5. Solve the system and graph the curves:
2 2
8 (x 3) + (y 5) = 10
<
2 2
: x + (y + 1) = 25:
Solution. Expanding both equations, we obtain
8
x2 + y2 6x 10y + 24 = 0
2 2

< x + y + 2y 24 = 0:
:
Subtracting these two equations, we get

6x 12y + 48 = 0 =) x + 2y 8=0
x=8 2y:

We can substitute x = 8 2y to either the rst equation or the second equation.


For convenience, we choose the second equation.
2 2
x + y + 2y 24 = 0
2 2
(8 2y) + y + 2y 24 = 0
2
y 6y + 8 = 0
y = 2 or y = 4
y = 2 =) x = 8 2(2) = 4 and y = 4 =) x = 8 2(4) = 0 The solutions are
(4; 2) and (0; 4).

0 The graphs of both equations are circles. One has center (3; 5) and radius 10, while
the other has center (0; 1) and radius 5. The graphs with the points
of intersection are show below.

72
All rights reserved. No part of this material may be reproduced or transmitted in any form or by any means -
electronic or mechanical including photocopying – without written permission from the DepEd Central Office. First Edition, 2016.
1.6.4. Applications of Systems of Nonlinear Equations

Let us apply systems of equations to a problem involving modern-day


television sets.
?
Example 1.6.6. The screen size of television sets is given in inches. This
indicates the length of the diagonal. Screens of the same size can come in di
erent shapes. Wide-screen TV’s usually have screens with aspect ratio 16 :
9, indicating the ratio of the width to the height. Older TV models often have
aspect ratio 4 : 3. A 40-inch LED TV has screen aspect ratio 16 : 9. Find the
length and the width of the screen.

Solution. Let w represent the width and h the height of the screen. Then, by
Pythagorean Theorem, we have the system
8 w +16 = 40 =) 9w + = 1600
2 2 2 2
w h 2 w h
< = = h=

:
h 9 ) 16 9w 2

2 2
w + h = 1600 =) w 2 + 16
2
= 1600
337w
256 = 1600
w= r 34:86
337

409 600

19x 19(34:86)
h = 16 16 = 19:61
Therefore, a 40-inch TV with aspect ratio 16 : 9 is about 35:86 inches wide and
19:61 inches high. 2

More Solved Examples


Solve the system and graph the curves.
2 2
1. 8 x y = 21
<
0 x+y = 7
Solution: We can write y in terms of x using the second equation as y = 7 x.

73
All rights reserved. No part of this material may be reproduced or transmitted in any form or by any means -
electronic or mechanical including photocopying – without written permission from the DepEd Central Office. First Edition, 2016.
Substituting this into the rst equation, we have
2 2
x (7 x) = 21
14x 70= 0
x= 5:

Thus, the point of intersection is (5; 2).

2 2
8x + y x + 6y + 5 = 0
<
2.
0 x+y+1 = 0
Solution: We can write y in terms of x using the second equation as y = (x
+ 1).
Substituting this into the rst equation, we have
2 2
x + ((x + 1)) x + 6((x + 1)) + 5 = 0
2
2x 5x = 0
x(2x 5) = 0;

5
which yields x = 0 and x = 2. Thus, the points of intersection are (0; 1)

5 7

74
All rights reserved. No part of this material may be reproduced or transmitted in any form or by any means -
electronic or mechanical including photocopying – without written permission from the DepEd Central Office. First Edition, 2016.
8
2
< (y 2) = 4(x 4)
3.
2
(y 4) = x 5
Solution: We can rewrite the rst equation as
2
x 4= (y 2) ;
4
which can be substituted into the second equation by rewriting it as
(y 2)2
(y 4)2 = (x 4) 1 = 1
4
which upon expansion yields
2
3y 28y + 64 = 0:

This equation has roots y = 16=3 and y = 4, giving us the points (5; 4) and
61 16
9; 3 .

75
All rights reserved. No part of this material may be reproduced or transmitted in any form or by any means -
electronic or mechanical including photocopying – without written permission from the DepEd Central Office. First Edition, 2016.
1
2
4. 8 x = 2 (y + 5) 2
< y2 + 10y + (x 2)2 = 9

:
Solution: We can rewrite the rst equation as
2
(y + 5) = 2x + 4;
which can be substituted into the second equation by completing the square
to get
2 2
(y + 10y + 25) + (x 2) =9+25
2 2
(y + 5) + (x 2) = 16
2
(2x + 4) + (x 2) = 16
2
x 2x 8 = 0;
This equation has roots x = 2 and x = 4, giving us the points (2; 5),

(4; 5 p 12), and 4; 5 + p 12 .

Find the system of equations that represents the given problem and solve.

0 The di erence of two numbers is 12, and the sum of their squares is 144.
Find the numbers.
Solution: If x and y are the two numbers, then we have the resulting system
(
x y = 12
2 2
x +y = 144;
where the rst equation yields x = y + 12. Combining this with the second
2 2
equation yields (y + 12) + y = 144 or equivalently 2y(y + 12) = 0, giving
us the ordered pairs (12; 0) and (0; 12).

76
All rights reserved. No part of this material may be reproduced or transmitted in any form or by any means -
electronic or mechanical including photocopying – without written permission from the DepEd Central Office. First Edition, 2016.
Supplementary Problems 1.6
1. Solve the system and graph the curves:

2
(a) 8 x + 3x y + 2 = 0
< y 5x = 1
2
: (y 2) = 9(x + 2)
(b) 8 2 2
< 9x + 4y + 18x 16y = 0
2 2
: (x + 1) + 2(y 4) = 12
(c) 8 2
< y 8y = 4x 16
2 2
: x 2x 4y + 8y 2 = 0
(d) 8 2 2
< 5x 10x + 12y + 24y 58 = 0
:
8
< 2 2
(e) x +y = 2
0 x y = 4

1 Ram is speeding along a highway when he sees a police motorbike parked


on the side of the road right next to him. He immediately starts slowing down,
but the police motorbike accelerates to catch up with him. It is assumed that
the two vehicles are going in the same direction in parallel paths.
The distance that Ram has traveled in meters t seconds after he starts to
2
slow down is given by d (t) = 150 + 75t 1:2t . The distance that the police
2
motorbike travels can be modeled by the equation d (t) = 4t . How long
will it take for the police motorbike to catch up to Ram?
0 The square of a certain number exceeds twice the square of another
1 5
number by 8. Also, the sum of their squares is 16 . Find possible pairs
of numbers that satisfy these conditions.
1 Solve the system of equations
8
2 2
0 x + y = 41
0 xy = 20

2 2
5. Determine the value(s) of k such that the circle x + (y 6) = 36 and the
2
parabola x = 4ky will intersect only at the origin.

77
All rights reserved. No part of this material may be reproduced or transmitted in any form or by any means -
electronic or mechanical including photocopying – without written permission from the DepEd Central Office. First Edition, 2016.
Topic Test 1 for Unit 1
1. Identify the graph of each of the following equations.
2 2 3 2 2
(a) x x + y + 3y 2 =0 (c) 3x 42x 4y 24y + 99 = 0
2 2 2
(b) x + 4x 14y = 52 (d) 7x 112x + 2y + 448 = 0

RSTUVWXYZ0[Determine and sketch the conic with the given equation.


Identify the impor-tant parts of the conic and include them in the graph.
2 2
(a) 25x + 7y 175 = 0
2 2
(b) 64x + 128x + 36y + 288y 1792 = 0

3. Find the equation of the conic with the given properties.

(a) parabola; vertex at (1; 3); directrix x = 7


12 1 12 49
(b) hyperbola; asymptotes y = 5
x 5
and y = 5
x 5
; one vertex at
(3; 5)

4. Solve the following system of equations:


8
2 2
(x 1) + (y + 1) = 5
2
: y = 2(x 1) 8

A doorway is in the shape of a rectangle capped by a semi-ellipse. If the


rectangle is 1 m wide and 2 m high while the ellipse is 0.3 m high at the
center, can a cabinet that is 2.26 m high, 0.5 m wide, and 2 m long be
pushed through the doorway? Assume that the cabinet cannot be laid
down on its side.

A point moves so that its distance from the point (0; 1) is twice its distance
from the line x = 3. Derive the equation (in standard form) of the curve that
is traced by the point, and identify the curve.

78
All rights reserved. No part of this material may be reproduced or transmitted in any form or by any means -
electronic or mechanical including photocopying – without written permission from the DepEd Central Office. First Edition, 2016.
Topic Test 2 for Unit 1
1. Identify the graph of each of the following equations.
2
(a) y + 8x 10y = 15
2 2
(b) x + 10x + y + 18y + 110 = 0
2 2
(c) 9x + 36x + 4y 8y + 4 = 0
2 2
(d) 11x + 132x + 17y 136y 124 = 0

0 Determine and sketch the conic with the given equation. Identify the
impor-tant parts of the conic and include them in the graph.
2 2 2 2
(a) x y = 64 (b) 4x + 24x + 49y 196y + 36

3. Find the equation of the conic with the given properties.

(a) parabola; directrix y = 2; focus at (7 ; 12)


0 ellipse; vertical or horizontal major axis; one vertex at (5; 12); one
covertex at (1; 3)

0 Solve the following system of equations:


89 4 + 54 + 2
2 2
x y x 45 = 0
< (x + 3) = 4y + 4

:
0 Nikko goes to his garden to water his plants. He holds the water hose 3
feet above the ground, with the hoses opening as the vertex and the water
ow following a parabolic path. The water strikes the ground a horizontal
distance of 2 feet from where the opening is located. If he were to stand
on a 1.5 feet stool, how much further would the water strike the ground?

1 A point moves so that its distance from the point (2; 0) is two-thirds its dis-
tance from the line y = 5. Derive the equation (in standard form) of the
curve that is traced by the point, and identify the curve.

79
All rights reserved. No part of this material may be reproduced or transmitted in any form or by any means -
electronic or mechanical including photocopying – without written permission from the DepEd Central Office. First Edition, 2016.
Unit 2

Mathematical Induction

Batad Rice Terraces in Ifugao, by Ericmontalban, 30 September 2012,

https://commons.wikimedia.org/wiki/File%3ABatad rice terraces in Ifugao.jpg. Public Domain.

Listed as one of the United Nations Educational, Scienti c and Cultural


Organization (UNESCO) World Heritage sites since 1995, the two-millennium-old
Rice Terraces of the Philippine Cordilleras by the Ifugaos is a living testimony of
mankind’s creative engineering to adapt to physically-challenging environment in
nature. One of the ve clusters of terraces inscribed in the UNESCO list is the
majestic Batad terrace cluster (shown above), which is characterized by its
amphitheater-like, semicircular terraces with a village at its base.

81
All rights reserved. No part of this material may be reproduced or transmitted in any form or by any means -
electronic or mechanical including photocopying – without written permission from the DepEd Central Office. First Edition, 2016.
Lesson 2.1. Review of Sequences and Series

Learning Outcomes of the Lesson


At the end of the lesson, the student is able to:
0 illustrate a series; and
1 di erentiate a series from a sequence.

Lesson Outline
0 Sequences and series
1 Di erent types of sequences and series (Fibonacci sequence, arithmetic
and geometric sequence and series, and harmonic series)
2 Di erence between sequence and series

Introduction
In this lesson, we will review the de nitions and di erent types of
sequences and series.

Lesson Proper
Recall the following de nitions:

A sequence is a function whose domain is the set of positive


integers or the set f1; 2; 3; : : : ; ng.

A series represents the sum of the terms of a sequence.


If a sequence is nite, we will refer to the sum of the terms of the
sequence as the series associated with the sequence. If the sequence
has in nitely many terms, the sum is de ned more precisely in calculus.

A sequence is a list of numbers (separated by commas), while a series is a


sum of numbers (separated by \+" or \" sign). As an illustration, 1 ; 1 ;1 ; 1
2 3 4
is a sequence, and 1 1 + 1 1 = 7 is its associated series.
2 3 4 12
The sequence with nth term a n is usually denoted by fa ng, and the
associated series is given by
0 = a1 + a2 + a3 + + an:

81
All rights reserved. No part of this material may be reproduced or transmitted in any form or by any means -
electronic or mechanical including photocopying – without written permission from the DepEd Central Office. First Edition, 2016.
Example 2.1.1. Determine the rst ve terms of each de ned sequence, and
give their associated series.
n
(1) f2 ng (3) f(1) g
2
(2) f1 + 2n + 3n g (4) f1 + 2 + 3 + + ng
Solution. We denote the nth term of a sequence by a n, and S = a1 + a2 + a3 +
a4 + a5.
(1) an = 2 n
First ve terms: a1 = 2 1 = 1, a2 = 2 2 = 0, a3 = 1, a4 = 2, a5 = 3
Associated series: S = a1 + a2 + a3 + a4 + a5 = 1 + 0 1 2 3=5
2
0 an = 1 + 2n + 3n
2
First ve terms: a1 = 1 + 2 1 + 3 1 = 6, a2 = 17, a3 = 34, a4 = 57, a5 = 86
Associated series: S = 6 + 17 + 34 + 57 + 86 = 200
n
1 an = (1)
1 2
First ve terms: a1 = (1) =1, a 2 = (1) = 1, a3 = 1, a4 = 1,
a5 = 1
Associated series: S = 1 + 1 1+1 1=1
(4) an = 1 + 2 + 3 + +n
First ve terms: a1 = 1, a2 = 1+2 = 3, a3 = 1+2+3 = 6, a4 = 1+2+3+4 = 10, a5 =
1 + 2 + 3 + 4 + 5 = 15
Associated series: S = 1 + 3 + 6 + 10 + 15 = 35 2

The sequence fang de ned by an = an1 + an2 for n 3, where a1 =


a2 = 1, is called a Fibonacci sequence. It terms are 1; 1; 2; 3; 5; 8; 13; : : :.

An arithmetic sequence is a sequence in which each term after the


rst is obtained by adding a constant (called the common di erence)
to the preceding term.

If the nth term of an arithmetic sequence is a n and the common di erence


is d, then
an = a1 + (n 1)d:
The associated arithmetic series with n terms is given by

Sn = n(a1 + an) = n[2a1 + (n 1)d] :


2 2

82
All rights reserved. No part of this material may be reproduced or transmitted in any form or by any means -
electronic or mechanical including photocopying – without written permission from the DepEd Central Office. First Edition, 2016.
A geometric sequence is a sequence in which each term after the
rst is obtained by multiplying the preceding term by a constant
(called the common ratio).

If the nth term of a geometric sequence is a n and the common ratio is r, then
n1
an = a1r :
The associated geometric series with n terms is given by
n
Sn = 8 a1(1 r )
> na1 if r = 1
<
if r = 1.

> 6
0 (1 r)
The proof of this sum formula is an example in Lesson 2.3.
When 1 < r < 1, the in nite geometric series

2 n1
a1 + a1r + a1r + + a1r +
has a sum, and is given by a
1
S= :
1 r

If fan 1g is an arithmetic sequence, then the sequence with nth term


bn = an is a harmonic sequence.

More Solved Examples


1. How many terms are there in an arithmetic sequence with rst term 5,
common di erence 3, and last term 76?
Solution: a1 = 5, d = 3, an = 76. Find n.
an = 5 + (n 1)(3) = 76
76 5
n 1= 3 = 27; ) n = 28
0 List the rst three terms of the arithmetic sequence if the 25th term is 35
and the 30th term is 5.
Solution: a24 = a1 + 24d = 35 and a30 = a1 + 29d =
5 Eliminating a1 by subtraction, 5d = 30, or d = 6
This implies that a1 = 179, and the rst three terms are 179, 173, 167.

83
All rights reserved. No part of this material may be reproduced or transmitted in any form or by any means -
electronic or mechanical including photocopying – without written permission from the DepEd Central Office. First Edition, 2016.
0 Find the sum of all positive three-digit odd integers.
Solution: Find sn if a1 = 101 = 1 + 50(2), an = 999 = 1 + 499(2).
There are 450 terms from a1 to an, hence n =
450. sn = 450(101 + 999) = 247 500
2
1 The seventh term of a geometric sequence is 6 and the tenth term is
162. Find the fth term.
6 9
Solution: a7 = a1r = 6 and a10 = a1r = 162.
a 3
Eliminating a1 by division: 10 = r = 162 = 27. Thus r = 3
a
7 6
2
Since a5r = a7, a5 = 6 = 2.
9 3
0 Insert three numbers (called geometric means) between 6 and 32=27, so
that the ve numbers form a geometric sequence.
Solution: If a1 = 6 and there are three terms between a 1 and 32=27, then
a5 = 32=27.
32 16 2
4 4
a5 = 6(r) = 27 ) r = 81 ) r = 3
One possible set of three numbers is 4; 8=3; 16=9, the other is 4; 8=3; 16=9.

0 A ball dropped from the top of a building 180 m high always rebounds
three-fourths the distance it has fallen. How far (up and down) will the ball
have traveled when it hits the ground for the 6th time?

h i
s6 =
180 1 3
3
4
6
1
4

0 The Cantor set is formed as follows. Divide a segment of one unit into
three equal parts. Remove the middle one-third of the segment. From
each of the two remaining segments, remove the middle third. From each
of the remaining segments, remove the middle third. This process is
continued inde nitely. Find the total length of the segments removed.
Solution: Let an represent the total length removed in the nth iteration.
Hence a1 = 1=3, a2 = 2=9, a3 = 4=27, and so on.
2 1
3
This means r = . The sum to in nity is s = 2 = 1.
3 1
3
The total length of the segments removed is 1 unit.

84
All rights reserved. No part of this material may be reproduced or transmitted in any form or by any means -
electronic or mechanical including photocopying – without written permission from the DepEd Central Office. First Edition, 2016.
0 The 7th term of an arithmetic sequence is 25. Its rst, third, and 21st term
form a geometric sequence. Find the rst term and the common di erence
of the sequence.
Solution: a7 = a1 + 6d = 25 ) a1 = 25 6d
a
a3 2
= , or a1a21 = a3 .
21

a1 a3
2
(25 6d) (25 6d + 20d) = (25 6d + 2d)
d = 0 and an = 25 for all n, or d = 4 and a1 = 1.

0 Let fang be an arithmetic sequence and fbng an arithmetic sequence of


th
positive integers. Prove that the sequence with n term abn is arithmetic.
Solution: Let the common di erence of fa ng be d and of fbng be c.
abn+1 abn = [a1 + (bn+1 1) d] [a1 + (bn 1) d] = bn+1 bn = c
This proves that the di erence between any two consecutive terms of fa bn
g is a constant independent of n.
3
0 Let fang be a geometric sequence. Prove that fa ng is a geometric
sequence. Solution: Let r be the common ratio of fa n g.
3 n1 3 3 3 n1
a n = (a1r ) =a 1 (r ) .
3 3 3
Thus fa ng is a geometric sequence with rst term a 1 and common ratio r .

1 If fang is a sequence such that its rst three terms form both an arithmetic
and a geometric sequence, what can be concluded about fa ng?
Solution: There is a real number r such that a 2 = a1r and a3 = a1r2.
Since a1 , a2 and a3 form an arithmetic sequence, then a2 a1 = a3 a2, or
a3 2a2 + a1 = 0.
2 1 2
a3 2a2 + a1 = a1r 2a1r + a1 = a (r 1) = 0 ) a1 = 0 or r = 1.
If a1 = 0, then a1 = a2 = a3 = 0. If r = 1, then a1 = a2 = a3.
In all cases, a1 = a2 = a3.

Supplementary Problems 2.1


0 Find the 5th term of the arithmetic sequence whose 3rd term is 35 and
whose 10th term is 77.
2
1 Suppose that the fourth term of a geometric sequence is 9 and the sixth
8
term is 81 . Find the rst term and the common ratio.

85
All rights reserved. No part of this material may be reproduced or transmitted in any form or by any means -
electronic or mechanical including photocopying – without written permission from the DepEd Central Office. First Edition, 2016.
0 The partial sum in the arithmetic series with rst term 17 and a common di
erence 3 is 30705. How many terms are in the series?

1 An arithmetic sequence a1; a2; : : : ; a100 has a sum of 15,000. Find the rst
term and the common di erence if the sum of the terms in the sequence
a3; a6; a9; : : : ; a99 is 5016.

2 The sum of an in nite geometric series is 108, while the sum of the rst 3
terms is 112. Determine the rst term of this series.
32 20
6. Evaluate the in nite series +3 3 21
+ +
3k+1 2k1
+ .
51 52 5k

0 Let n = 0:123 = 0:123123 : : : be a nonterminating repeating decimal. Find


a rational number that is equal to n by expressing n as an in nite
geometric series. Simplify your answer.

1 An arithmetic sequence whose rst term is 2 has the property that its sec-ond,
third, and seventh terms are consecutive terms of a geometric sequence.
Determine all possible second terms of the arithmetic sequence.

2 Eighty loaves of bread are to be divided among 4 people so that the


amounts they receive form an arithmetic progression. The rst two together
receive one-third of what the last two receive. How many loaves does
each person receive?

3 Given a and b, suppose that three numbers are inserted between them so
that the ve numbers form a geometric sequence. If the product of the
three inserted numbers between a and b is 27, show that ab = 9.
2
11. For what values of n will the in nite series (2n 1) + (2n 1) + : : : +
i
(2n 1) + : : : have a nite value?

Lesson 2.2. Sigma Notation

Learning Outcomes of the Lesson


At the end of the lesson, the student is able to use the sigma notation to
represent a series.

Lesson Outline
0 De nition of and writing in sigma notation

86
All rights reserved. No part of this material may be reproduced or transmitted in any form or by any means -
electronic or mechanical including photocopying – without written permission from the DepEd Central Office. First Edition, 2016.
0 Evaluate sums written in sigma notation
1 Properties of sigma notation
2 Calculating sums using the properties of sigma notation

Introduction
The sigma notation is a shorthand for writing sums. In this lesson, we will
see the power of this notation in computing sums of numbers as well as
algebraic expressions.

2.2.1. Writing and Evaluating Sums in Sigma Notation

Mathematicians use the sigma notation to denote a sum. The uppercase


Greek letter (sigma) is used to indicate a \sum." The notation consists of
several components or parts.

Let f(i) be an expression involving an integer i. The expression

f(m) + f(m + 1) + f (m + 2) + + f(n)

can be compactly written in sigma notation, and we write it as


n
X
f (i);
i=m

which is read \the summation of f(i) from i = m to n." Here, m and n


are integers with m n, f(i) is a term (or summand) of the
summation, and the letter i is the index, m the lower bound, and n
the upper bound.

Example 2.2.1. Expand each summation, and simplify if possible.


4
i=2
nX
i
X

(1) (2i + 3) (3) ai


=1

5 6 pn
Xi X
i
(2) 2 (4) n+1
=0 n=1

Solution. We apply the de nition of sigma notation.

4
X
0 (2i + 3) = [2(2) + 3] + [2(3) + 3] + [2(4) + 3] = 27
i=2

87
All rights reserved. No part of this material may be reproduced or transmitted in any form or by any means -
electronic or mechanical including photocopying – without written permission from the DepEd Central Office. First Edition, 2016.
X
5
i 0 1 2 3 4 5
0 2 = 2 + 2 + 2 + 2 + 2 + 2 = 63
i=0
n
X

0 ai = a1 + a2 + a3 + + an
i=1
p p p p
(4) 6 p n =1 + 2 + 3 +2+ 5 + 6 2
X

n=1
n+1 2 3 4 5 6 7
Example 2.2.2. Write each expression in sigma notation.
1 1 1 1
(1) 1 + 2 + 3 + 4 + + 100

(2) 1 + 2 3+4 5+6 7+8 9 + 25


0 a2 + a4 + a6 + a8 + + a20
1 1 1 1 1 1 1
1 1+ 2+ 4+ 8+16 +32 +64 +128
1 1 1 1 100 1
X
Solution. (1) 1 + 2 + 3 + 4 + + 100 = n=1 n

(2) 1+2 3+4 5+ 25


1 2 3 4
= (1) 1+(1) 2+(1) 3+(1) 4
5
+ (1) 5+ +(1) 25 25
X
25
j
0 (1) j
j=1

0 a2 + a4 + a6 + a8 + + a20
0 10
a
2(1)
+a
2(2)
+a
2(3)
+a
2(4)
++ a
2(10)

X
0 a2i
i=1

1 1 1 1 1 1 1 7 1
+ + + + = Xk
(4) 1+ 2 + 4 + 8 16 32 64 128 =0 2k 2

The sigma notation of a sum expression is not necessarily unique. For ex-
ample, the last item in the preceding example can also be expressed in
sigma notation as follows:
8
1 1 1 1 1 1 1 1
+ + + + = Xk
1+2 +4 +8 16 32 64 128 =1 2k1 :
However, this last sigma notation is equivalent to the one given in the example.

88
All rights reserved. No part of this material may be reproduced or transmitted in any form or by any means -
electronic or mechanical including photocopying – without written permission from the DepEd Central Office. First Edition, 2016.
2.2.2. Properties of Sigma Notation

We start with nding a formula for the sum of


n
X

i=1+2+3+ +n
i=1

in terms of n.
The sum can be evaluated in di erent ways. One informal but simple
approach is pictorial.

n
X n(n + 1)

i=1+2+3+ +n=
i=1

Another way is to use the formula for an arithmetic series with a 1 = 1 and
an = n:
0= n(a 1
+ a ) n(n + 1)
n = :
22
We now derive some useful summation facts. They are based on the
axioms of arithmetic addition and multiplication.
n n
X Xi
cf(i) = c f(i), c any real number.
i=m =m

Proof.
n
X

cf(i) = cf(m) + cf(m + 1) + cf(m + 2) + + cf(n)


i=m

89
All rights reserved. No part of this material may be reproduced or transmitted in any form or by any means -
electronic or mechanical including photocopying – without written permission from the DepEd Central Office. First Edition, 2016.
= c[f(m) + f(m + 1) + + f(n)]
n
X
=c f(i) 2
i=m

n n n
X X X
[f(i) + g(i)] = f(i) + g(i)
i=m i=m i=m

Proof.
n
X
[f(i) + g(i)]
i=m
0 [f(m) + g(m)] + + [f(n) + g(n)]
1 [f(m) + + f(n)] + [g(m) + + g(n)]
n n
X
i X
= f(i) + g (i) 2
=m i=m

n
Xi

c = c(n m + 1)
=m

Proof.
n
c=c+c+c+ +c
=| ( {z }
=m
Xi nm+1 terms
cn m + 1) 2

A special case of the above result which you might encounter more often
is the following:
n
X
c = cn:
i=1

Telescoping Sum
n
X
[f(i + 1) f(i)] = f(n + 1) f(m)
i=m

90
All rights reserved. No part of this material may be reproduced or transmitted in any form or by any means -
electronic or mechanical including photocopying – without written permission from the DepEd Central Office. First Edition, 2016.
Proof.
n
X
f(i + 1) f(i)
i=m
= [f(m + 1) f(m)] + [f(m + 2) f(m + 1)]
+ [f(m + 3) f(m + 2)] + + [f(n + 1) f(n)]

Note that the terms, f(m + 1); f(m + 2); : : : ; f(n), all cancel out. Hence, we have
n
X
i [f(i + 1) f(i)] = f(n + 1) f(m): 2
=m

30
X
i

Example 2.2.3. Evaluate: (4i 5).


=1

Solution.
30 30 30
Xi X X
5
(4i 5) = 4i
=1 i=1 i=1
30 30
X
X
5
i

=4 i
=1 i=1

= 4 (30)(31) 5(30)
2
= 1710 2

Example 2.2.4. Evaluate:


1 1 1 1
1 2+ 2 3+ 3 4 + + 99 100 :
Solution.
1 1 1 1
1 2 + 2 3 +3 4 + + 99 100
99
Xi 1
= i(i + 1)
=1

99 i+1 i
Xi

= i(i + 1)
=1
=
i=1
99 i(i + 1) i(i + 1)
X
i +1 i

91
All rights reserved. No part of this material may be reproduced or transmitted in any form or by any means -
electronic or mechanical including photocopying – without written permission from the DepEd Central Office. First Edition, 2016.
=
i=1 i
99 i+1
X
1 1

= 99
i=1
i+1 i
X
1 1

Using f(i) = 1 and the telescoping-sum property, we get


i
:
99 i =1
100 1 =
i(i + 1)
=
100 2
X
1 1 1 99

3 n
f(i) = i . Xi
2
Example 2.2.5. Derive a formula for i using a telescoping sum with terms
=1

Solution. The telescoping sum property implies that


nX
i
3 3 3 3 3
i (i 1) =n 0 =n :
=1

On the other hand, using expansion and the other properties of


summation, we have
n n
X 3 Xi
3 2
i
3
(i 1) = (i
3
i + 3i 3i + 1)
n n n
i=1 =1

X X X
= 3 i2 3 i+ 1
i=1 i=1 i=1
n
3 n(n + 1) + n:
X

2
=3 i
i=1
2

Equating the two results above, we obtain

n
2 3n(n + 1) + n = n3
3 i
Xi
=1
2
n
Xi
2 3
6 i 3n(n + 1) + 2n = 2n
=1

n
Xi
2 3
6 i = 2n 2n + 3n(n + 1)
=1

92
All rights reserved. No part of this material may be reproduced or transmitted in any form or by any means -
electronic or mechanical including photocopying – without written permission from the DepEd Central Office.
First Edition, 2016.
2
= 2n(n 1) + 3n(n + 1)
= 2n(n 1)(n + 1) + 3n(n + 1)
= n(n + 1)[2(n 1) + 3]
= n(n + 1)(2n + 1):

Finally, after dividing both sides of the equation by 6, we obtain the desired formula
n
2
X
i = n(n + 1)(2n + 1) : 2
i

6
=1

More Solved Examples


1. Expand the following sums and simplify if possible:
5
X
(a) (i2 i + 1)
i=1

6 i+1 2
X
(b) i2 2
i=3
5
0
X

x3iy153i
i=0
9
x2i+1
X
(d)
(i + 1)2
i=1
1
0
X

3i+2 2i+1
i=1

Solution:
5

(a) Xi (i
2 2 2 2
i + 1) = (1 1+1)+(2 2 + 1) + : : : + (5 5 + 1) = 45 or
=1
5(5 + 1)(2(5) + 1) 5(5 + 1) +5=45
6 2
(b) 6 i+1 2 i
2 =3
2
4 2 2 +:::+6
2
7
2
2 = 802

X
2
i=3

5
X
i9 6 12 3 15

(c) x3iy153i = x3(0)y153(0) + : : : + x3(5)y153(5) = xy15 + x3y12 + x6y9 +


=0
xy +x y +x y

93
All rights reserved. No part of this material may be reproduced or transmitted in any form or by any means -
electronic or mechanical including photocopying – without written permission from the DepEd Central Office. First Edition, 2016.
3 5
9
Xi
x2i+1 x x x19
3 5 7
19

(d) 2 = 2 + 2 +:::+ 2 = 4x + 9x + 16x +


=1 (i + 1) (1+1) (2+1) (9+1)
: : : + 100x
1 1 2 i+1
X X
(e) i=1 3i+2 2i+1 = 27 i=1 3 , which is an in nite geometric series with
2 4 1 4=9
Xi = 36.
jrj = 3 < 1 and a1 = 9, giving us 3i+2 2i+1 = 271 (2=3)
=
1
2
0
X
2. Evaluate i
[2(i 1) + 2].
=
1
2
0 20
Solution: (2(i 1)+2)= 2i = 4 (20)(21) = 840.
X
X i
2
i=1 =1

1 1 1 1
3. Find a formula for 1(3) + 2(4) + 3(5) + + n(n + 2) given any positive
integer n.
n
1 1 1 1 Xi 1
Solution: We have 1(3)+ 2(4)+ 3(5) + + n(n + 2) = i(i + 2). Rewriting
=
1
yields
= (2 1) + (i i) = 1 11 1
i(i + 2) i(i + 2) i i + 2 i(i + 2)
or equivalently i+2=2 i i+2 :
1 1 1 1

Expanding the sum term by term,

n i=1 i(i + 2)
=
1 n 1 1
i=1 2 i i+2
1
X X

= 1 3+24+35
1 1 1 1 1
+ n 2 n + n 1 n+1 + n n+2
1 1 1 1 1 1

1 1
=1+ 1
2 n+1 n+2
n(3n + 2)
0 2(n + 1)(n + 2):
94
All rights reserved. No part of this material may be reproduced or transmitted in any form or by any means -
electronic or mechanical including photocopying – without written permission from the DepEd Central Office.
First Edition, 2016.
N
Xi 1 97
4. Determine the value of N such that i2 + 3i + 2 = 98.
=0
Solution: Rewrite the sum as

N N
1 1
Xi 2 X
=1 i + 3i + 2 = i=1 (i + 1)(i + 2)
=
i=1
N i+1 i+2
X
1 1

1
Set f(i) = i + 1 and use telescoping sums to get
2 =
N
i=1
i + 3i + 2 i=1
N
i+2 i+1

X X
1 1 1

= N+2 1
1 1

N+1
= :
N+2

97
Since we want the sum to be equal to 98, N = 96.

Supplementary Problems 2.2


1. Expand the following sums:
0 p i
X

0 3 2
i=3
5
x2i
X
(b)
2i
i=1

X
5
i i1
0 (1) x
i=2

0 Write the following in sigma notation. 95

All rights reserved. No part of this material may be reproduced or transmitted in any form or by any means -
electronic or mechanical including photocopying – without written permission from the DepEd Central Office.
First Edition, 2016.
2 3 4
(a) (x + 5) (x + 3) + (x + 1) (x 1)
2 2 2
1 2 3 10
(b) 3 + 3 + 3 +:::+ 3
3 4 5 11
(c) a3 + a6 + a9 + : : : + a81

3. Evaluate the following sums


150
X
0 (4i + 2)
i=1
120
X
0 i(i 5)
i=3
50
X
0 (2i 1)(2i + 1)
i=1

50
X
50 Xi
50
X
g(i) + 3f(i)
4. If f(i) = 20 and g(i) = 30, what is the value of p 2 ?
i=1 =1 i=1
200
X
200
i X
2 2
5. If s = (i 1) i , express i in terms of s.
=1 i=1

X X Xi
6. If s = n ai and t = n bi, does it follow that n
a
i =s?
b
i=1 i=1 =1 i t

Lesson 2.3. Principle of Mathematical Induction

Learning Outcomes of the Lesson


At the end of the lesson, the student is able to:
0 illustrate the Principle of Mathematical Induction; and
1 apply mathematical induction in proving identities.

Lesson Outline
State the Principle of Mathematical Induction
Prove summation identities using mathematical induction
Prove divisibility statements using mathematical induction
Prove inequalities using mathematical induction

96
All rights reserved. No part of this material may be reproduced or transmitted in any form or by any means -
electronic or mechanical including photocopying – without written permission from the DepEd Central Office. First Edition, 2016.
Introduction
We have derived and used formulas for the terms of arithmetic and geometric
sequences and series. These formulas and many other theorems involving positive
integers can be proven with the use of a technique called mathematical induction.

2.3.1. Proving Summation Identities

The Principle of Mathematical Induction


Let P (n) be a property or statement about an integer n. Suppose
that the following conditions can be proven:

P (n0) is true (that is, the statement is true when n = n 0).


If P (k) is true for some integer k n 0, then P (k + 1) is true (that is, if
the statement is true for n = k, then it is also true for n = k + 1).

Then the statement P (n) is true for all integers n n0.

The Principle of Mathematical Induction is often compared to climbing an


in nite staircase. First, you need to be able to climb up to the rst step.
Second, if you are on any step (n = k), you must be able to climb up to the
next step (n = k + 1). If you can do these two things, then you will be able to
climb up the in nite staircase.

Part 1 Part 2

Another analogy of the Principle of Mathematical Induction that is used is


toppling an in nite line of standing dominoes. You need to give the rst domino
a push so that it falls down. Also, the dominoes must be arranged so that if
the kth domino falls down, the next domino will also fall down. These two
conditions will ensure that the entire line of dominoes will fall down.

97
All rights reserved. No part of this material may be reproduced or transmitted in any form or by any means -
electronic or mechanical including photocopying – without written permission from the DepEd Central Office. First Edition, 2016.
Standing Domino Tiles, by Nara Cute, 16 October 2015,

https://commons.wikimedia.org/wiki/File:Wallpaper kartu domino.png. Public Domain.

There are many mathematical results that can be proven using


mathematical induction. In this lesson, we will focus on three main
categories: summation identities, divisibility statements, and inequalities.
Let us now take a look at some examples on the use of mathematical
induction in proving summation identities.
Example 2.3.1. Using mathematical induction, prove that
n(n + 1)
1+2+3+ +n=

for all positive integers n.

Solution. We need to establish the two conditions stated in the Principle of


Math-ematical Induction.

Part 1.
Prove that the identity is true for n = 1.
The left-hand side of the equation consists of one term equal to 1. The
right-hand side becomes
1(1+1) = 2 =1:
2 2
Hence, the formula is true for n = 1.

98
All rights reserved. No part of this material may be reproduced or transmitted in any form or by any means -
electronic or mechanical including photocopying – without written permission from the DepEd Central Office. First Edition, 2016.
Part 2. Assume that the formula is true for n = k 1:
k(k + 1)
1+2+3+ +k= :

We want to show that the formula is true for n = k + 1; that is,


(k + 1)(k + 1 + 1)
1+2+3+ + k + (k + 1) = :

Using the formula for n = k and adding k + 1 to both sides of the equation,
we get
k(k + 1)
1+2+3+ + k + (k + 1) = + (k + 1)

= k(k + 1) + 2(k + 1)
2
= (k + 1)(k + 2)
2
= (k + 1) [(k + 1) + 1]
2

We have proven the two conditions required by the Principle of Mathematical


Induction. Therefore, the formula is true for all positive integers n. 2

Example 2.3.2. Use mathematical induction to prove the formula for the sum
of a geometric series with n terms:
n
a1 (1 r )
Sn = ; r 6= :1
r

Solution. Let an be the nth term of a geometric series. From Lesson 2.1, we
n1
know that an = a1r .

Prove that the formula is true for n = 1.


a (1 r1)
1 = a1 = S1
r
The formula is true for n = 1.
k
a1(1 r )
Part 2. Assume that the formula is true for n = k 1: S k = 1 r . We
want to prove that it is also true for n = k + 1; that is,

S
a1(1 rk+1)
k+1 = :
1 r
99
All rights reserved. No part of this material may be reproduced or transmitted in any form or by any means -
electronic or mechanical including photocopying – without written permission from the DepEd Central Office. First Edition, 2016.
We know that

Sk+1 = a1 + a2 + + ak +ak+1
| {z }
Sk

Sk + ak+1
a1 1 rk
+ a1rk 1 r
a +
= 1 1 r 1 r1 (1 )
k
k ar r
= 1 1 1+ r
a rk rk rk+1
= 1 1 r
a 1 rk+1

By the Principle of Mathematical Induction, we have proven that


n
a1(1 r )
Sn =
1 r
for all positive integers n. 2

Example 2.3.3. Using mathematical induction, prove that


n(n + 1)(2n + 1)
12 + 2 2 + 3 2 + + n2 =
6
for all positive integers n.

Solution. We again establish the two conditions stated in the Principle of


Math-ematical Induction.

Part 1
1(1+1)(2 1+1) 123
= =1=12
6 6
The formula is true for n = 1.

Part 2
k(k + 1)(2k + 1)

Assume: 12 + 22 + 32 + + k2 = .

2 2 2 2 2
Prove: 1 + 2 + 3 + + k + (k + 1)
= (k + 1)(k + 2) [2(k + 1) + 1]
6
(k + 1)(k + 2)(2k + 3): 6

100
All rights reserved. No part of this material may be reproduced or transmitted in any form or by any means -
electronic or mechanical including photocopying – without written permission from the DepEd Central Office. First Edition, 2016.
2 2 2 2 2
1 +2 +3 + + k + (k + 1)
k(k + 1)(2k + 1)
= + (k + 1)2
6
2
= k(k + 1)(2k + 1) + 6(k + 1)
6
= (k + 1) [k(2k + 1) + 6(k + 1)]
6
2
= (k + 1) (2k + 7k + 6)
6
(k + 1)(k + 2)(2k + 3)
6

Therefore, by the Principle of Mathematical Induction,


n(n + 1)(2n + 1)
2 2 2 2
1 +2 +3 + +n = 6
for all positive integers n. 2

2.3.2. Proving Divisibility Statements

We now prove some divisibility statements using mathematical induction.


Example 2.3.4. Use mathematical induction to prove that, for every positive
n
integer n, 7 1 is divisible by 6.

Solution. Similar to what we did in the previous session, we establish the two
conditions stated in the Principle of Mathematical Induction.

Part 1
1
7 1=6=6 1
1
7 1 is divisible by 6.

Part 2
k
Assume: 7 1 is divisible by 6.
k+1
To show: 7 1 is divisible by 6.
k+1 k k k k k
7 1=7 7 1=6 7 +7 1 = 6 7 + (7 1)
k
By de nition of divisibility, 6 7 is divisible by 6. Also, by the hypothesis
k
(assumption), 7 1 is divisible by 6. Hence, their sum (which is equal to
k+1
7 1) is also divisible by 6.

101
All rights reserved. No part of this material may be reproduced or transmitted in any form or by any means -
electronic or mechanical including photocopying – without written permission from the DepEd Central Office.
First Edition, 2016.
n
Therefore, by the Principle of Math Induction, 7 1 is divisible by 6 for all
positive integers n. 2
0
divisible by 6. Hence, a stronger
Note that 7 1 = 1 1 = 0 = 6 0 is also n
and more precise result in the preceding example is: 7 1 is divisible by 6 for
every nonnegative integer n. It does not make sense to substitute negative
n
values of n since this will result in non-integer values for 7 1.
Example 2.3.5. Use mathematical induction to prove that, for every nonnega-
3
tive integer n, n n + 3 is divisible by 3.

Solution. We again establish the two conditions in the Principle of


Mathematical Induction.

Part 1 Note that claim of the statement is that it is true for every nonnegative
integer n. This means that Part 1 should prove that the statement is true for n
= 0.
3
0 0+3=3=3(1)
3
0 0+ 3 is divisible by 3.
3
Part 2. We assume that k k + 3 is divisible by 3. By de nition of divisibility,
3
we can write k k + 3 = 3a for some integer a.
3
To show: (k + 1) (k + 1) + 3 is divisible by 3.

3 3 2
(k + 1) (k + 1) + 3 = k + 3k + 2k + 3
3 2
= (k k + 3) + 3k + 3k
2
3a + 3k + 3k
2
3(a + k + k)

2 3
Since a+k +k is also an integer, by de nition of divisibility, (k + 1) (k + 1) + 3
is divisible by 3.
3
Therefore, by the Principle of Math Induction, n n + 3 is divisible by 3 for
all positive integers n. 2

?
2.3.3. Proving Inequalities
Finally, we now apply the Principle of Mathematical Induction in proving
some inequalities involving integers.
n
Example 2.3.6. Use mathematical induction to prove that 2 > 2n for every
integer n 3.

102
All rights reserved. No part of this material may be reproduced or transmitted in any form or by any means -
electronic or mechanical including photocopying – without written permission from the DepEd Central Office. First Edition, 2016.
Solution. Just like the previous example, we establish the two conditions in
the Principle of Mathematical Induction.

Part 1
3
2 =8>6=2(3)
3
This con rms that 2 > 2(3).
Part 2
k
Assume: 2 > 2k, where k is an integer with k 3
k+1
To show: 2 > 2(k + 1) = 2k + 2
We compare the components of the assumption and the inequality we
need to prove. On the left-hand side, the expression is doubled. On the right-
hand side, the expression is increased by 2. We choose which operation we
want to apply to both sides of the assumed inequality.

Alternative 1. We double both sides.


k k
Since 2 > 2k, by the multiplication property of inequality, we have 2 2 >
2 2k.
k+1
2 > 2(2k) = 2k + 2k > 2k + 2 if k 3.
k+1
Hence, 2 > 2(k + 1).
Alternative 2. We increase both sides by 2.
k k
Since 2 > 2k, by the addition property of inequality, we have 2 + 2 > 2k + 2.
k k k
2(k + 1) = 2k + 2 < 2 + 2 < 2 + 2 if k 3.
k k k k+1
The right-most expression above, 2 + 2 , is equal to 2 2 = 2 .
k+1
Hence, 2(k + 1) < 2 .
n
Therefore, by the Principle of Math Induction, 2 > 2n for every integer
n 3. 2
We test the above inequality for integers less than 3.
0
2 = 1> 0 = 2(0) True
21 = 2= 2(1) False
22 = 4= 2(2)
False
The inequality is not always true for nonnegative integers less than 3. This
illustrates the necessity of Part 1 of the proof to establish the result. However,
n
the result above can be modi ed to: 2 2n for all nonnegative integers n.
Before we discuss the next example, we review the factorial notation. Recall

103
All rights reserved. No part of this material may be reproduced or transmitted in any form or by any means -
electronic or mechanical including photocopying – without written permission from the DepEd Central Office. First Edition, 2016.
that 0! = 1 and, for every positive integer n, n! = 1 2 3 n. The factorial also
satis es the property that (n + 1)! = (n + 1) n!.
n
Example 2.3.7. Use mathematical induction to prove that 3 < (n + 2)! for
every positive integer n. Can you re ne or improve the result?

Solution. We proceed with the usual two-part proof.

Part 1
1 1
3 =3<6=3!=(1+2)! =) 3 <(1+2)!
Thus, the desired inequality is true for n = 1.

Part 2
k
Assume: 3 < (k + 2)!
k+1
To show: 3 < (k + 3)!
k
Given that 3 < (k + 2)!, we multiply both sides of the inequality by 3 and
obtain
k
3 3 < 3 [(k + 2)!] :

This implies that


k
3 3 < 3 [(k + 2)!] < (k + 3) [(k + 2)!] ; since k > 0,

and so
k+1
3 < (k + 3)!:
n
Therefore, by the Principle of Math Induction, we conclude that 3 < (n +
2)! for every positive integer n.
The left-hand side of the inequality is de ned for any integer n. The right-
hand side makes sense only if n + 2 0, or n 2.
When n = 2: 3 2 = 1 <1=0!=(2+2)!
9
1
When n = 1: 3 1 = <1=1!=(1+2)!
3
0
When n = 0: 3 = 1 < 2 = 2! = (0 + 2)!
n
Therefore, 3 < (n + 2)! for any integer n2. 2

104
All rights reserved. No part of this material may be reproduced or transmitted in any form or by any means -
electronic or mechanical including photocopying – without written permission from the DepEd Central Office. First Edition, 2016.
More Solved Examples
Use mathematical induction to prove the given statements below.

2 n1 n
1. 2 3 + 2 3 + : : : + 2 3 =3 3 for n 1
Solution:
Part 1.

2
2 3=6=3 3.
The formula is true for n = 1.
Part 2.
2 k1 k
Assume: P = 2 3 + 2 3 + : : : + 2 3 =3 3.
2 k k+1
To show: 2 3 + 2 3 + : : : + 2 3 = 3 3.

2 k k
2 3+2 3 +:::+2 3 =P+2 3
k k
=3 3+2 3
k
=3 3 3

= 3k+1 3:
1
2 n1 n
2. 1 + 4 + 4 + : : : + 4 = 3 (4 1) for n 1
Solution:
Part 1.
1
1
1 = 3 (4 1).
The formula is true for n = 1.
Part 2.
Assume: P =1+4+42+:::+4 1 = 3 4 1 .
k1 1 k

2 k
To show: 1 + 4 + 4 + : : : + 4 = 3 4k+1 1 .
2 k k
1+4+4 +:::+4 =P+4
= 1 4
k k
3
1 +4

4 k 1
= 3 4 3
1

= 3 4k+1 1:
105
All rights reserved. No part of this material may be reproduced or transmitted in any form or by any means -
electronic or mechanical including photocopying – without written permission from the DepEd Central Office.
First Edition, 2016.
2
3. 1 22 1 32 1 (n 1) 1 n2 = 2n for n 2.
1 1 1 1 n+1

Solution:
Part 1.
1 3 2+1
= = .
22 4 2(2)

The formula is true for n = 2.

Part 2. 2 2 2 2
Assume: P = 1 2 1 3 1 (k 1) 1 k = 2k .
1 1 1 1 k+1
2
To show: 1 22 1 k2 1 (k + 1) = 2(k + 1).
1 1 1 k+2

1 22 1 (k + 1)2 = P 1 (k + 1)2
1 1 1

2
k + 1 (k + 2k + 1) 1
2
= 2k (k + 1)
k + 1 k(k + 2)
2
= 2k (k + 1)
= k+2 :
2(k + 1)

n+1 2n1
4. Prove that 4 +5 is divisible by 21 for all integers n 1.
Solution:
Part 1.

41+1 + 52(1)1 = 21.

The number is divisible by 21 for n = 1.

Part 2.
k+1 2k1
Assume: 4 +5 is divisible by 21.
Prove: 4k+2 + 52(k+1)1 is divisible by 21.
k+1 2k1 k+1 2k1 2k1
4k+2 + 52(k+1)1 =4 4 + 25 5 =44 +5 + 21 5
k+1
21 52k1 (assumption), 4 + 52k1 is
is divisible by 21 and by the hypothesis k+2 2(k+1)1
divisible by 21. Hence, their sum which is equal to 4 +5 is divisible
by 21.

106
All rights reserved. No part of this material may be reproduced or transmitted in any form or by any means -
electronic or mechanical including photocopying – without written permission from the DepEd Central Office. First Edition, 2016.
2
n > 2n + 3 for n 4.
Solution:
Part 1.
4
2 = 16 > 7 = 2(2) + 3 The inequality is true for n = 4.
Part 2
2
Assume: k > 2k + 3
2
Prove: (k + 1) > 2(k + 1) + 3
2
We expand (k + 1) and use the inequality in the hypothesis to get

2 2
(k + 1) = k + 2k + 1 > (2k + 3) + 2k + 1 = 4(k + 1) > 2(k + 1) + 3 if k > 0.

2
Therefore, by the principle of math induction, n > 2n + 3 for n 4.

n+3
Prove that 2 < (n + 3)! for n 4.
Solution:
Part 1.
4+3 7
2 = 2 < 1 2 3 7 = (4 + 3)! The inequality is true for n = 1. Part
2
k+3
Assume: 2 < (k + 3)!
k+4
Prove: 2 < (k + 4)!
k+3
Given that 2 < (k + 3)!, we multiply both sides of the inequality by 2 and
obtain
k+3
22 < 2 [(k + 3)!].

This implies that

k+4
2 < 2 [(k + 3)!] < (k + 4) [(k + 3)!], if k > 0.

k+3
Therefore, by the principle of math induction, 2 < (k+3)! for every
positive integer n.

107
All rights reserved. No part of this material may be reproduced or transmitted in any form or by any means -
electronic or mechanical including photocopying – without written permission from the DepEd Central Office. First Edition, 2016.
Supplementary Problems 2.3
Prove the following by mathematical induction:

1 2 3 n n+2
2 3 n n
1. 2 + 2 + 2 + +2 = 2 2 for n 1
n
Xi n(n + 3)
2. (i +1)= 2
=1

1(1!) + 2(2!) + : : : + n(n!) = (n + 1)! 1.


The sum of the rst n odd numbers is equal to n: 2.
5. 1 2 1 3 1 4 ::: 1 n = 2n
1 1 1 1 1

n
X n
i (1) n(n + 1)
6. (1) i =
i2
2
=1

3n+1 3n+1
4 +2 + 1 is divisible by 7
n+2 2n+1
11 + 12 is divisible by 133
2n+1 n+2 n+2 2n+1
5 2 +3 2 is divisible by 19
n
10. 11 6 is divisible by 5
10 n 5 n+2
3 + 3 + 4 is divisible by 3
2 n
n < 2 for n 5.
1 1 1 1 1
3 3 3 3
13. 1 + 2 +3 +:::+ n 2 n for n 1.

p
14. The sequence an = 2an1 ; a1 = p 2 is increasing; that is, an < an+1.
4

Lesson 2.4. The Binomial Theorem

Learning Outcomes of the Lesson


At the end of the lesson, the student is able to:
n
illustrate Pascal’s Triangle in the expansion of (x + y) for small positive
integral values of n;

108
All rights reserved. No part of this material may be reproduced or transmitted in any form or by any means -
electronic or mechanical including photocopying – without written permission from the DepEd Central Office. First Edition, 2016.
prove the Binomial Theorem;
n
determine any term in (x + y) , where n is a positive integer, without ex-
panding; and
solve problems using mathematical induction and the Binomial Theorem.

Lesson Outline
n
Expand (x + y) for small values of n using Pascal’s Triangle
Review the de nition of and formula for combination
State and prove the Binomial Theorem
Compute all or speci ed terms of a binomial expansion
Prove some combination identities using the Binomial Theorem

Introduction
n
In this lesson, we study two ways to expand (a + b) , where n is a positive
integer. The rst, which uses Pascal’s Triangle, is applicable if n is not too big,
and if we want to determine all the terms in the expansion. The second
n
method gives a general formula for the expansion of (a + b) for any positive
integer n. This formula is useful especially when n is large because it avoids
the process of going through all the coe cients for lower values of n obtained
through Pascal’s Triangle.

2.4.1. Pascal’s Triangle and the Concept of Combination

Consider the following powers of a + b:


1
(a + b) = a + b
2 2 2
(a + b) = a + 2ab + b
3 3 2 2 3
(a + b) = a + 3a b + 3ab + b
4 4 3 2 2 3 4
(a + b) = a + 4a b + 6a b + 4ab + b
5 5 4 3 2 2 3 4 5
(a + b) = a + 5a b + 10a b + 10a b + 5ab + b

We now list down the coe cients of each expansion in a triangular array
as follows:
n=1: 1 1
n=2: 1 2 1
n=3: 1 3 3 1
n=4: 1 4 6 4 1
n=5: 1 5 10 10 5 1

109
All rights reserved. No part of this material may be reproduced or transmitted in any form or by any means -
electronic or mechanical including photocopying – without written permission from the DepEd Central Office. First Edition, 2016.
The preceding triangular array of numbers is part of what is called the
Pas-cal’s Triangle, named after the French mathematician, Blaise Pascal
(1623-1662). Some properties of the Triangle are the following:

Each row begins and ends with 1.

Each row has n + 1 numbers.

The second and second to the last number of each row correspond to the
row number.

There is symmetry of the numbers in each row.

The number of entries in a row is one more than the row number (or one
more than the number of entries in the preceding row).

Every middle number after rst row is the sum of the two numbers above it.

It is the last statement which is useful in constructing the succeeding rows of


the triangle.
Example 2.4.1. Use Pascal’s Triangle to expand the expression (2x 3y)5.

Solution. We use the coe cients in the fth row of the Pascal’s Triangle.
5 5 4 3 2
(2x 3y) = (2x) + 5(2x) (3y ) + 10(2x) (3y )
2 3 4
10(2x) (3y ) + 5(2x)(3y )
5
(3y )
5 4 3 2 2 3
= 32x 240x y + 720x y 1080x y
4 5
+ 810xy 243y 2
8
Example 2.4.2. Use Pascal’s Triangle to expand (a + b) .

Solution. We start with the sixth row (or any row of the Pascal’s Triangle that
we remember).

n=6: 1 6 15 20 15 6 1
n=7: 1 7 21 35 35 21 7 1
n=8: 1 8 28 56 70 56 28 8 1
Therefore, we get
8 8 7 6 2 5 3
(a + b) = a + 8a b + 28a b + 56a b
4 4 3 5 2 6
+ 70a b + 56a b + 28a b
7 8
+ 8ab + b 2

110
All rights reserved. No part of this material may be reproduced or transmitted in any form or by any means -
electronic or mechanical including photocopying – without written permission from the DepEd Central Office. First Edition, 2016.
n
We observe that, for each n, the expansion of (a + b) starts with
n
a and the exponent of a in the succeeding terms decreases by 1,
while the exponent of b increases by 1. This observation will be
shown to be true in general.

Let us review the concept of combination. Recall that C(n; k) or n counts


k n k

the number of ways of choosing objects from a set of objects. It is


to know some properties of C(n; k):

(1) C(n; 0) = C(n; n) = 1,

(2) C(n; 1) = C(n; n 1) = n, and

(3) C(n; k) = C(n; n k).

These properties can explain some of the observations we made on the


num-bers in the Pascal’s Triangle. Also recall the general formula for the
number of combinations of n objects taken k at a time:
C(n; k) = k = k !(n ! k)!;
n n

where 0! = 1 and, for every positive integer n, n! = 1 2 3 n.


5 8

Example 2.4.3. Compute 3 and 5 .


Solution.
5 5! 5!
3 = (5 3)!3! =2!3! = 10
8 8! 10!
5 = (8 5)!5! =3!5! = 56 2
5
You may observe that the value of 3 and the fourth coe cient in the fth row of
8
Pascal’s Triangle are the same. In the same manner, 5 is equal to the sixth coe
8
cient in the expansion of (a + b) (see Example 2.4.2). These observed
equalities are not coincidental, and they are, in fact, the essence embodied in
the Binomial Theorem, as you will see in the succeeding sessions.

2.4.2. The Binomial Theorem

As the power n gets larger, the more laborious it would be to use Pascal’s Triangle
n
(and impractical to use long multiplication) to expand (a + b) . For example, using
Pascal’s Triangle, we need to compute row by row up to the thirtieth row

111
All rights reserved. No part of this material may be reproduced or transmitted in any form or by any means -
electronic or mechanical including photocopying – without written permission from the DepEd Central Office. First Edition, 2016.
30
to know the coe cients of (a + b) . It is, therefore, delightful to know that it is
possible to compute the terms of a binomial expansion of degree n without
going through the expansion of all the powers less than n.
We now explain how the concept of combination is used in the expansion
n
of (a + b) .
n
(a + b) = (a + b)(a + b)(a + b) (a + b)
| {z }
n factors
When the distributive law is applied, the expansion of (a + b) n consists of
terms of the form ambi, where 0 m; i n. This term is obtained by choosing a for
m of the factors and b for the rest of the factors. Hence, m + i = n, or m = n i.
This means that the number of times the term a ni bi will appear in the
expansion of (a + b)n equals the number of ways of choosing (n i) or i factors from the n
factors, which is exactly C(n; i). Therefore, we have
i
n
(a + b) = i=0
n i ani b :
n

X
To explain the reasoning above, consider the case n = 3.
3
(a + b) = (a + b)(a + b)(a + b)
aaa + aab + aba + abb + baa + bab + bba + bbb
3 2 2 3
a + 3a b + 3ab + b

That is, each term in the expansion is obtained by choosing either a or b in each
3 2
factor. The term a is obtained when a is chosen each time, while a b is
obtained when a is selected 2 times, or equivalently, b is selected exactly once.
We will give another proof of this result using mathematical induction. But
rst, we need to prove a result about combinations.

Pascal’s Identity
If n and k are positive integers with k n, then
k = k + k 1 :
n+1 n n

Proof. The result follows from the combination formula.


! +
k + k 1 = k!(n k)! (k 1)!(n ! k + 1)!
n n n n

n!(n k + 1) + n!(k)
=
k!(n k + 1)!

112
All rights reserved. No part of this material may be reproduced or transmitted in any form or by any means -
electronic or mechanical including photocopying – without written permission from the DepEd Central Office.
First Edition, 2016.
=n!(n k + 1 + k)
k!(n + 1 k)!
n!(n + 1)
= k!(n + 1 k)!
(n + 1)!
= k!(n + 1 k)!
= n+1 2
k

Pascal’s identity explains the method of constructing Pascal’s Triangle, in


which an entry is obtained by adding the two numbers above it. This identity
is also an essential part of the second proof of the Binomial Theorem, which
we now state.

The Binomial Theorem For any


positive integer n,
i
n n
(a + b) = i=0
i ani
n
b:

X
Proof. We use mathematical induction.

Part 1
1i=0
ia1i bi =
1
0 a1b0 1
+ 1a0b1 = a + b
1

X
Hence, the formula is true for n = 1.
Part 2. Assume that
(a + b)k = i= 0
k i
k
aki bi:

We want to show that X

k+1
(a + b) = i=0
k+1 k+1 i ak+1i bi:

X
k+1 k
(a + b) = (a + b)(a + b)
0
Xk ki i
(a + b) i a b
i=0

113
All rights reserved. No part of this material may be reproduced or transmitted in any form or by any means -
electronic or mechanical including photocopying – without written permission from the DepEd Central Office. First Edition, 2016.
=a
i=0
k i aki
k
k i
b + b i=0
i aki bi
k

X X
=
i=0
k i aki+1 k i
b + i=0
i aki bi+1
k k

X X
0
k0 ak+1b0 + Xi=1ki ak+1i2 bi 3
+ 0akb1 + 1 ak1 b + 2ak2 b
k k k
+ + k 1 a1bk + k a0bk+1
k k

0
ak+1 + X ki ak+1i bi
i=1

+
i=1
k i k1 ak+1i bi + bk+1

k + 1 ak +1 b0 0
i
+
i=1
k i k
+ i k
1ak+1i b

k + 1 a0bk+1 k + 1
=
i=0k+1 k + 1ak+1i bi i

X
The last expression above follows from Pascal’s Identity.
Therefore, by the Principle of Mathematical Induction,
i
n
(a + b) = i=1
n i ani b
n

X
for any positive integer n. 2

2.4.3. Terms of a Binomial Expansion

We now apply the Binomial Theorem in di erent examples.


6
Example 2.4.4. Use the Binomial Theorem to expand (x + y) .

114
All rights reserved. No part of this material may be reproduced or transmitted in any form or by any means -
electronic or mechanical including photocopying – without written permission from the DepEd Central Office. First Edition, 2016.
Solution.
k
6 6
(x + y) = k=0
k x6k 6
y
X
= 0 x6y0 + 1 x5y1 +2 x4y2
6 6 6
+ 3 x3y3 + 4 x2y4 + 5x1y5
6 6 6

6 0 6
0 6 xy
6 5 4 2 3 3
x + 6x y + 15x y + 20x y
2 2 5 6
+ 15x y + 6xy + y 2
n
Since the expansion of (a + b) begins with k = 0 and ends with k = n, the
expansion has n + 1 terms. The rst term in the expansion is n an = an, the
, and the n 0
n n1 n=1 n n1
second term is 1 a b = na b, the second to the last term is n1 ab =
n1 n n n
nab last term is b = b .
k expansion is n ank +1bk1 . If n is even, there is a

The th term of the k1


n term. If n is odd, there are two middle
middle term, which is the 2 + 1 th
n+1 n+1 th terms.
terms, the th and 2 +1 nk k
2
represented by n a b . Notice that, in any term,
The general term is often k
exponents of a and b is n. The combination
n is the coe cient
the sum of the b k k This allows us to compute any particular term without

of the term involving . n


needing to expand (a + b) and without listing all the other terms.

Example 2.4.5. Find the fth term in the expansion of 2x p y 20.


Solution. The fth term in the expansion of a fth power corresponds to k = 4.
p
4(2x)204 ( y)4 = 4845 65536x16 y2
20

= 317521920x16y2 2

x 6

Example 2.4.6. Find the middle term in the expansion of 2 + 3y .

Solution. Since there are seven terms in the expansion, the middle term is
the fourth term (k = 3), which is
3 3 3
6 x 3 x 135x y

3
3 2 (3y) = 20 8 27y3 = 2 : 2

115
All rights reserved. No part of this material may be reproduced or transmitted in any form or by any means -
electronic or mechanical including photocopying – without written permission from the DepEd Central Office. First Edition, 2016.
Example 2.4.7. Find the term involving x (with exponent 1) in the expansion
y 8
2

2
of x x .
Solution. The general term in the expansion is

8 8k 2y k 8 (2) kyk
2
k x x =k x162k xk

8
=k (2) kx162k k yk
8
k
=k (2) kx163k y :
The term involves x if the exponent of x is 1, which means 16 3k = 1, or
k = 5. Hence, the term is
5 (2) 5xy5 = 1792xy 5: 2
8

?
2.4.4. Approximation and Combination Identities
We continue applying the Binomial Theorem.
? 8
Example 2.4.8. (1) Approximate (0:8) by using the rst three terms in the
expansion of (1 0:2)8. Compare your answer with the calculator value.
8
Use 5 terms in the binomial expansion to approximate (0:8) . Is there an
improvement in the approximation?

Solution.
8 8k k
(0:8) = (1 8 8
0:2) = k=0
k 8
(1) (0:2)

X
8 =
k=0
k 8
(0:2) k

X
(1) 2k=0
k 8
(0:2) k= 0
8
+ 1
8
(0:2) + 2
8
(0:2) 2

X
=1 1:6 + 1:12 = 0:52
The calculator value is 0:16777216, so the error is 0:35222784.

116
All rights reserved. No part of this material may be reproduced or transmitted in any form or by any means -
electronic or mechanical including photocopying – without written permission from the DepEd Central Office.
First Edition, 2016.
2
(2) 4
k=0
k 8
(0:2) k = 0 +
8
1
8
(0:2) + 2 8
(0:2)

X +
3 (0:2) 3 +4 (0:2) 4

8 8

= 0:52 0:448 + 0:112 = 0:184


The error is 0:01622784, which is an improvement on the previous
estimate. 2

Example 2.4.9. Use the Binomial Theorem to prove that, for any positive in-teger n,
n k = 2n: k=0
n

X
n
Solution. Set a = b = 1 in the expansion of (a + b) . Then
n n
2 = (1 + 1) = k=0
n k n
(1)nk n k
(1) = k=0
k :
n
2

X X
Example 2.4.10. Use the Binomial Theorem to prove that
0 + 2 + 4 + + 100
100 100 100 100
= 1 + 3 + 5+ + 99
100 100 100 100

100
Solution. Let a = 1 and b = 1 in the expansion of ( a + b) . Then
k
1+(1) 100 = 100 100 (1)100k (1) :

X
0= 0 + 1 (1) + 2 (1) 2+ 3 (1) 3
100 100 100 100
+ + 99 (1) 99 + 100 (1) 100
100 100

k k
If k is even, then (1) = 1. If k is odd, then (1) = 1. Hence, we have
0= 0 1+23
100 100 100 100
100 100
+ 99 + 100

117
All rights reserved. No part of this material may be reproduced or transmitted in any form or by any means -
electronic or mechanical including photocopying – without written permission from the DepEd Central Office. First Edition, 2016.
Therefore, after transposing the negative terms to other side of the equation, we obtain
0 + 2+ 4 + + 100
100 100 100 100
= 1 + 3 + 5 ++99 2
100 100 100 100

More Solved Examples


4 2 5
1. Use the Binomial Theorem to expand (2x 3y ).
5
k
4 2 5 4 5k 2 20 16 2 12 4
Solution: 2x 3y = 2x 3y = 32x 240x y + 720x y
k=0
86 4 8 X
10

1080x y + 810x y 243y


3 28
2. Determine the 20th term in the expansion of (x 3y) .
2
8

Solution: We see that k = 19 should yield the 20th term, yielding 3 19 19 x27y19.
2 2 20
x x y
2 2
3. Find the term containing y in the expansion of y 2x .
x y 2

; 2
Solution: Setting a = y b = 2x , the (k + 1)th term in the binomial
2
20 x 20k y k (1) k n
expansion is (1) k . To get
=
x203k y3k20
k
ky2x2 2 k
x 21 20

y2 , we get 20 3k = 2 ) k = 6, yielding 26 6 x2y2.


2 16
3 5
4. Determine the term not involving x in the expansion of x + x .
3 2
Solution: Setting a = x ; b = , the (k + 1)th term in the binomial expansion
x5
16 16k 2 k 16
5 k 488k
is k x3 x =2 k x . To get the term without x, we get
6 16
48 8k = 0 ) k = 6, yielding 2 6 .
9 10
Determine the coe cient of x in the expansion of (1 + 2x) .

118
All rights reserved. No part of this material may be reproduced or transmitted in any form or by any means -
electronic or mechanical including photocopying – without written permission from the DepEd Central Office. First Edition, 2016.
Solution: Setting a = 1; b = 2x, the (k + 1)th term in the binomial expansion
10 10
10k k k 9
of the rst factor is k (1) (2x) = 2 k k x . To get x , we set k = 9,
10
9
yielding 2 9 x9.
n
X
k n

n
6. Prove that i=0 (1) k 3nk = 2 .
Solution: Set a = 3; b = 1.
7.is a + b?

p p 5 p p
If 3+
2 is written in the form a 3 + b 2 where a; b are integers, what
p p 5 5 p 5k p k
32 . Note that if
Solution: We have 3+ 2 k X
5

k=0 p

5 k is odd (or equivalently, k is even), the term has a factor of 3, while


5 5 5
p
the rest have a factor of 2. Thus, a = 0 +2 + 4 = 16 and
b= 1 + 3 + 5 = 16 yields a + b = 32.

5 5 5

Supplementary Problems 2.4


1. Use the Binomial Theorem to expand the following:
5
(a) (2x 3y)
px 2 4
2
(b) 3 x
p 4
(c) (1 + x )
2. Without expanding completely, nd the indicated value(s) in the expansion
of the following:

(a) (2 + x)9, two middle terms


p 2 10
(b) 2 + q , 3rd term
2 4 21
(c) (x + y ) , last 2 terms
1 20

(d) px , middle term

119
All rights reserved. No part of this material may be reproduced or transmitted in any form or by any means -
electronic or mechanical including photocopying – without written permission from the DepEd Central Office.
First Edition, 2016.
4 5
y x 15
2
3
(e) x + 4y , term not involving y
13
1
2 2 2
(f) 2x x , term involving x
6 3
(g) (1 2x) , coe cient of x
(h) 2y7=3 2y5=3 30 , coe cient of y
1

1
2

p
8 7=2
0 ( x 3) , coe cient of x
p 6 3=2
1 ( x + 2) , coe cient of x
10 20
Approximate (2:1) by using the rst 5 terms in the expansion of (2 + 0:1) .
Compare your answer with the calculator result.
34
In the expansion of (4x + 3) , the kth value and the (k + 1)st terms have
equal coe cients. What is the value of k?

Determine the value of


0 1 3 + 2 32 3 33 +:::+ 18 318 19 319
19 19 19 19 19 19

120
All rights reserved. No part of this material may be reproduced or transmitted in any form or by any means -
electronic or mechanical including photocopying – without written permission from the DepEd Central Office. First Edition, 2016.
Topic Test 1 for Unit 2
Determine if the given sequence is arithmetic, geometric, or neither by
writing A, G, or O, respectively.

1 1 3 9 27
0 3; 2; 4; 16 ; 32; : : :
1 1 1 1 1
1 2; 7; 12 ; 17 ; 21 ;:::
2 0;3;8;15;24;:::

Three numbers form an arithmetic sequence, the common di erence being 5.


If the last number is increased by 1, the second by 2, and the rst by 4, the
resulting numbers form a geometric sequence. Find the numbers.
3 2
50 2i + 9i + 13i + 6
Xi
3. Evaluate the sum .

=1 i2 + 3i + 2

4. Find the indicated terms in the expansion of the given expression.


8 3 28
1 (b) (n 3m) , 20th term

8
(a) x2 2 , term involving x
Prove the statement below for all positive integers n by mathematical induc-
tion.
1 1 1 n
1 3 +3 5 + + (2n 1)(2n + 1) =2n + 1
On his 20th birthday, Ian deposited an amount of 10,000 pesos to a time-deposit
scheme with a yearly interest of 4%. Ian decides not to withdraw any amount
of money or earnings and vows to keep it in the same time-deposit scheme
year after year. Show that the new amounts in Ian’s time-deposit account in
each succeeding birthday represent a geometric sequence, and use this to
determine the value of the money during Ian’s 60th birthday.

121
All rights reserved. No part of this material may be reproduced or transmitted in any form or by any means -
electronic or mechanical including photocopying – without written permission from the DepEd Central Office. First Edition, 2016.
Topic Test 2 for Unit 2
Determine if the given sequence is arithmetic, geometric, or neither by writing
A, G, or O, respectively.

2 8 32 128 512
0 3; 15 ; 75; 375; 1875 ;:::
1 2 3 4 5
1 2; 3; 4; 5; 6; : : :
11 21
2 3; 2;8; 2;13;:::
The sum of the rst two terms of an arithmetic sequence is 9 and the sum of
the rst three terms is also 9. How many terms must be taken to give a sum
of 126?

Evaluate the following sums.


50 30 i2 2i + 1
X X

(a) i=1 (2i + 1)(i 3) (b) i=1 r 4


3 1 8
Find the term not involving x in the expansion of x + x .

Prove that the following statements are true for all positive integers n by
mathematical induction.
n(3n 1)
(a) 1 + 4 + 7 + : : : + (3n 2) =
n n1
(b) 3 + 7 + 8 is divisible by 12.

122
All rights reserved. No part of this material may be reproduced or transmitted in any form or by any means -
electronic or mechanical including photocopying – without written permission from the DepEd Central Office. First Edition, 2016.
Unit 3

Trigonometry

Puerto Princesa Subterranean River National Park, by Giovanni G. Navata, 12 November 2010,

https://commons.wikimedia.org/wiki/File%3AUnderground River.jpg. Public Domain

Named as one of the New Seven Wonders of Nature in 2012 by the


New7Wonders Foundation, the Puerto Princesa Subterranean River National
Park is world-famous for its limestone karst mountain landscape with an
underground river. The Park was also listed as UNESCO World Heritage Site
in 1999. The under-ground river stretches about 8.2 km long, making it one of
the world’s longest rivers of its kind.

123
All rights reserved. No part of this material may be reproduced or transmitted in any form or by any means -
electronic or mechanical including photocopying – without written permission from the DepEd Central Office. First Edition, 2016.
Lesson 3.1. Angles in a Unit Circle

Learning Outcomes of the Lesson


At the end of the lesson, the student is able to:
illustrate the unit circle and the relationship between the linear and angular
measures of arcs in a unit circle.
convert degree measure to radian measure, and vice versa.
illustrate angles in standard position and coterminal angles.

Lesson Outline
Linear and angular measure of arcs
Conversion of degree to radian, and vice versa
Arc length and area of the sector
Angle in standard position and coterminal angles

Introduction
Angles are being used in several elds like engineering, medical imaging,
elec-tronics, astronomy, geography and many more. Added to that,
surveyors, pilots, landscapers, designers, soldiers, and people in many other
professions heavily use angles and trigonometry to accomplish a variety of
practical tasks. In this les-son, we will deal with the basics of angle measures
together with arc length and sectors.

3.1.1. Angle Measure

An angle is formed by rotating a ray about its endpoint. In the gure shown
below, the initial side of \AOB is OA, while its terminal side is OB. An angle is
said to be positive if the ray rotates in a counterclockwise direction, and the
angle is negative if it rotates in a clockwise direction.

124
All rights reserved. No part of this material may be reproduced or transmitted in any form or by any means -
electronic or mechanical including photocopying – without written permission from the DepEd Central Office. First Edition, 2016.
An angle is in standard position if it is drawn in the xy-plane with its vertex
at the origin and its initial side on the positive x-axis. The angles , , and in the
following gure are angles in standard position.

To measure angles, we use degrees, minutes, seconds, and radians.

A central angle of a circle measures one degree, written 1 , if it inter-


1 0
cepts 360 of the circumference of the circle. One minute, written 1 , is
1 00 1 0
60 of 1 , while one second, written 1 , is 60 of 1 .

For example, in degrees, minutes, and seconds,


18
10 3001800 = 10 30 + 60
0

0
10 30:3
10 + 30:3 60
10:505

and
79:251 = 79 (0:251 60)0
0
79 15:06
0 00
79 15 (0:06 60)
0 00
79 15 3:6 :

Recall that the unit circle is the circle with center at the origin and radius 1
unit.

125
All rights reserved. No part of this material may be reproduced or transmitted in any form or by any means -
electronic or mechanical including photocopying – without written permission from the DepEd Central Office. First Edition, 2016.
A central angle of the unit circle that intercepts an arc of the circle
with length 1 unit is said to have a measure of one radian, written
1 rad. See Figure 3.1.

Figure 3.1

In trigonometry, as it was studied in Grade 9, the degree measure is often used.


On the other hand, in some elds of mathematics like calculus, radian measure of
angles is preferred. Radian measure allows us to treat the trigonometric functions
as functions with the set of real numbers as domains, rather than angles.

Example 3.1.1. In the following gure, identify the terminal side of an angle in
standard position with given measure.

(1) degree measure: 135 , 135 , 90 , 405


3 3
(2) radian measure: 4
rad, 4
rad, 2
rad, 2
rad

126
All rights reserved. No part of this material may be reproduced or transmitted in any form or by any means -
electronic or mechanical including photocopying – without written permission from the DepEd Central Office.
First Edition, 2016.
OC OD OE OB
! ! ! !
Solution. (1) 135 : ; 135 : ;90 : ; and 405 :
! ! !
(2) radian measure: rad: OB; 3 rad: OD; 3 rad: OE; and rad:
4 4 2 2
! 2
OE
Since a unit circle has circumference 2 , a central angle that measures
360 has measure equivalent to 2 radians. Thus, we obtain the following
conversion rules.

Converting degree to radian, and vice versa

To convert a degree measure to radian, multiply it by 180 .


180
To convert a radian measure to degree, multiply it by .

Figure 3.2 shows some special angles in standard position with the
indicated terminal sides. The degree and radian measures are also given.

Figure 3.2

Example 3.1.2. Express 75 and 240 in radians.

Solution.
5 5

75 180 = 12 =) 75 = 12 rad
4 4

240 180 = 3 =) 240 = 3 rad 2


127
All rights reserved. No part of this material may be reproduced or transmitted in any form or by any means -
electronic or mechanical including photocopying – without written permission from the DepEd Central Office. First Edition, 2016.
11
Example 3.1.3. Express 8 rad and 6 rad in degrees.

Solution.
180
8 = 22:5 =) 8 rad = 22:5

11 180 11 rad = 330 2


6 6

3.1.2. Coterminal Angles

Two angles in standard position that have a common terminal side are called
coterminal angles. Observe that the degree measures of coterminal angles di
er by multiples of 360 .

Two angles are coterminal if and only if their degree measures di


er by 360k, where k 2 Z.
Similarly, two angles are coterminal if and only if their radian
mea-sures di er by 2 k, where k 2 Z.

As a quick illustration, to nd one coterminal angle with an angle that mea-


sures 410 , just subtract 360 , resulting in 50 . See Figure 3.3.

Figure 3.3

Example 3.1.4. Find the angle coterminal with 380 that has measure

(1) between 0 and 360 , and

128
All rights reserved. No part of this material may be reproduced or transmitted in any form or by any means -
electronic or mechanical including photocopying – without written permission from the DepEd Central Office. First Edition, 2016.
(2) between 360 and 0 .

Solution. A negative angle moves in a clockwise direction, and the angle 380
lies in Quadrant IV.

(1) 380 + 2 360 = 340

(2) 380 +360 =20 2

3.1.3. Arc Length and Area of a Sector

In a circle, a central angle whose radian measure is subtends an arc that is


the fraction 2 of the circumference of the circle. Thus, in a circle of radius r
(see Figure 3.4), the length s of an arc that subtends the angle is

s = 2 circumference of circle = 2 (2 r) = r :

Figure 3.4

In a circle of radius r, the length s of an arc intercepted by a central


angle with measure radians is given by

s=r:

Example 3.1.5. Find the length of an arc of a circle with radius 10 m that
subtends a central angle of 30 .

Solution. Since the given central angle is in degrees, we have to convert it


into radian measure. Then apply the formula for an arc length.

30 = rad
180 6
129
All rights reserved. No part of this material may be reproduced or transmitted in any form or by any means -
electronic or mechanical including photocopying – without written permission from the DepEd Central Office.
First Edition, 2016.
5

s = 10 6 = 3 m 2
Example 3.1.6. A central angle in a circle of radius 4 m is subtended by an
arc of length 6 m. Find the measure of in radians.

Solution.
=s = 6 = 3 rad 2
r 4 2
A sector of a circle is the portion of the interior of a circle bounded by the
initial and terminal sides of a central angle and its intercepted arc. It is like
a \slice of pizza." Note that an angle with measure 2 radians will de ne a
sector that corresponds to the whole \pizza." Therefore, if a central angle of a
sector has measure radians, then the sector makes up the fraction 2 of a
complete circle. See Figure 3.5. Since the area of a complete circle with
2
radius r is r , we have
2 1 2
Area of a sector = 2 ( r ) = 2 r :

Figure 3.5

In a circle of radius r, the area A of a sector with a central angle


measuring radians is
1 2
= 2r :

Example 3.1.7. Find the area of a sector of a circle with central angle 60 if
the radius of the circle is 3 m.

Solution. First, we have to convert 60 into radians. Then apply the formula for
computing the area of a sector.

60= rad
180 3
130
All rights reserved. No part of this material may be reproduced or transmitted in any form or by any means -
electronic or mechanical including photocopying – without written permission from the DepEd Central Office.
First Edition, 2016.
1 2 3 2

A = 2 (3 ) 3 = 2 m 2
Example 3.1.8. A sprinkler on a golf course fairway is set to spray water over
a distance of 70 feet and rotates through an angle of 120 . Find the area of
the fairway watered by the sprinkler.

Solution.
120 = 2 rad
180 3
1 2 2 4900 2

A = 2 (70 ) 3 = 3 5131 ft 2

More Solved Examples


5
1. Find the equivalent degree measure of 48 radians.
5 5 180 75

Solution: 48 rad = 48 = 4
Find the equivalent angle measure in degrees and in radians of an angle
3
tracing 2 5 revolutions.
Solution: One revolution around a circle is equivalent to tracing 360 .
25 rev = 25 rev 1 rev = 936
3 3 360

26

936 = 936 180 = 5 rad


Find the smallest positive angle coterminal with 2016 .
Solution: Add 6 complete revolutions or 6(360 ) = 2160 to the given angle
(or keep on adding 360 until you get a positive angle).

2016 + 2160 = 144

137
Find the largest negative angle coterminal with 5 .
Solution: Subtract 14 complete revolutions or 14(2 ) = 28 to the given
angle (or keep on subtracting 2 until you get a negative angle).

137 3
28 = rad
5 5

131
All rights reserved. No part of this material may be reproduced or transmitted in any form or by any means -
electronic or mechanical including photocopying – without written permission from the DepEd Central Office. First Edition, 2016.
Find the length of the arc of a circle with radius 15 cm that subtends a central
angle of 84 .
Solution:
7
84 = 84 = rad
180 15
s = 15 7 = 7 cm
15

A central angle in a circle of radius 12 inches is subtended by an arc of


length 27 inches. Find the measure of in degrees.
Solution:
s
s=r =) =r

12 9
= 27 = 4rad
9 9 180 405

4 rad = 4 =
Find the area of a sector of a circle with central angle of 108 if the radius of
the circle is 15 cm.
Solution:
3
108 = 108 = rad
180 5
1 3 135
A = (15)2 = cm2
2 5 2

Given isosceles right triangle ABC with AC as the hypotenuse (as shown
below), a circle with center at A and radius AB intersects AC at D. What is
the ratio of the area of sector BAD to the area of the region BCD?

132
All rights reserved. No part of this material may be reproduced or transmitted in any form or by any means -
electronic or mechanical including photocopying – without written permission from the DepEd Central Office. First Edition, 2016.
Solution: Let r be the radius of the circle; that is, r = AB.
2
1 r
2
\A = 4 rad =) Area of sector BAD = 2r 4 = 8 2 2
4r r
Area of region BCD = Area of 4ABC Area 2of sector BAD = 8
r
area of sector BAD 8
= 2
4r r
2
=
area of the region BCD 8 4

Supplementary Problems 3.1


1
0 How many degrees is 1 5 of a complete revolution?
11
1 How many radians is 5 of a complete revolution?
2 What is the length of an arc of a circle with radius 4 cm that subtends a
central angle of 216 ?
6
3 Find the length of an arc of a circle with radius cm that subtends a
central angle of 99 .
4 What is the smallest positive angle coterminal with 2110 ?
107
5 Find the largest negative angle coterminal with 6 .
7
6 Find the area of a sector of a circle with central angle of 6 if the diameter
of the circle is 9 cm.
7 Find the area of a sector of a circle with central angle of 108 if the radius
of the circle is 15 cm.

8 What is the radius of a circle in which a central angle of 150 determines a


2
sector of area 15 in ?
5
9 Find the radius of a circle in which a central angle of 4 determines a
2
sector of area 32 in .
11. A central angle of a circle of radius 6 inches is subtended by an arc of length 6 inches. What is
the central angle in degrees (rounded to two decimal places)?
2
12. An arc of length 5 cm subtends a central angle of a circle with radius 3 cm. What is in degrees?

133
All rights reserved. No part of this material may be reproduced or transmitted in any form or by any means -
electronic or mechanical including photocopying – without written permission from the DepEd Central Office. First Edition, 2016.
Two overlapping circles of radii 1 cm are drawn such that each circle passes
through the center of the other. What is the perimeter of the entire region?

The length of arc AB of a circle with center at O is equal to twice the length of the
radius r of the circle. Find the area of sector AOB in terms of r.
The angle of a sector in a given circle is 20 and the area of the sector is equal
2
to 800 cm . Find the arc length of the sector.
In Figure 3.6, AE and BC are arcs of two concentric circles with center at D. If
AD = 2 cm, BD = 8 cm, and \ADE = 75 , nd the area of the region AECB.
In Figure 3.7, AB and DE are diameters. If AB = 12 cm and \AOD = 126 , nd
the area of the shaded region.

Figure 3.6 Figure 3.7 Figure 3.8

A point moves outside an equilateral triangle of side 5 cm such that its


distance from the triangle is always 2 cm. See Figure 3.8. What is the
length of one complete path that the point traces?

Figure 3.9
The segment of a circle is the region bounded by a chord and the arc
subtended by the chord. See Figure 3.9. Find the area of a segment of a
circle with a central angle of 120 and a radius of 64 cm.

134
All rights reserved. No part of this material may be reproduced or transmitted in any form or by any means -
electronic or mechanical including photocopying – without written permission from the DepEd Central Office. First Edition, 2016.
Figure 3.10

In Figure 3.10, diameter AB of circle O measures 12 cm and arc BC measures


120 . Find the area of the shaded region.

Lesson 3.2. Circular Functions

Learning Outcomes of the Lesson


At the end of the lesson, the student is able to:
illustrate the di erent circular functions; and
use reference angles to nd exact values of circular functions.

Lesson Outline
Circular functions
Reference angles

Introduction
We de ne the six trigonometric function in such a way that the domain of
each function is the set of angles in standard position. The angles are measured
either in degrees or radians. In this lesson, we will modify these trigonometric
functions so that the domain will be real numbers rather than set of angles.

135
All rights reserved. No part of this material may be reproduced or transmitted in any form or by any means -
electronic or mechanical including photocopying – without written permission from the DepEd Central Office. First Edition, 2016.
3.2.1. Circular Functions on Real Numbers

Recall that the sine and cosine functions (and four others: tangent, cosecant,
secant, and cotangent) of angles measuring between 0 and 90 were de ned
in the last quarter of Grade 9 as ratios of sides of a right triangle. It can be
veri ed that these de nitions are special cases of the following de nition.

Let be an angle in standard position and P ( ) = P (x; y) the point


on its terminal side on the unit circle. De ne
1
sin = y csc = y ; y 6= 0
1
cos = x sec = x ; x 6= 0
y x
tan = x; x 6= 0 cot = y ; y 6= 0

Example 3.2.1. Find the values of cos 135 , tan 135 , sin(60 ), and sec(60 ).

Solution. Refer to Figure 3.11(a).

(a) (b)

Figure 3.11

From properties of 45 -45 and 30 -60 right triangles (with hypotenuse 1


unit), we obtain the lengths of the legs as in Figure 3.11(b). Thus, the
coordinates of A and B are
p p2 1 p3
2

A= 2 ; 2 ! and B = 2 ; 2 !:

136
All rights reserved. No part of this material may be reproduced or transmitted in any form or by any means -
electronic or mechanical including photocopying – without written permission from the DepEd Central Office. First Edition, 2016.
3
2
Therefore, we get
p
2
cos 135 = 2 ; tan 135 = 1;
p
3
sin(60 ) = 2 ; andsec(60 ) = 2: 2
2
From the last example, we may then also say
that p cos 4 rad = 2
and so on.

p
3
2 ;

From the above de nitions, we de ne the same six functions on real numbers.
These functions are called trigonometric functions.

Let s be any real number. Suppose is the angle in standard


position with measure s rad. Then we de ne

sin s = sin csc s = csc

cos s = cos sec s = sec


tan s = tan cot s = cot

From the last example, we then have


p
2

cos 4 = cos 4 rad = cos 45 = 2


and
p
3

sin 3 = sin 3 rad = sin(60 ) = 2 :


In the same way, we have
tan 0 = tan(0 rad) = tan 0 = 0:

3 3 3
Example 3.2.2. Find the exact values of sin 2 , cos 2 , and tan 2 .
Solution. Let P be the point on the unit circle and on the terminal side of

3 3
the angle in the standard position with measure 2 rad. Then P 2 = (0; 1),
and so
sin 3 = 1; cos 3 = 0;
2 2
but tan 3 is unde ned. 2
2

137
All rights reserved. No part of this material may be reproduced or transmitted in any form or by any means -
electronic or mechanical including photocopying – without written permission from the DepEd Central Office.
First Edition, 2016.
3
Example 3.2.3. Suppose s is a real number such that sin s = 4
and cos s > 0.
Find cos s.

Solution. We may consider s as the angle with measure s rad. Let P (s) = (x;
y) be the point on the unit circle and on the terminal side of angle s.
2 2
Since P (s) is on the unit circle, we know that x + y = 1. Since sin s = y =
3
4
, we get

3 2 7 p7
2 2
x =1 y =1 4 = 16 =) x = 4 :
p 7
Since cos s = x > 0, we have cos s = 4
. 2

Let P (x1; y1) and Q(x; y) be points on the terminal side of an angle in
standard position, where P is on the unit circle and Q on the circle of radius r
(not necessarily 1) with center also at the origin, as shown above. Observe
that we can use similar triangles to obtain

cos = x1 =x1 =x and sin = y1 = y1 =y :


1 r 1 r
We may then further generalize the de nitions of the six circular functions.

138
All rights reserved. No part of this material may be reproduced or transmitted in any form or by any means -
electronic or mechanical including photocopying – without written permission from the DepEd Central Office. First Edition, 2016.
Let be an angle in standard position, Q(x; y) any point on the ter-
p

2 2
minal side of , and r = x + y > 0. Then
y r
sin = r csc = y ; y 6= 0
x r
cos = r sec = x ; x 6= 0
y x
tan = x ;x 6= 0 cot = y ; y 6= 0

3
We then have a second solution for Example 3.2.3 as follows. With sin s = 4
and sin s = y , we may choose y = 3 and r = 4 (which is always positive). In
r
x
this case, we can solve for x, which is positive since cos s = 4
is given to be
positive.
p
7
p 2 2
p
4= x + (3) =) x= 7 =) cos s = 4
3.2.2. Reference Angle

We observe that if 1 and 2 are coterminal angles, the values of the six circular or
trigonometric functions at 1 agree with the values at 2. Therefore, in nding the value of a
circular function at a number , we can always reduce to a number
between 0 and 2 . For example, sin 14
3
= sin 14
3
4 = sin 2 . Also, observe
3
2

from Figure 3.12 that sin 3 = sin 3 .

Figure 3.12

139
All rights reserved. No part of this material may be reproduced or transmitted in any form or by any means -
electronic or mechanical including photocopying – without written permission from the DepEd Central Office. First Edition, 2016.
In general, if 1, 2, 3, and 4 are as shown in Figure 3.13 with P ( 1) = (x1; y1),
then each of the x-coordinates of P ( 2), P ( 3), and P ( 4) is x1, while the y-
coordinate is y1. The correct sign is determined by the location of the angle.
Therefore, together with the correct sign, the value of a particular circular
function at an angle can be determined by its value at an angle 1 with radian
measure between 0 and 2 . The angle 1 is called the reference angle of .

Figure 3.13

The signs of the coordinates of P ( ) depends on the quadrant or axis


where it terminates. It is important to know the sign of each circular function
in each quadrant. See Figure 3.14. It is not necessary to memorize the table,
since the sign of each function for each quadrant is easily determined from
its de nition. We note that the signs of cosecant, secant, and cotangent are
the same as sine, cosine, and tangent, respectively.

Figure 3.14

Using the fact that the unit circle is symmetric with respect to the x-axis, the
y-axis, and the origin, we can identify the coordinates of all the points using the

140
All rights reserved. No part of this material may be reproduced or transmitted in any form or by any means -
electronic or mechanical including photocopying – without written permission from the DepEd Central Office. First Edition, 2016.
coordinates of corresponding points in the Quadrant I, as shown in Figure
3.15 for the special angles.

Figure 3.15

Example 3.2.4. Use reference angle and appropriate sign to nd the exact value of each expression.
(1) sin 11 and cos 11 (3) sin 150
6 6

7
8

(2) cos 6 (4) tan 3


11
Solution. (1) The reference angle of 6 is 6 , and it lies in Quadrant IV
wherein sine and cosine are negative and positive, respectively.

sin 11 = sin = 1
6 6 2
p
cos 11 = cos = 3
6 6 2
7
(2) The angle 6 lies in Quadrant II wherein cosine is negative, and its refer-
ence angle is . p
6

7 3
cos = cos =
6 6 2
(3) sin 150 = sin 30 = 1
2
p 3
(4) tan 8 = tan = sin 3 = 2 = p 3 2
3 3 cos 1
3 2

141
All rights reserved. No part of this material may be reproduced or transmitted in any form or by any means -
electronic or mechanical including photocopying – without written permission from the DepEd Central Office. First Edition, 2016.
More Solved Examples
17
If P ( ) is a point on the unit circle and = 3 , what are the coordinates of
P( )?
Solution: 17 is coterminal with 5 which terminates in QIV. The reference
3 3 p
17 1 3

angle is 3 , therefore P 3 =2 ; 2 . 5
2. If P ( ) is a point on the unit circle and = 6 , nd the values of the six
trigonometric functions of .
Solution: The angle 5 terminates in QIII, the reference angle is , therefore
6 6

5
p 3 1

P 6 = 2 ; 2 .
p 2 2p 3
5 3 5

cos 6 = 2 sec 6 = p3 = 3
1 5
=
5

sin 6 2 csc 6=2


1 p3 5 p3
5

p
tan 6 =p3 = 3 cot 6= 1 = 3
3. Find the six trigonometric functions of the angle if the terminal side of in
standard position passes through the point (5; 12).

p 2 2
Solution: x = 5, y = 12, r= (5) + (12) = 13.
cos = x = 5 sec = r = 13
r 13 x 5
sin = y = 12 csc = r = 13
r 13 y 12
y 12 x 5
tan = = cot = =
x 5 y 12

25 3
4. Given sec = 24 and 2 , nd sin + cos .
2
Solution: r = 25, x = 24, y= (25) (24) 2= 7.

Since is in QIII, y = 7. p
sin + cos = 7 + 24 = 31 :
25 25 25

5. If tan A = 4 , determine 2 sin Acos A .


5 3 cos A

142
All rights reserved. No part of this material may be reproduced or transmitted in any form or by any means -
electronic or mechanical including photocopying – without written permission from the DepEd Central Office. First Edition, 2016.
29
6

11
6 , what are the coordinates

2 , nd sec +tan
3 sec tan

4
3 and
Solution:
4 sin A 4
3 cos A tan A = 5 =) cos A = 5 5 3(1) = 5
=3 cos A 3 cos A = 3

2 sin A cos A 2 sin A 1 cos A 2 4 1 1


What is the reference angle of Solution: 29
29
6 is coterminal with ? Find the value of tan 6 .
7 angle is .
29 6 in QIII, so its reference 6
tan
6 p
3

= tan 6 = 3
5
For what angle in the third quadrant is cos = sin 3 ?
Solution:
5
sin = cos
3
p3 7
cos = 2 and in QIII =) = 6

Supplementary Problems 3.2


33
1. In what quadrant is P ( ) located if = 4 ?
17
2. In what quadrant is P ( ) located if = 6
?

In what quadrant is P ( ) located if sec > 0 and cot < 0?

In what quadrant is P ( ) located if tan > 0 and cos < 0 ?


5
If P ( ) is a point on the unit circle and = 6 , what are the coordinates of P( )?

6. If P ( ) is a point on the unit circle and =


of P ( )?

7. If cos > 0 and tan =


3
If tan = 5 and is in QIII, what is sec ?
If csc = 2 and cos < 0, nd sec .

10. Find the values of the other trigonometric functions of if cot = sin <
0.

143
All rights reserved. No part of this material may be reproduced or transmitted in any form or by any means -
electronic or mechanical including photocopying – without written permission from the DepEd Central Office. First Edition, 2016.
11. Find the values of the other trigonometric functions of if csc = 4 and does
not terminate in QIII.

The terminal side of an angle in standard position contains the point (7; 1).
Find the values of the six trigonometric functions of .

13. The terminal side of an angle in standard position contains the point (2; 4).
Find the values of the six trigonometric functions of .

14. If the terminal point of an arc of length lies on the line joining the origin
2 2
and the point (3; 1), what is cos sin ?
15. If the terminal point of an arc of length lies on the line joining the origin
2 2
and the point (2; 6), what is sec csc ?
35 35
Determine the reference angle of 4 , and nd cos 4 .
3 2
If 2 < < 2 , nd if cos = sin 3 .
Evaluate the sum of sin 30 + sin 60 + sin 90 + + sin 510 + sin 540 .
7
19. If f(x) = sin 2x + cos 2x + sec 2x + csc 2x + tan 2x + cot 2x, what is f 8 ?
13 25
20. Evaluate the sum of sec 6 + sec 6 + sec 6 + + sec 1096 .

Lesson 3.3. Graphs of Circular Functions and Situational


Problems

Learning Outcomes of the Lesson


At the end of the lesson, the student is able to:
determine the domain and range of the di erent circular functions;
graph the six circular functions with its amplitude, period, and phase shift;
and
solve situational problems involving circular functions.

Lesson Outline
Domain and range of circular functions
Graphs of circular functions
Amplitude, period, and phase shift

144
All rights reserved. No part of this material may be reproduced or transmitted in any form or by any means -
electronic or mechanical including photocopying – without written permission from the DepEd Central Office. First Edition, 2016.
Introduction
There are many things that occur periodically. Phenomena like rotation of
the planets and comets, high and low tides, and yearly change of the
seasons follow a periodic pattern. In this lesson, we will graph the six circular
functions and we will see that they are periodic in nature.

3.3.1. Graphs of y = sin x and y = cos x

Recall that, for a real number x, sin x = sin for an angle with measure x
radians, and that sin is the second coordinate of the point P ( ) on the unit
circle. Since each x corresponds to an angle , we can conclude that

sin x is de ned for any real number x or the domain of the sine function is
R, and
the range of sine is the set of all real numbers between 1 and 1 (inclusive).

From the de nition, it also follows that sin(x+2 ) = sin x for any real number x.
This means that the values of the sine function repeat every 2 units. In this case,
we say that the sine function is a periodic function with period 2 .
Table 3.16 below shows the values of y = sin x, where x is the equivalent
radian measure of the special angles and their multiples from 0 to 2 . As
commented above, these values determine the behavior of the function on R.

2 3 5
x 0 6 4 3 2 3 4 6
1 p 2 p3 p3 p2 1
y 0 2 2 2
1 2 2 2
0
0 0:5 0:71 0:87 1 0:87 0:71 0:5 0
7 5 4 3 5 7 11
x 6 4 3 2 3 4 6
2
1 p 2 p 3 p p2 1
y 2 2 2
1 2
3

2 2
0
0:5 0:71 0:87 1 0:87 0:71 0:5 0
Table 3.16

From the table, we can observe that as x increases from 0 to 2 , sin x also
3
increases from 0 to 1. Similarly, as x increases from 2 to 2 , sin x also
increases from 1 to 0. On the other hand, notice that as x increases from 2
3
to , sin x decreases from 1 to 0. Similarly, as x increases from to 2 , sin x
decreases from 0 to 1.

145
All rights reserved. No part of this material may be reproduced or transmitted in any form or by any means -
electronic or mechanical including photocopying – without written permission from the DepEd Central Office. First Edition, 2016.
To sketch the graph of y = sin x, we plot the points presented in Table 3.16,
and join them with a smooth curve. See Figure 3.17. Since the graph repeats
every 2 units, Figure 3.18 shows periodic graph over a longer interval.

Figure 3.17

Figure 3.18

We can make observations about the cosine function that are similar to
the sine function.

y = cos x has domain R and range [1; 1].

y = cos x is periodic with period 2 . The graph of y = cos x is shown in


Figure 3.19.

Figure 3.19

146
All rights reserved. No part of this material may be reproduced or transmitted in any form or by any means -
electronic or mechanical including photocopying – without written permission from the DepEd Central Office. First Edition, 2016.
From the graphs of y = sin x and y = cos x in Figures 3.18 and 3.19, re-
spectively, we observe that sin(x) = sin x and cos(x) = cos x for any real
number x. In other words, the graphs of y = cos(x) and y = cos x are the
same, while the graph of y = sin(x) is the same as that of y = sin x.
In general, if a function f satis es the property that f(x) = f(x) for all x in its
domain, we say that such function is even. On the other hand, we say that a
function f is odd if f(x) = f (x) for all x in its domain. For example, the functions
2 3
x and cos x are even, while the functions x 3x and sin x are odd.

3.3.2. Graphs of y = a sin bx and y = a cos bx

Using a table of values from 0 to 2 , we can sketch the graph of y = 3 sin x,


and compare it to the graph of y = sin x. See Figure 3.20 wherein the solid
curve belongs to y = 3 sin x, while the dashed curve to y = sin x. For
instance, if x = 2 , then y = 1 when y = sin x, and y = 3 when y = 3 sin x. The
period, x-intercepts, and domains are the same for both graphs, while they di
er in the range. The range of y = 3 sin x is [3; 3].

Figure 3.20

In general, the graphs of y = a sin x and y = a cos x with a > 0 have the
same shape as the graphs of y = sin x and y = cos x, respectively. If a < 0,
there is a re ection across the x-axis.

In the graphs of y = a sin x and y = a cos x, the number jaj is called


its amplitude. It dictates the height of the curve. When jaj < 1, the
graphs are shrunk vertically, and when jaj > 1, the graphs are
stretched vertically.

Now, in Table 3.21, we consider the values of y = sin 2x on [0; 2 ].

147
All rights reserved. No part of this material may be reproduced or transmitted in any form or by any means -
electronic or mechanical including photocopying – without written permission from the DepEd Central Office.
First Edition, 2016.
2 3 5
x 0 6 4 3 2 3 4 6
p3 p 3 p3 p3
y 0 2
1 2
0 2
1 2
0
0 0:87 1 0:87 0 0:87 1 0:87 0
7 5 4 3 5 7 11
x 6 4 3 2 3 4 6
2
p p 3 p3 p 3
y 2
3
1 2
0 2
1 2
0
0:87 1 0:87 0 0:87 1 0:87 0
Table 3.21

Figure 3.22

Figure 3.22 shows the graphs of y = sin 2x (solid curve) and y = sin x
(dashed curve) over the interval [0; 2 ]. Notice that, for sin 2x to generate
periodic values similar to [0; 2 ] for y = sin x, we just need values of x from 0
to . We then expect the values of sin 2x to repeat every units thereafter. The
period of y = sin 2x is .

2
If b =6 0, then both y = sin bx and y = cos bx have period given by jbj .
If 0 < jbj < 1, the graphs are stretched horizontally, and if jbj > 1, the
graphs are shrunk horizontally.

To sketch the graphs of y = a sin bx and y = a cos bx, a; b 6= 0, we may


proceed with the following steps:

2
Determine the amplitude jaj, and nd the period jbj . To draw one cycle of
the graph (that is, one complete graph for one period), we just need to
2
complete the graph from 0 to jbj .

148
All rights reserved. No part of this material may be reproduced or transmitted in any form or by any means -
electronic or mechanical including photocopying – without written permission from the DepEd Central Office. First Edition, 2016.
Divide the interval into four equal parts, and get ve division points: x 1 = 0,
2
x2, x3, x4, and x5 = jbj , where x3 is the midpoint between x1 and x5 (that
1
is, 2 (x1 + x5) = x3), x2 is the midpoint between x 1 and x3, and x4 is the
midpoint between x3 and x5.

Evaluate the function at each of the ve x-values identi ed in Step 2. The


points will correspond to the highest point, lowest point, and x-
intercepts of the graph.

Plot the points found in Step 3, and join them with a smooth curve similar to
the graph of the basic sine curve.

Extend the graph to the right and to the left, as needed.

Example 3.3.1. Sketch the graph of one cycle of y = 2 sin 4x.

2
Solution. (1) The period is 4 = 2 , and the amplitude is 2.
Dividing the interval [0; 2 ] into 4 equal parts, we get the following x-
3
coordinates: 0, 8 , 4 , 8 , and 2 .
When x = 0, 4 , and 2 , we get y = 0. On the other hand, when x = 8 , we
have y = 2 (the amplitude), and y = 2 when x = 38 .
Draw a smooth curve by connecting the points. There is no need to
proceed to Step 5 because the problem only asks for one cycle.

x
Example 3.3.2. Sketch the graph of y = 3 cos 2
.

2
Solution. (1) The amplitude is j 3j = 3, and the period is 1 =4.
2

We divide the interval [0; 4 ] into four equal parts, and we get the following
x-values: 0, , 2 , 3 , and 4 .

149
All rights reserved. No part of this material may be reproduced or transmitted in any form or by any means -
electronic or mechanical including photocopying – without written permission from the DepEd Central Office. First Edition, 2016.
(3) We have y = 0 when x = and 3 , y = 3 when x = 0 and 4 , and y = 3
when x = 2 .

We trace the points in Step 3 by a smooth curve.

We extend the pattern in Step 4 to the left and to the right.

1
Example 3.3.3. Sketch the graph of two cycles of y = 2 sin 23x .
2x
Solution. Since the sine function is odd, the graph of y = 12 sin is the same
3
as that of y = 1 sin 2x .
2 3

1 2
(1) The amplitude is 2 , and the period is 2 =3.
3

Dividing the interval [0; 3 ] into four equal parts, we get the x-coordinates of
the ve important points:
0+33 0+ 3
2
3 3 +3 9
= ; = ; 2
= :
2 2 2 4 2 4

(3) We get y = 0 when x = 0, 3 , and 3 , y = 1 when 3 , and y = 1 when


9 2 2 4 2
4 .

We trace the points in Step 3 by a smooth curve.

We extend the pattern in Step 4 by one more period to the right.

150
All rights reserved. No part of this material may be reproduced or transmitted in any form or by any means -
electronic or mechanical including photocopying – without written permission from the DepEd Central Office. First Edition, 2016.
3.3.3. Graphs of y = a sin b(x c) + d and y = a cos b(x c) + d

We rst compare the graphs of y = sin x and y = sin x


values and the 5-step procedure discussed earlier. 3 using a table of
As x runs from to 7 , the value of the expression x runs from 0 to 2 . So
3 3 3
for one cycle of the graph of y = sin x , we then expect to have the graph of
3

y = sin x starting from x = 3 . This is con rmed by the values in Table 3.23. We
then apply a similar procedure to complete one cycle of the graph; that is, divide
7
the interval [ 3 ; 3 ] into four equal parts, and then determine the key values of x
in sketching the graphs as discussed earlier. The one-cycle graph of y = sin x
(dashed curve) and the corresponding one-cycle graph of y = sin x (solid

curve) are shown in Figure 3.24. 3


5 4 11 7
x 3 6 3 6 3
3
x 3
0 2 2
2

sin x 3
0 1 0 1 0

Table 3.23

Figure 3.24

Observe that the graph of y = sin x shifts units to the right of


y x 3 3

= sin . Thus, they have the same period,


The graphs of

y = a sin b(x c) and y = a cos b(x c)

have the same shape as y = a sin bx and y = a cos bx,


respectively, but shifted c units to the right when c > 0 and shifted
jcj units to the left if c < 0. The number c is called the phase shift of
the sine or cosine graph.

151
All rights reserved. No part of this material may be reproduced or transmitted in any form or by any means -
electronic or mechanical including photocopying – without written permission from the DepEd Central Office. First Edition, 2016.
Example 3.3.4. In the same Cartesian plane, sketch one cycle of the graphs
of y = 3 sin x and y = 3 sin x + 4 .

Solution. We have sketched the graph of y = 3 sin x earlier at the start of the
lesson. We consider y = 3 sin x + 4 . We expect that it has the same shape
as that of y = 3 sin x, but shifted some units.
Here, we have a = 3, b = 1, and c = 4 . From these constants, we get
the amplitude, the period, and the phase shift, and these are 3, 2 , and 4,
respectively.
One cycle starts at x = and ends at x = + 2 = 7 . We now compute
4 4 4
the important values of x.

4
+ 7 3 + 3 3
4
+ 7 5
4
= ; 44
= ; 4
=
2 4 2 4 2 4

3 5 7
x 4 4 4 4 4

y = 3 sin x + 4 0 3 0 3 0

While the e ect of c in y = a sin b(x c) and y = a cos b(x c) is a


horizontal shift of their graphs from the corresponding graphs of y
= a sin bx and y = a cos bx, the e ect of d in the equations y = a
sin b(x c) + d and y = a cos b(x c) + d is a vertical shift. That is, the
graph of y = a sin b(x c) + d has the same amplitude, period, and
phase shift as that of y = a sin b(x c), but shifted d units upward
when d > 0 and jdj units downward when d < 0.

152
All rights reserved. No part of this material may be reproduced or transmitted in any form or by any means -
electronic or mechanical including photocopying – without written permission from the DepEd Central Office. First Edition, 2016.
Example 3.3.5. Sketch the graph of

y = 2 cos 2 x 6 3:
Solution. Here, a = 2, b = 2, c = 6 , and d = 3. We rst sketch one cycle of the
graph of y = 2 cos 2 x 6 , and then extend this graph to the left and to the right, and then
move the resulting graph 3 units downward.
6 .
The graph of y = 2 cos 2 x 6 has amplitude 2, period , and phase shift

Start of one cycle: 6


7
End of the cycle: 6 + = 6

6
+ 7
6 =
2 6
+ 2
3
5 2 + 7 11
; = ; 3 6
=
2 3 2 12 2 12

5 2 11 7
x 6 12 3 12 6

y = 2 cos 2 x 2 0 2 0 2
y = 2 cos 2 x 6 3 5 3 1 3 5

Before we end this sub-lesson, we make the following observation, which will
be used in the discussion on simple harmonic motion (Sub-Lesson 3.3.6).

153
All rights reserved. No part of this material may be reproduced or transmitted in any form or by any means -
electronic or mechanical including photocopying – without written permission from the DepEd Central Office. First Edition, 2016.
Di erent Equations, The Same Graph

The graphs of y = sin x and y = sin(x + 2 k), k any integer, are the
same.
2. The graphs of y = sin x, y = sin(x + ), y = cos(x 2 ), and
y = cos(x + ) are the same.
2
3. In general, the graphs of

y = a sin b(x c) + d;

y = a sin[b(x c) + + 2 k] + d;
y = a cos[b(x c) 2 + 2 k] + d;
and
y = a cos[b(x c) + + 2 k] + d;
2
where k is any integer, are all the same.
Similar observations are true for cosine.

3.3.4. Graphs of Cosecant and Secant Functions


1
We know that csc x = sin x if sin x 6= 0. Using this relationship, we can sketch
the graph of y = csc x.
First, we observe that the domain of the cosecant function is

fx 2 R : sin x 6= g0= fx 2 R : x 6=k ; k 2 Zg:

Table 3.25 shows the key numbers (that is, numbers where y = sin x crosses
the x-axis, attain its maximum and minimum values) and some neighboring
points, where \und" stands for \unde ned," while Figure 3.26 shows one cycle
of the graphs of y = sin x (dashed curve) and y = csc x (solid curve). Notice
the asymptotes of the graph y = csc x.

5 7 3 11
x 0 6 2 6 6 2 6
2
1 1 1 1
y = sin x 0 2
1 2
0 2
1 2
0
y = csc x und 2 1 2 und 2 1 2 und
Table 3.25

154
All rights reserved. No part of this material may be reproduced or transmitted in any form or by any means -
electronic or mechanical including photocopying – without written permission from the DepEd Central Office. First Edition, 2016.
Figure 3.26

We could also sketch the graph of csc x directly from the graph of y = sin
x by observing the following facts:

(1) If sin x = 1 (or 1), then csc x = 1 (or 1).

At each x-intercept of y = sin x, y = csc x is unde ned; but a vertical


asymptote is formed because, when sin x is close to 0, the value of csc
x will have a big magnitude with the same sign as sin x.

Refer to Figure 3.27 for the graphs of y = sin x (dashed curve) and y = csc x
(solid curve) over a larger interval.

Figure 3.27

155
All rights reserved. No part of this material may be reproduced or transmitted in any form or by any means -
electronic or mechanical including photocopying – without written permission from the DepEd Central Office. First Edition, 2016.
Like the sine and cosecant functions, the cosine and secant functions are
also reciprocals of each other. Therefore, y = sec x has domain

k
fx 2 R : cos x 6= 0g = fx 2 R : x 6= 2 ; k odd integerg:

Similarly, the graph of y = sec x can be obtained from the graph of y = cos x.
These graphs are shown in Figure 3.28.

Figure 3.28

x
Example 3.3.6. Sketch the graph of y = 2 csc 2 .
x
Solution. First, we sketch the graph of y = 2 sin 2 , and use the technique
x
dis-cussed above to sketch the graph of y = 2 csc 2 .

The vertical asymptotes of y = 2 csc x are the x-intercepts of y = 2 sin x:


2 2

x = 0; 2 ; x 4 ; : : :. After setting up the asymptotes, we now sketch the graph


of y = 2 csc 2 as shown below.

156
All rights reserved. No part of this material may be reproduced or transmitted in any form or by any means -
electronic or mechanical including photocopying – without written permission from the DepEd Central Office. First Edition, 2016.
Example 3.3.7. Sketch the graph of y = 2 sec 2x.

Solution. Sketch the graph of y = cos 2x (note that it has period ), then sketch
the graph of y = sec 2x (as illustrated above), and then move the resulting
graph 2 units upward to obtain the graph of y = 2 sec 2x.

157
All rights reserved. No part of this material may be reproduced or transmitted in any form or by any means -
electronic or mechanical including photocopying – without written permission from the DepEd Central Office. First Edition, 2016.
3.3.5. Graphs of Tangent and Cotangent Functions
sin x
We know that tan x = cos x , where cos x 6= 0. From this de nition of the tangent
function, it follows that its domain is the same as that of the secant function,
which is
k
fx 2 R : cos x 6= 0g = fx 2 R : x 6= 2 ; k odd integerg:

We note that tan x = 0 when sin x = 0 (that is, when x = k , k any integer), and
k
that the graph of y = tan x has asymptotes x = 2 , k odd integer. Furthermore,
by recalling the signs of tangent from Quadrant I to Quadrant IV and its values,
we observe that the tangent function is periodic with period .
To sketch the graph of y = tan x, it will be enough to know its one-cycle

graph on the open interval 2 ; 2 . See Table 3.29 and Figure 3.30.
x 2 3 4 6
0
p3
y = tan x und p3 1 3 0

x 6 4 3 2
p 3
y = tan x 3 1 p3 und
Table 3.29

Figure 3.30

cos x
In the same manner, the domain of y = cot x = sin x is
fx 2 R : sin x 6= 0g = fx 2 R : x 6= k ; k 2 Zg;

and its period is also . The graph of y = cot x is shown in Figure 3.31.

158
All rights reserved. No part of this material may be reproduced or transmitted in any form or by any means -
electronic or mechanical including photocopying – without written permission from the DepEd Central Office. First Edition, 2016.
Figure 3.31

In general, to sketch the graphs of y = a tan bx and y = a cot bx, a 6= 0 and


b > 0, we may proceed with the following steps:
for y = a tan bx, and on 0; b for y = a cot bx.
(1) Determine the period . Then we draw one cycle of the graph on ;
b 2b 2b

Determine the two adjacent vertical asymptotes. For y = a tan bx, these
vertical asymptotes are given by x = 2b . For y = a cot bx, the vertical
asymptotes are given by x = 0 and x = b .
Divide the interval formed by the vertical asymptotes in Step 2 into four
equal parts, and get three division points exclusively between the
asymp-totes.

Evaluate the function at each of these x-values identi ed in Step 3. The


points will correspond to the signs and x-intercept of the graph.

Plot the points found in Step 3, and join them with a smooth curve ap-
proaching to the vertical asymptotes. Extend the graph to the right and
to the left, as needed.
1
Example 3.3.8. Sketch the graph of y = 2 tan 2x.

Solution. The period of the function is 2 , and the adjacent asymptotes are x =
; 3 ;::: Dividing the interval ; into four equal parts, the key x-values
4 4 . 4 4

are 8, 0, and 8 .
x 8
0 8

y= 1 tan 2x 1 0 1
2 2 2

159
All rights reserved. No part of this material may be reproduced or transmitted in any form or by any means -
electronic or mechanical including photocopying – without written permission from the DepEd Central Office. First Edition, 2016.
x
Example 3.3.9. Sketch the graph of y = 2 cot 3 on the interval (0; 3 ).

Solution. The period of the function is 3 , and the adjacent asymptotes are x
= 0 and x = 3 . We now divide the interval (0; 3 ) into four equal parts, and
3 3 9
the key x-values are 4 , 2 , and 4 .

3 3 9
x 4 2 4

y = 2 cot x 2 0 2
3

3.3.6. Simple Harmonic Motion

Repetitive or periodic behavior is common in nature. As an example, the


time-telling device known as sundial is a result of the predictable rising and
setting of the sun everyday. It consists of a at plate and a gnomon. As the
sun moves across the sky, the gnomon casts a shadow on the plate, which is
calibrated to tell the time of the day.

160
All rights reserved. No part of this material may be reproduced or transmitted in any form or by any means -
electronic or mechanical including photocopying – without written permission from the DepEd Central Office. First Edition, 2016.
Sundial, by liz west, 29 March 2007,

https://commons.wikimedia.org/wiki/File:Sundial 2r.jpg. Public Domain.

Some motions are also periodic. When a weight is suspended on a


spring, pulled down, and released, the weight oscillates up and down.
Neglecting resis-tance, this oscillatory motion of the weight will continue on
and on, and its height is periodic with respect to time.

t = 0 sec t = 2:8 sec

161
All rights reserved. No part of this material may be reproduced or transmitted in any form or by any means -
electronic or mechanical including photocopying – without written permission from the DepEd Central Office. First Edition, 2016.
t = 6:1 sec t = 9 sec

Periodic motions are usually modeled by either sine or cosine function,


and are called simple harmonic motions. Unimpeded movements of objects
like oscilla-tion, vibration, rotation, and motion due to water waves are real-
life occurrences that behave in simple harmonic motion.

Equations of Simple Harmonic Motion


The displacement y (directed height or length) of an object
behaving in a simple harmonic motion with respect to time t is
given by one of the following equations:

y = a sin b(t c) + d

or
y = a cos b(t c) + d:
In both equations, we have the following information:
1
amplitude = jaj = 2 (M m) - the maximum displacement above
and below the rest position or central position or equilibrium,
where M is the maximum height and m is the minimum height;
2
period = jbj - the time required to complete one cycle (from one
highest or lowest point to the next);
jbj
frequency = 2 - the number of cycles per unit of time;
c - responsible for the horizontal shift in time; and
d - responsible for the vertical shift in displacement.

Example 3.3.10. A weight is suspended from a spring and is moving up and down in
a simple harmonic motion. At start, the weight is pulled down 5 cm below the resting
position, and then released. After 8 seconds, the weight reaches its

162
All rights reserved. No part of this material may be reproduced or transmitted in any form or by any means -
electronic or mechanical including photocopying – without written permission from the DepEd Central Office. First Edition, 2016.
highest location for the rst time. Find the equation of the motion.

Solution. We are given that the weight is located at its lowest position at t =
0; that is, y = 5 when t = 0. Therefore, the equation is y = 5 cos bt.
Because it took the weight 8 seconds from the lowest point to its immediate
highest point, half the period is 8 seconds.
1 2 t
2 b =8 =) b = 8 =) y = 5 cos 8 2
?
Example 3.3.11. Suppose you ride a Ferris wheel. The lowest point of the
wheel is 3 meters o the ground, and its diameter is 20 m. After it started, the
Ferris wheel revolves at a constant speed, and it takes 32 seconds to bring
you back again to the riding point. After riding for 150 seconds, nd your
approximate height above the ground.

Solution. We ignore rst the xed value of 3 m o the ground, and assume that
the central position passes through the center of the wheel and is parallel to
the ground.
Let t be the time (in seconds) elapsed that you have been riding the Ferris
wheel, and y is he directed distance of your location with respect to the
assumed central position at time t. Because y = 10 when t = 0, the
appropriate model is y = 10 cos bt for t 0.
Given that the Ferris wheel takes 32 seconds to move from the lowest point
to the next, the period is 32.
2 t
b =32 =) b = 16 =) y = 10 cos 16
150
When t = 150, we get y = 10 cos 16 3:83.
Bringing back the original condition given in the problem that the riding point is 3 m o the
ground, after riding for 150 seconds, you are approximately located
3:83 + 13 = 16:83 m o the ground. 2

In the last example, the central position or equilibrium may be vertically shifted
from the ground or sea level (the role of the constant d). In the same way, the
starting point may also be horizontally shifted (the role of the constant c). Moreover,
as observed in Sub-Lesson 3.3.3 (see page 154), to nd the function that describes a
particular simple harmonic motion, we can either choose
y = a sin b(t c) + d
or
y = a cos b(t c) + d;
and determine the appropriate values of a, b, c, and d. In fact, we can assume that
a and b are positive numbers, and c is the smallest such nonnegative number.

163
All rights reserved. No part of this material may be reproduced or transmitted in any form or by any means -
electronic or mechanical including photocopying – without written permission from the DepEd Central Office. First Edition, 2016.
Example 3.3.12. A signal buoy in Laguna Bay bobs up and down with the
height h of its transmitter (in feet) above sea level modeled by h(t) = a sin bt
+ d at time t (in seconds). During a small squall, its height varies from 1 ft to
9 ft above sea level, and it takes 3:5 seconds from one 9-ft height to the next.
Find the values of the constants a, b, and d.

Solution. We solve the constants step by step.

The minimum and maximum values of h(t) are 1 ft and 9 ft, respectively.
Thus, the amplitude is a = 1 (M m) = 1 (9 1)=4.
2 2

Because it takes 3:5 seconds from one 9-ft height to the next, the
2 4
period is 3:5. Thus, we have b = 3:5, which gives b = 7 .
Because the lowest point is 1 ft above the sea level and the amplitude is 4,
it follows that d = 5. 2

Example 3.3.13. A variable star is a star whose brightness uctuates as ob-


served from Earth. The magnitude of visual brightness of one variable star
ranges from 2:0 to 10:1, and it takes 332 days to observe one maximum
brightness to the next. Assuming that the visual brightness of the star can be
modeled by the equation y = a sin b(t c) + d, t in days, and putting t = 0 at a
time when the star is at its maximum brightness, nd the constants a, b, c,
and d, where a; b > 0 and c the least nonnegative number possible.

Solution.
a = M m = 10:1 2:0 = 4:05
2 2
2
b = 332 =) b = 166
d = a + m = 4:05 + 2:0 = 6:05
For the (ordinary) sine function to start at the highest point at t = 0, the least
3
possible horizontal movement to the right (positive value) is 2 units.
3 3 3
bc = c= = = 249 2
2 =) 2b 2 166
?
Example 3.3.14. The path of a fast-moving particle traces a circle with
equa-tion
2
(x + 7) + (y 5)2 = 36:
It starts at point (1; 5), moves clockwise, and passes the point (7; 11) for the
rst time after traveling 6 microseconds. Where is the particle after traveling
15 microseconds?

164
All rights reserved. No part of this material may be reproduced or transmitted in any form or by any means -
electronic or mechanical including photocopying – without written permission from the DepEd Central Office. First Edition, 2016.
Solution. As described above, we may choose sine or cosine function. Here,
we choose the sine function to describe both x and y in terms of time t in
microsec-onds; that is, we let
x = a sin b(t c) + d and y = e sin f(t g) + h;

where we appropriately choose the positive values for a, b, e, and f, and the
least nonnegative values for c and g.
The given circle has radius 6 and center (7; 5). De ning the central
position of the values of x as the line x = 7 and that of the values of y as the
line y = 5, we get a = e = 6, d = 7, and h = 5.
From the point (1; 5) to the point (7; 11) (moving clockwise), the particle
has traveled three-fourths of the complete cycle; that is, three-fourths of the
period must be 2.
3 2 3 2
4 b =4 f =6 =) b = f = 4

As the particle starts at (1; 5) and moves clockwise, the values of x start
at its highest value (x = 1) and move downward toward its central position (x
= 7) and continue to its lowest value (x = 13). Therefore, the graph of a sin bt
3
+ d has to move 2b = 6 units to the right, and so we get c = 6.
As to the value of g, we observe the values of y start at its central position
(y = 5) and go downward to its lowest value (y = 1). Similar to the argument
used in determining c, the graph of y = e sin f t + h has to move b = 4 units to
the right, implying that g = 4.
Hence, We have the following equations of x and y in terms of t:
x = 6 sin (t 6) 7 and y = 6 sin (t 4)+5:
4 4

When t = 15, we get

x = 6 sin 4 (15 6) 7=7+3 p 2 2:76


and
p
y = 6 sin 4 (15 4)+5=5+3 2 9:24:
That is, after traveling for 15 microseconds, the particle is located near the point
(2:76; 9:24). 2

More Solved Examples


2
Solution: y = 4 sin 4 3 = y = 4 sin 3 (x ) 3 = P= 1 =6
1. Find the period of the function y = 4 sin 4 x
3.
x 1

) ) 3

165
All rights reserved. No part of this material may be reproduced or transmitted in any form or by any means -
electronic or mechanical including photocopying – without written permission from the DepEd Central Office. First Edition, 2016.
In the function y = 3 tan(2kx ), the period is 4 . Find the value of k and the
phase shift of the graph of the function.
Solution: The period of the tangent function is P = b .

y = 3 tan(2kx ) =) y = 3 tan 2k x 2k =) P = 2k =4:


1 =4= k= 1 and Phase shift = = =4
1

2k ) 8 1 1 2k 28
3. Sketch the graph of function y = 2 sin 2 x+ 6 +2 over one period. Determine
the domain and range of the function.
Solution: The graph is a vertical translation of y = 1 sin 1 x+ by 2 units
2 2 6
2
1
complete cycle may
upward. The period of the given function is =4 . One
2
start at x = and end at x = +4 = 23 .
6 6 6
The critical points for the graph are

5 11 17 23
x= ;x= ;x= ;x= ; and x = :
6 6 6 6 6

The domain of the function is R and its range is 5


2 ; 3
2 .

4. Sketch the graph of the function y = 2 cos( x 2 ) + 3 over two periods. Find
the domain and range of the function.

Solution: The graph of the given function is a vertical translation of y = 2


cos( x 2 ) by 3 units upward. The period of the function is 2 . One complete
5
cycle may start and end at x = 2 and x = 2 , respectively. The next
5 9
complete cycle starts at x = 2 and ends at x = 2 .
3 5 7 9
critical points: 2 ; ; 2 ; 2 ; 2 ; 3 ; 2 ; 4 ; 2

166
All rights reserved. No part of this material may be reproduced or transmitted in any form or by any means -
electronic or mechanical including photocopying – without written permission from the DepEd Central Office. First Edition, 2016.
The domain of the given cosine function is R, and its range is [1; 5].
1
Sketch the graph of the function y = x
5. the domain and range of the function. 4 tan 4 over three periods. Find
Solution: The period of the function is . One complete cycle may start at x
5
= 4 and end at x = 4 .

3
The domain of the function is fxjx 6= 4 + k ; k 2 Zg, and its range is R.
Sketch the graph of the function y = 3 cot 1 x+ + 2 over three periods.
6. Find the domain and range of the function. 2 12
1 1
Solution: y = 3 cot 2 x + 12 + 2 = 3 cot 2 x+ 6 +2 =) P=2
11

One complete cycle may start at x = 6 and end at x = 6 .

167
All rights reserved. No part of this material may be reproduced or transmitted in any form or by any means -
electronic or mechanical including photocopying – without written permission from the DepEd Central Office. First Edition, 2016.
The domain of the function is fxjx 6= 6 + 2k ; k 2 Zg, and its range is R.
7. The graph of the function g(x) is the same as that of f(x) = 3 sin x but
3
shifted 2 units downward and 2 units to the right. What is g( )?
Solution: The function f(x) = 3 sin x 3 when shifted 2 units downward
and 2 units to the right is
g(x) = 3 sin x 3 2 2 = 3 sin x 56 2:

5 1
g( ) = 3 sin 2=
6 2

8. The graph of the function h(x) is the same as that of f(x) = 3 sin(2x 3 )+1
but shifted 3 units upward and 2 units to the left. What is h( 5 )? 6

Solution: h(x) = 3 sin 2 x+ 2 3 + 1 + 3 = 3 sin(2x 2 )+4

5 5 8 3p 3

h 6 = 3 sin 2 6 2 +4= 2
9. Sketch the graph of y = 2 sec 1 x over two periods. Find the domain

and range of the function. 2 4


Solution: The period of the function is 4 . One complete cycle may start at
17
x = 4 and end at x = 4 .

168
All rights reserved. No part of this material may be reproduced or transmitted in any form or by any means -
electronic or mechanical including photocopying – without written permission from the DepEd Central Office. First Edition, 2016.
5
The domain of the function is fxjx 6= 4 + 2k ; k 2 Zg, and its range is
(; 2] [ [2; 1).
Sketch the graph of y = csc x + + 2 over two periods. Find the domain

10. and range of the function. 3


Solution: The period of the function is 2 . One complete cycle may start and
end at x = 3 and x = 53 , respectively.

The domain of the function is fxjx 6= 3 + k ; k 2 Zg, and its range is (; 1]


[ [3; 1).

169
All rights reserved. No part of this material may be reproduced or transmitted in any form or by any means -
electronic or mechanical including photocopying – without written permission from the DepEd Central Office. First Edition, 2016.
Supplementary Problems 3.3
1
1. What is the period of the function y = 2 cos 4 x 2 ?

a bx
2. The amplitude and period of the function y = 4 2
cos 3
are 3 and 4 ,

respectively. Find jaj + b.


3. In the function y = 2 3 cot 4 (x 2), the period is 2. Find the value of k.
k
4. 5? 4 3
What are the minimum and maximum values of the function y = 3 sin 3
x+ 2

5. Given the function y = 3 sin 3 x + 2 5, nd the value of y when x = 8.


4 3 9
6. Given the function y = 2 cot 4 x + 3, nd the value of y when x = 7.
3 6 1 6
function y = 2 x 3
7. Find the domain and range of the sin + 2?

2x 3 3 4
8. Find the range of the function y = 3 sec 3 .
Find the equation of the secant function whose graph is the graph of y = 3
sec 2x shifted units to the right and 3 units downward.
Find the equation of the sine function whose graph is the graph of y =
2 sin 2 x 4 + 1 shifted 2 units to the left and 3 units upward.
2x
11. Given the tangent function y = 1 3 tan , nd the equations of all its
4
vertical asymptotes. x
, nd the equations of all its
Given the cosecant function y = csc 3
vertical asymptotes.
Sketch the graph over one period, and indicate the period, phase shift,
domain, and range for each.
1 1 3
(a) y = 2 sin x+ 1 (c) y = csc (2x ) 1
4 4 2 4
1 1 2
(b) y = tan 2 2x + 3 2 (d) y = sec 2 4x + 3 +2

1
0 A point P in simple harmonic motion has a frequency of 2 oscillation per
minute and amplitude of 4 ft. Express the motion of P by means of an
equation in the form d = a sin bt.
15. A mass is attached to a spring, and then pulled and released 8 cm below its resting position
at the start. If the simple harmonic motion is modeled by y = a cos 101 (t c), where a > 0, c
the least nonnegative such number, and t in seconds, nd the location of the mass 10
seconds later.

170
All rights reserved. No part of this material may be reproduced or transmitted in any form or by any means -
electronic or mechanical including photocopying – without written permission from the DepEd Central Office. First Edition, 2016.
Lesson 3.4. Fundamental Trigonometric Identities

Learning Outcomes of the Lesson


At the end of the lesson, the student is able to:
determine whether an equation is an identity or a conditional equation;
derive the fundamental trigonometric identities;
simplify trigonometric expressions using fundamental trigonometric identi-
ties; and
prove other trigonometric identities using fundamental trigonometric identi-
ties.

Lesson Outline
Domain of an equation
Identity and conditional equation
Fundamental trigonometric identities
Proving trigonometric identities

Introduction
In previous lessons, we have de ned trigonometric functions using the
unit circle and also investigated the graphs of the six trigonometric functions.
This lesson builds on the understanding of the di erent trigonometric
functions by discovery, deriving, and working with trigonometric identities.

3.4.1. Domain of an Expression or Equation

Consider the following expressions:


p 2 x x

2x + 1; x 1; x2 3x 4 ; p x 1 :
What are the real values of the variable x that make the expressions de ned
in the set of real numbers?
In the rst expression, every real value of x when substituted to the
expression makes it de ned in the set of real numbers; that is, the value of
the expression is real when x is real.
In the second expression, not every real value of x makes the expression de ned p
in R. For example, when x = 0, the expression becomes 1, which is not a
real number.
p
2 2
x 12R () x 1 0 () x 1 or x 1

171
All rights reserved. No part of this material may be reproduced or transmitted in any form or by any means -
electronic or mechanical including photocopying – without written permission from the DepEd Central Office. First Edition, 2016.
p
2
Here, for x 1 to be de ned in R, x must be in (; 1] [ [1; 1).
In the third expression, the values of x that make the denominator zero
make the entire expression unde ned.

2
x 3x 4 = (x
4)(x + 1) = 0 () x = 4 or x = 1
x
Hence, the expression x2 3x 4 is real when x 6= 4 andx 6=1.
p
In the fourth expression, because the expression x 1 is in the
denominator, x must be greater than 1. Although the value of the entire
expression is 0 when x = 0, we do not include 0 as allowed value of x
because part of the expression is not real when x = 0.
In the expressions above, the allowed values of the variable x constitute
the domain of the expression.

The domain of an expression (or equation) is the set of all real


values of the variable for which every term (or part) of the
expression (equation) is de ned in R.

In the expressions above, the domains of the rst, second, third, and fourth
expressions are R, (; 1] [ [1; 1), R n f1; 4g, and (1; 1), respectively.
Example 3.4.1. Determine the domain of the expression/equation.

2
x 1 px + 1
3 2
(a) x + 2x 8x 1 x
(b) tan sin cos 2

2 p 2 2

(c) x 1 + x =p3 x2 1
2
cos z
(d) z = 4 sin z 1
1 + sin z
3 2
Solution. (a) x + 2x 8x = x(x + 4)(x 2) = 0 () x = 0; x =
4; or x = 2
p

x + 1 2 R () x + 1 0 () x1
1 x=0 () x = 1
Domain = [1; 1) n f4; 0; 1; 2g
[1; 0) [ (0; 1) [ (1; 2) [ (2; 1)

172
All rights reserved. No part of this material may be reproduced or transmitted in any form or by any means -
electronic or mechanical including photocopying – without written permission from the DepEd Central Office.
First Edition, 2016.
sin
(b) tan sin cos 2 = cos sin cos 2
k
cos = 0 ()= k odd integer 2 ;
k
Domain = R n f 2 j k odd integerg
2 p
(c) The expression 1+x is always positive, and so 1 + x2 is de ned in R. On
p3 2
the other hand, the expression x 1 is also de ned in R, but it cannot
be zero because it is in the denominator. Therefore, x should not be 1
and 1.

Domain = R n f1; 1g
3
(d) 1 + sin z = 0 () z= 2 + 2k ; k 2 Z
3
Domain = R n f 2 + 2k jk 2 Zg 2

3.4.2. Identity and Conditional Equation

Consider the following two groups of equations:

Group A Group B
2 2
(A1) x 1=0 (B1) x 1 = (x 1)(x + 1)
2 2 2 2
(A2) (x + 7) = x + 49 (B2) (x + 7) = x + 14x + 49
x2 4 x2 4
(A3) = 2x 1 (B3) =x+2
x 2 x 2

In each equation in Group A, some values of the variable that are in the
domain of the equation do not satisfy the equation (that is, do not make the
equation true). On the other hand, in each equation in Group B, every
element in the domain of the equation satis es the given equation. The
equations in Group A are called conditional equations, while those in Group
B are called identities.

An identity is an equation that is true for all values of the variable


in the domain of the equation. An equation that is not an identity is
called a conditional equation. (In other words, if some values of the
variable in the domain of the equation do not satisfy the equation,
then the equation is a conditional equation.)

Example 3.4.2. Identify whether the given equation is an identity or a condi-


tional equation. For each conditional equation, provide a value of the variable
in the domain that does not satisfy the equation.

3
p3 p
(1) x 2= x p3 2 x2 + 2x + 3 4
173
All rights reserved. No part of this material may be reproduced or transmitted in any form or by any means -
electronic or mechanical including photocopying – without written permission from the DepEd Central Office.
First Edition, 2016.
2 2
(2) sin = cos + 1
(3) sin = cos 1
1 px 1 2p x + x
(4) 1 + p x = 1 x

Solution. (1) This is an identity because this is simply factoring of di erence of


two cubes.
This is a conditional equation. If = 0, then the left-hand side of the equation is
0, while the right-hand side is 2.
This is also a conditional equation. If = 0, then both sides of the equation are
equal to 0. But if = , then the left-hand side of the equation is 0, while the
right-hand side is 2.
This is an identity because the right-hand side of the equation is obtained by
rationalizing the denominator of the left-hand side. 2

3.4.3. The Fundamental Trigonometric Identities

Recall that if P (x; y) is the terminal point on the unit circle corresponding to ,
then we have
sin = y csc = 1 tan = y
y x
cos = x sec = 1 cot = x :
x y

From the de nitions, the following reciprocal and quotient identities


immedi-ately follow. Note that these identities hold if is taken either as a real
number or as an angle.

Reciprocal Identities

csc = 1 sec = 1 cot = 1


sin cos tan

Quotient Identities
sin cos
tan = cot =
cos sin

We can use these identities to simplify trigonometric expressions.

174
All rights reserved. No part of this material may be reproduced or transmitted in any form or by any means -
electronic or mechanical including photocopying – without written permission from the DepEd Central Office. First Edition, 2016.
Example 3.4.3. Simplify:
tan cos cos
(1) (2)
sin cot
tan cos sin cos
Solution. (1) = cos =1
sin sin
cos cos
(2) cot = cos = sin 2
sin

2
If P (x; y) is the terminal point on the unit circle corresponding to , then x
2
+ y = 1. Since sin = y and cos = x, we get
2 2
sin + cos = 1:
2 2
By dividing both sides of this identity by cos and sin , respectively, we obtain
2 2 2 2
tan + 1 = sec and 1 + cot = csc :

Pythagorean Identities
2 2
sin + cos =1
2 2 2 2
tan + 1 = sec 1 + cot = csc

Example 3.4.4. Simplify:


2
1 + tan
2 2 2 2
(1) cos + cos tan (2) 1 + cot
2 2 2 2 2
Solution. (1) cos + cos tan = (cos )(1 + tan )
2 2
cos sec
1
2 2 1 2
1 + tan sec cos
2 sin 2
(2) 2 = 2 = 1 = 2 = tan 2
1 + cot csc sin
2
cos
In addition to the eight identities presented above, we also have the
following identities.

Even-Odd Identities

sin( )= sin cos( ) = cos


tan( )= tan

175
All rights reserved. No part of this material may be reproduced or transmitted in any form or by any means -
electronic or mechanical including photocopying – without written permission from the DepEd Central Office. First Edition, 2016.
The rst two of the negative identities can be obtained from the graphs of
the sine and cosine functions, respectively. (Please review the discussion on
page 147.) The third identity can be derived as follows:

tan( ) = sin( ) = sin = tan :


cos( ) cos

The reciprocal, quotient, Pythagorean, and even-odd identities constitute


what we call the fundamental trigonometric identities.
We now solve Example 3.2.3 in a di erent way.
3
Example 3.4.5. If sin = 4
and cos > 0. Find cos .

2 2
Solution. Using the identity sin + cos = 1 with cos > 0, we have
p s :
cos = 1 sin2 = 1 4 =
4 2
2
3 p7

5
Example 3.4.6. If sec = 2 and tan < 0, use the identities to nd the values of
the remaining trigonometric functions of .

Solution. Note that lies in QIV.

1 2
cos = =
sec 5
sin = p 1 cos2 = s 1 5 =
5
2
2 p 21

p
1 5 21
csc = =
sin 21
p 21 p
sin 5 21
tan = = =
cos 2

5
2

1 2 p 21
cot = tan = 21 2

3.4.4. Proving Trigonometric Identities

We can use the eleven fundamental trigonometric identities to establish other


identities. For example, suppose we want to establish the identity

csc cot = sin :


1 + cos

176
All rights reserved. No part of this material may be reproduced or transmitted in any form or by any means -
electronic or mechanical including photocopying – without written permission from the DepEd Central Office. First Edition, 2016.
To verify that it is an identity, recall that we need to establish the truth of the
equation for all values of the variable in the domain of the equation. It is not
enough to verify its truth for some selected values of the variable. To prove it,
we use the fundamental trigonometric identities and valid algebraic
manipulations like performing the fundamental operations, factoring, canceling,
and multiplying the numerator and denominator by the same quantity.
Start on the expression on one side of the proposed identity (preferably
the complicated side), use and apply some of the fundamental trigonometric
identities and algebraic manipulations, and arrive at the expression on the
other side of the proposed identity.

Expression Explanation
csc cot Start on one side.
= 1 cos
Apply some reciprocal
sin sin
and quotient identities.
1cos
= Add the quotients.
sin
1 cos 1 + cos
Multiply the numerator
= sin 1 + cos and denominator by
+ cos .
2
0 cos Multiply.
(sin )(1 + cos )
2
sin
(sin )(1 + cos ) Apply a Pythagorean
sin identity.
=
1 + cos Reduce to lowest terms.

Upon arriving at the expression of the other side, the identity has been
estab-lished. There is no unique technique to prove all identities, but
familiarity with the di erent techniques may help.

Example 3.4.7. Prove: sec x cos x = sin x tan x.

Solution.
1
sec x cos x = cos x cos x
1 cos2 x sin2 x sin x
= cos x = cos x = sin x cos x = sin x tan x 2
1 + sin 1 sin 2
Example 3.4.8. Prove: = 4 sin sec
1 sin 1 + sin

177
All rights reserved. No part of this material may be reproduced or transmitted in any form or by any means -
electronic or mechanical including photocopying – without written permission from the DepEd Central Office. First Edition, 2016.
Solution.
2 2
1 + sin 1 sin (1 + sin ) (1 sin )
=
1 sin 1 + sin (1 sin )(1 + sin )
2 2
1 + 2 sin + sin 1 + 2 sin sin
= 2
1 sin
4 sin
2
= cos
2
= 4 sin sec 2

More Solved Examples


1. Express each of the other circular functions of in terms of cos .
Solution:
1
sec = cos

2 2 2 2 p 2
sin + cos = 1 =) sin = 1 cos =) sin = 1 cos
1 = 1
csc = sin p 2
1 cos
cos p cos
cot = sin = 1 cos2
p 2
sin 1 cos
tan = =
cos cos
2
If tan = a, express cos in terms of a.
Solution:
2 2
sin sin 1 cos
2 2 2 2
a = cos =) a =cos =) a = cos
2 2 2
a cos = 1 cos
1
2 2 2 2 2 2 2
a cos + cos = 1 =) cos (a + 1) = 1 =) cos = a +1
4 4
3. Given a = cos x, simplify and express sin x cos x in terms of a.
4 4 2 2 2 2
Solution: sin x cos x = (sin x + cos x)(sin x cos x)
2 2
= sin x cos x
2 2
= 1 cos x cos x
2 2
= 1 2 cos x = 1 2a

178
All rights reserved. No part of this material may be reproduced or transmitted in any form or by any means -
electronic or mechanical including photocopying – without written permission from the DepEd Central Office. First Edition, 2016.
2 2
Simplify (csc x sec x) + (csc x + sec x) .
2 2
Solution: (csc x sec x) + (csc x + sec x)
0 (csc2 x 2 csc x sec x + sec2 x) + (csc2 x + 2 csc x sec x + sec2 x)
1 2 csc2 x + 2 sec2 x
2 2
2 2
= sin x + cos x
2 2 2 2
= 2(cos x + sin x) = 2 = 2 csc x sec x
2 2 2 2
sin x cos x sin x cos x
csc
5. Verify the identity tan + cot = cos .
Solution:
1 1
csc sin sin 1 cos sin
2 2
tan + cot = sin +cos = sin + cos = sin 1 = cos
cos sin cos sin

6. Establish the identity csc + cot 1 = 1 + cos .


cot csc + 1 sin
csc + cot 1
Solution:
cot csc + 1
csc + cot 1 csc + cot
= cot csc + 1 csc + cot
(csc + cot 1)(csc + cot )
= (cot csc )(csc + cot ) + (csc + cot )
(csc + cot 1)(csc + cot )
2 2
= cot csc + csc + cot
= (csc + cot 1)(csc + cot )
1 + csc + cot
= csc + cot = 1 +cos = 1 + cos
sin sin sin

Supplementary Problems 3.4


1. Using fundamental identities, simplify the expression tan x sin x .
sin x
1
2. Using fundamental identities, simplify the expression csc x cot x .
2
cos A
3. Simplify sin A + .
1 + sin A

179
All rights reserved. No part of this material may be reproduced or transmitted in any form or by any means -
electronic or mechanical including photocopying – without written permission from the DepEd Central Office. First Edition, 2016.
2 2
4. Simplify (1 cos A)(1 + cot A).
csc x + sec x

5. Express in terms of sine and cosine.


tan x cot x

6. Express in terms of sine and cosine.


tan x + sin x
7. Express in terms cosine only.

1
8. Express 1 + tan2 x in terms sine only.
9. If cot = a, express sin cos in terms of a.
10. If sec = a > 0 and sin > 0, express sin cos in terms of a.

For numbers 11 - 20, establish the identities.

csc a + 1 1 + sin a
11. =
csc a 1 1 sin a
12. 1 + sin a 1 sin a = 4 tan a sec a
sin a 1 + sin a
cos a
13. sec a + tan a = 1 sin a
csc a + sec a tan a = tan a sec a
2
csc a
11
2
1 cos a + 1 + cos a = 2 csc a
sin3cos3
= 1 + sin cos sin
cos
tan sin cos
2 2
17. 1 tan = 2 cos 1
tan2 + sec + 1 = tan + sin tan + cot
cotsin sec 2 2
19. = cos sin
sec csc
2 2 2 4
20. tan sec sec + 1 = tan

180
All rights reserved. No part of this material may be reproduced or transmitted in any form or by any means -
electronic or mechanical including photocopying – without written permission from the DepEd Central Office. First Edition, 2016.
Lesson 3.5. Sum and Di erence Identities

Learning Outcomes of the Lesson


At the end of the lesson, the student is able to:
derive trigonometric identities involving sum and di erence of two angles;
simplify trigonometric expressions using fundamental trigonometric identities
and sum and di erence identities;
prove other trigonometric identities using fundamental trigonometric identi-
ties and sum and di erence identities; and
solve situational problems involving trigonometric identities.

Lesson Outline
The sum and di erence identities for cosine, sine, and tangent functions
Cofunction identities
More trigonometric identities

Introduction
In previous lesson, we introduced the concept of trigonometric identity, pre-
sented the fundamental identities, and proved some identities. In this lesson, we
derive the sum and di erence identities for cosine, sine, and tangent functions,
establish the cofunction identities, and prove more trigonometric identities.

3.5.1. The Cosine Di erence and Sum Identities

Let u and v be any real numbers with 0 < v u < 2 . Consider the unit circle
with points A = (1; 0), P 1, P2, P3, and u and v with corresponding angles as
shown below. Then P1P2 = AP3.

181
All rights reserved. No part of this material may be reproduced or transmitted in any form or by any means -
electronic or mechanical including photocopying – without written permission from the DepEd Central Office. First Edition, 2016.
Recall that P1 = P (u) = (cos u; sin u), P2 = P (v) = (cos v; sin v), and P3 =
P (u v) = (cos(u v); sin(u v)), so that

p 2 2
P1P2 = (cos u cos v) + (sin u sin v) ;
while
2 2
AP3 = [cos(u v) 1] + [sin(u v) 0] :
expressions and expanding the squares, we get

Equating these two p


2 2 2 2
(cos u cos v) + (sin u sin v) = [cos(u v) 1] + sin (u v)

2 2 2 2
cos u 2 cos u cos v + cos v + sin u 2 sin u sin v + sin v
2 2
= cos (u v) 2 cos(u v) + 1 + sin (u v)
2 2
Applying the Pythagorean identity cos +sin = 1 and simplifying the resulting
equations, we obtain
2 2 2 2
(cos u + sin u) + (cos v + sin v ) 2 cos u cos v 2 sin u sin v
2 2
= [cos (u v) + sin (u v )] 2 cos(u v) + 1

1 + 1 2 cos u cos v 2 sin u sin v = 1 2 cos(u v) + 1


cos(u v) = cos u cos v + sin u sin v:
We have thus proved another identity.
Although we assumed at the start that 0 < v u < 2 , but because cos( ) =
cos (one of the even-odd identities), this new identity is true for any real
numbers u and v. As before, the variables can take any real values or angle
measures.

Cosine Di erence Identity

cos(A B) = cos A cos B + sin A sin B

Replacing B with B , and applying the even-odd identities, we immediately


get another identity.

Cosine Sum Identity

cos(A + B) = cos A cos B sin A sin B

Example 3.5.1. Find the exact values of cos 105 and cos 12 .

182
All rights reserved. No part of this material may be reproduced or transmitted in any form or by any means -
electronic or mechanical including photocopying – without written permission from the DepEd Central Office. First Edition, 2016.
Solution.

cos 105 = cos(60 + 45 )


= cos 60 cos 45 sin 60 sin 45
1 p2 p3 p2
= 2 2 2 2
p2 p6
=
4

cos = cos
12 4 6
= cos cos + sin sin
4
p p 6 p 4 6
2 3 21
2 2 +2 2pp
6+ 2
= 2
4
3 12
Example 3.5.2. Given cos = 5 and sin = 13 , where lies in QIV and in QI, nd
cos( + ).

Solution. We will be needing sin and cos .


2
sin = p 1 cos = s 1 5 = 5
2
3 4
q 2
s
cos = 1 sin = 1 13 =13
12 2 5

cos( + ) = cos cos sin sin


=5 13 5 13
3 5 4 12

= 63 2
65

3.5.2. The Cofunction Identities and the Sine Sum and Di erence Identities

In the Cosine Di erence Identity, if we let A = 2 , we get

cos 2 B = cos 2 cos B + sin 2 sin B


183
All rights reserved. No part of this material may be reproduced or transmitted in any form or by any means -
electronic or mechanical including photocopying – without written permission from the DepEd Central Office. First Edition, 2016.
(0) cos B + (1) sin B
sin B:
From this identity, if we replace B with B, we have
2

h
cos 2 2 B i = sin 2 B

cos B = sin 2 B :
As for the tangent function, we have
sin B
2
tan 2 B = cos 2
B
= cos B
sin B
= cot B:
We have just derived another set of identities.

Cofunction Identities

cos 2 B = sin B sin 2 B = cos B

tan 2 B = cot B
Using the rst two cofunction identities, we now derive the identity for sin(A+
B).

h i
sin(A + B) = cos 2 (A + B)
h i
= cos 2 A B)
= cos 2 A cos B + sin 2 Asin B
= sin A cos B + cos A sin B

Sine Sum Identity

sin(A + B) = sin A cos B + cos A sin B

In the last identity, replacing B with B and applying the even-odd identities
yield
sin(A B) = sin[A + (B )]

184
All rights reserved. No part of this material may be reproduced or transmitted in any form or by any means -
electronic or mechanical including photocopying – without written permission from the DepEd Central Office. First Edition, 2016.
6+ 2
4
1
and sin = 2 , where 0 < < 2
= sin A cos(B ) + cos A sin(B )
= sin A cos B cos A sin B:

Sine Di erence Identity

sin(A B) = sin A cos B cos A sin B

Example 3.5.3. Find the exact value of sin 12 .


Solution.
5 sin 4 + 6
sin
12
sincos+ cossin
4p p646p
2 3 21
2 2 +2 2pp
= 2

Example 3.5.4. If sin = 133 and 2 < < ,


nd sin( + ) and sin( ).

Solution. We rst compute cos and cos .


s
p
2
3 2 4p 10
cos = 1 sin = 1
13 = 13
cos = q 1 sin2 = s 1 2 = 2
1 3

sin( + ) = sin cos + cos sin


=
13 p + p 2
2! 13

3 3 4 10 1
p p
410 33
26

sin( ) = sin cos cos sin


p!
1 4 10 3 3
=2 13 2 13
p p
4 10+3 3
= 2
26

185
All rights reserved. No part of this material may be reproduced or transmitted in any form or by any means -
electronic or mechanical including photocopying – without written permission from the DepEd Central Office. First Edition, 2016.
Example 3.5.5. Prove:

sin(x + y) = (1 + cot x tan y) sin x cos y:

Solution.

(1 + cot x tan y) sin x cos y = sin x cos y + cot x tan y sin x cos y
= sin x cos y + cos x sin y sin x cos y
sin x cos y
= sin x cos y + cos x sin y = sin(x + y) 2

3.5.3. The Tangent Sum and Di erence Identities

Recall that tan x is the ratio of sin x over cos x. When we replace x with A +
B, we obtain sin(A + B)

tan(A + B) = :

Using the sum identities for sine and cosine, and then dividing the numerator
and denominator by cos A cos B, we have

tan(A + B) = sin A cos B + cos A sin B


cos A cos B sin A sin B
sin A cos B cos A sin B
A cos B +cos A cos B
= cos
cos A cos B sin A sin B
cos A cos B cos A cos B

tan A + tan B : 1
tan A tan B
We have just established the tangent sum identity.
In the above identity, if we replace B with B and use the even-odd identity
tan( ) = tan , we get

tan(A B) = tan[A + (B )] = tan A + tan(B ) = tan A tan B :


1 tan A tan(B ) 1 + tan A tan B
This is the tangent di erence identity.

Tangent Sum and Di erence Identities


tan A + tan B
tan(A + B) =
1 tan A tan B
tan A tan B
tan(A B) =
1 + tan A tan B

186
All rights reserved. No part of this material may be reproduced or transmitted in any form or by any means -
electronic or mechanical including photocopying – without written permission from the DepEd Central Office. First Edition, 2016.
More Solved Examples
1. Given cos A = 3 , <A< 3, and tan B = 7 , B in QI, nd: (a) sin(A+B),
5 2 24

(b) cos(A + B), and (c) tan(A + B).


3 4
Solution: cos A = 5 and A in QIII =) sin A = 5
7 7 24
tan B = 24 and B in QIII =) sin B = 25 and cos B = 25
(a) sin(A + B) = sin A cos B + cos A sin B
4 24 3 7 117

= 5
25 + 5 25 =125
(b) cos(A + B) = cos A cos B sin A sin B
3 24 4 7 44

= 5 25 5 25 = 125
sin(A + B) 117
117

44
(c) tan(A + B) = = 125 = cos(A + B)
44
125

5 Solution: c
2. Find the exact value of cos 12 .
!
2
2
3. If A + B = 2 + 2k , k 2 Z, prove that sin A = cos B.

Solution: sin A = sin 2 + 2k B


0 sin 2 + 2k cos B cos 2 + 2k sin B = cos B

Find the value of


(tan 10 )(tan 15 )(tan 20 )(tan 15 ) (tan 65 )(tan 70 )(tan 75 )(tan 80 ):

Solution: From the previous item, we know that sin = cos(90 ). We


write each tangent in terms of sine and cosine.
(tan 10 )(tan 15 )(tan 20 )(tan 15 ) (tan 65 )(tan 70 )(tan 75 )(tan 80 )
= cos 10 cos 15 cos 20 cos 70 cos 75 cos 80
sin 10 sin 15 sin 20 sin 70 sin 75 sin 80

=1

187
All rights reserved. No part of this material may be reproduced or transmitted in any form or by any means -
electronic or mechanical including photocopying – without written permission from the DepEd Central Office. First Edition, 2016.
5. If A, B and C are the angles of a triangle and
p
(tan A)(tan B)(tan C) = 3 ;
3
nd tan A + tan B + tan C.
Solution:

A+B+C =180 =) tan(A + B + C) = tan 180 = 0


tan A + tan(B + C) =0 =) tan A + tan(B + C) = 0
tan A tan(B + C)
tan A + tan B + tan C = 0
1 tan B tan C
tan A tan A tan B tan C + tan B + tan C = 0
tan B tan C
=) tan A tan A tan B tan C + tan B + tan C = 0

tan A + tan B + tan C = tan A tan B tan C = p3


3

6. Establish the identity sin(A + B) = tan A + tan B .


cos(A B ) 1 + tan A tan B
Solution:
sin(A + B) =sin A cos B + cos A sin B
cos(A B) cos A cos B + sin A sin B
1
sin A cos B + cos A sin B cos A cos B
=cos A cos B + sin A sin B 1
cos A cos B
sin A cos B cos A sin B
+
= cos A cos B cos A cos B
cos A cos B sin A sin B
+
cos A cos B cos A cos B
tan A + tan B
=
1 + tan A tan B

Supplementary Problems 3.5


3 2
If 2 < < 2 , nd the radian measure of if cos = sin 3 .
4
For what angle in QIV is sin = cos 3 ?

If A + B = 2 + k , k 2 Z, prove that tan A = cot B.

188
All rights reserved. No part of this material may be reproduced or transmitted in any form or by any means -
electronic or mechanical including photocopying – without written permission from the DepEd Central Office.
First Edition, 2016.
5
4. What is the exact value of cot 12 ?

5. What is the exact value of sin 105 cos 15 ?


6. What is the exact value of tan 1875 ?
p
7. Let and be acute angles such that cot = 7 and csc = 10. Find
cos( + ).
8. Given sin = 8 and sin = 1 , nd cos( + ) if both and are in QIV.
17 2

9. If 3 sin x = 2, nd sin(x ) + sin(x + ).


10. Simplify: cos x + 2 + cos 2
.
4 4
11. Given sin A = 5 , 2 A , and cos B = 5 , B not in QI, nd: (a) sin(A B),
(b) cos(A B), and (c) tan(A B). Also, determine the quadrant in which
B terminate.
p p
12. Given csc A = 3, A in QI, and sec B = 2, sin B < 0, nd: (a) sin(A B), (b)
cos(A B), and (c) tan(A B ). Also, determine the quadrant in which
B terminate.
13. Given sin = 4 and cos = 5 , nd sin( + ) + sin().
5 13
14. Given sin = 2 , in QII, and cos = 3 , nd cos( + ) + cos().
3 4
p
15. If A and B are acute angles (in degrees) such that csc A = 17 and csc B =
p 34
3 , what is A + B?
16. If tan(x + y) = 1 and tan y = 1 , what is tan x?
3 2

tan + tan 23
17. Evaluate: 1 tan9 36
.
tan 23
9 36

18. Establish the identity:

sin(A + B + C) = sin A cos B cos C + cos A sin B cos C


+ cos A cos B sin C sin A sin B sin C:

19. Prove: sin 2 = 2 sin cos .


2
cot 1
20. Prove: cot 2 = .
2 cot

189
All rights reserved. No part of this material may be reproduced or transmitted in any form or by any means -
electronic or mechanical including photocopying – without written permission from the DepEd Central Office. First Edition, 2016.
Topic Test 1 for Unit 3
A central angle in a circle of radius 6 cm measures 37:5 . Find: (a) length of
the intercepted arc and (b) the area of the sector.

2. The point (1; 2) lies on the terminal side of the angle in standard position.
Find sin + cos + tan .
3. Given sin A = 12 , where A is not in QI, and csc B = 5 , where B is not in
13 3
QIII, nd: (a) cos(A B) and (b) tan(A B).
tan 57 + tan 78
Find the exact value of . 1 tan
57 tan 78
cos x tan x + sin x
If sin x = a and cos x 0, express in terms of a. tan x
6 6 4 2
6. Prove the identity cos x + sin x = 3 cos x 3 cos x 1.
A regular hexagon of side length 1 unit is inscribed in a unit circle such that
two of its vertices are located on the x-axis. Determine the coordinates of
the hexagon.

Determine the amplitude, period and phase shift of the graph of


x

y = 2 sin 2 + 3 1;
and sketch its graph over one period. Find the range of the function.

190
All rights reserved. No part of this material may be reproduced or transmitted in any form or by any means -
electronic or mechanical including photocopying – without written permission from the DepEd Central Office. First Edition, 2016.
Topic Test 2 for Unit 3
2
The area of a sector of a circle formed by a central angle of 30 is d 3 cm .
Find the length of the intercepted arc.

2. The point (8; 6) lies on the terminal side of the angle in standard position.
2
Find (sin + cos ) .
8

3. Given sin A = 17 and cos A > 0, evaluate sin 2 A + cos 2 A.


4. Find the exact value of sin 160 cos 35 sin 70 cos 55 .
5. Find the exact value of tan 7 .
12
6. Given cos A = 3 , where A is not in QII, and tan B = 24 , where B is not in
5 7

QI, nd: (a) sin(A + B) and (b) cot(A + B).


2
tan x
3
7. Establish the identity tan x + tan x = sin x cos x.
8. If sin x cos x = 1, nd sin x .
3 sec x
Determine the period and phase shift of the graph of y = tan x + 2,

9. and sketch its graph over two periods. 18 3

191
All rights reserved. No part of this material may be reproduced or transmitted in any form or by any means -
electronic or mechanical including photocopying – without written permission from the DepEd Central Office. First Edition, 2016.
Lesson 3.6. Double-Angle and Half-Angle Identities

Learning Outcomes of the Lesson


At the end of the lesson, the student is able to:
derive the double-angle and half-angle identities;
simplify trigonometric expressions using known identities;
prove other trigonometric identities using known identities; and
solve situational problems involving trigonometric identities.

The double-angle and half-angle identities for cosine, sine, and tangent
More trigonometric identities

Introduction
Trigonometric identities simplify the computations of trigonometric expres-
sions. In this lesson, we continue on establishing more trigonometric
1
identities. In particular, we derive the formulas for f (2 ) and f 2 , where f is
the sine, cosine, or tangent function.

3.6.1. Double-Angle Identities

Recall the sum identities for sine and cosine.


sin(A + B) = sin A cos B + cos A sin B
cos(A + B) = cos A cos B sin A sin B

When A = B, these identities becomes


sin 2A = sin A cos A + cos A sin A = 2 sin A cos A
and
cos 2A = cos A cos A sin A sin A = cos2 A sin2 A:

Double-Angle Identities for Sine and Cosine sin

2A = 2 sin A cos A cos 2A = cos2 A sin2 A

The double-identity for cosine has other forms. We use the Pythagorean
2 2
identity sin + cos = 1.
2
cos 2A = cos A sin2 A

192
All rights reserved. No part of this material may be reproduced or transmitted in any form or by any means -
electronic or mechanical including photocopying – without written permission from the DepEd Central Office.
First Edition, 2016.
2 2
= cos A (1 cos A)
2
= 2 cos A 1

2 2
cos 2A = cos A sin A
2 2
= (1 sin A) sin A
2
=1 2 sin A

Other Double-Angle Identities for Cosine


2 2
cos 2A = 2 cos A 1 cos 2A = 1 2 sin A

3
Example 3.6.1. Given sin t = 5 and 2 < t < , nd sin 2t and cos 2t.

Solution. We rst nd cos t using the Pythagorean identity. Since t lies in QII, we have
cos t = p 1 sin2 t = s 1 5 = 5
:

2
3 4

2
sin 2t = 2 sin t cos t cos 2t = 1 2 sin t
3 4 3 2

=2 5 5 =1 2 5
24 7
= = 2
25 25

In the last example, we may compute cos 2t using one of the other two
double-angle identities for cosine. For the sake of answering the curious
minds, we include the computations here.

2 2 2
cos 2t = cos t sin t cos 2t = 2 cos t 1
4 2 3 2 4 2

= 5 5 =2 5 1
= 7 = 7
25 25

In the three cosine double-angle identities, which formula to use depends


on the convenience, what is given, and what is asked.

Example 3.6.2. Derive an identity for sin 3x in terms of sin x.

193
All rights reserved. No part of this material may be reproduced or transmitted in any form or by any means -
electronic or mechanical including photocopying – without written permission from the DepEd Central Office. First Edition, 2016.
Solution. We use the sum identity for sine, the double-angle identities for
sine and cosine, and the Pythagorean identity.
sin 3x = sin(2x + x)
= sin 2x cos x + cos 2x sin x
= (2 sin x cos x) cos x + (1 2 sin2 x) sin x
2
= 2 sin x cos x + sin x 2 sin3 x
2
= 2(sin x)(1 sin x) + sin x 2 sin3 x
3
= 3 sin x 4 sin x 2

For the double-angle formula for tangent, we recall the tangent sum identity:
tan A + tan B
tan(A + B) = :
tan A tan B
When A = B, we obtain
tan A + tan A 2 tan A
tan(A + A) = = 2 :
1 tan A tan A 1 tan A

Tangent Double-Angle Identity


2 tan A
2
tan 2A = 1 tan A
1
Example 3.6.3. If tan = 3
and sec > 0, nd sin 2 , cos 2 , and tan 2 .

Solution. We can compute immediately tan 2 .


tan 2 = 2
1 tan2 11
tan 2 3
= 1 3 2 = 4

3
From the given information, we deduce that lies in QIV. Using one Pythagorean identity, we
compute cos through sec . (We may also use the technique dis-cussed in Lesson 3.2 by solving for x, y,
and r.) Then we proceed to nd cos 2 .
p s
sec = 1 + tan2 = 1+ 3
= 3
2
1 p 10

p
1 1 3 10
cos = sec
= p 10 = 10
3

3p 10 2 4
2
cos 2 = 2 cos 1=2 10 ! 1=5
sin 2 3
tan 2 = cos 2 =) sin 2 = tan 2 cos 2 = 5 2
194
All rights reserved. No part of this material may be reproduced or transmitted in any form or by any means -
electronic or mechanical including photocopying – without written permission from the DepEd Central Office. First Edition, 2016.
3.6.2. Half-Angle Identities

Recall two of the three double-angle identities for cosine:


2 2
cos 2A = 2 cos A 1 and cos 2A = 1 2 sin A:
2
From these identities, we obtain two useful identities expressing sin A and
2
cos A in terms of cos 2A as follows:
2 2
cos A = 1 + cos 2A and sin A = 1 cos 2A :
2 2

Some Useful Identities


2 2
cos A = 1 + cos 2A sin A = 1 cos 2A
2 2

From these identities, replacing A with A2 , we get


A
A 1 + cos 2 2 1+ cos A
2
cos 2 = 2 = 2
and
A 1 cos 2 A
2 1 cos A
2
sin 2 = 2 = 2 :
These are the half-angle identities for sine and cosine.

Half-Angle Identities for Sine


2
and Cosine
cos2 2 =1+ 2 sin 2= 2
A cos A A 1 cos A

Because of the \square" in the formulas, we get


r
cos 2 = and sin 2 = r
1+ 2 2 :

A cos A A 1 cos A

The appropriate signs of cos A and sin A depend on which quadrant A lies.
22 2

Example 3.6.4. Find the exact values of sin 22:5 and cos 22:5 .
Solution. Clearly, 22:5 lies in QI (and so sin 22:5 and cos 22:5 are both posi-
tive), and 22:5 is the half-angle of 45 .
p
2
1 cos 45 1 2 2 p2
sin 22:5 = =s =
r 2 2 p 2
p
p
s 1+ 2+
2
1 + cos 45 2 2
cos 22:5 = = = 2
r 2 2 p 2
1
9
5
All rights reserved. No part of this material may be reproduced or transmitted in any form or by any means -
electronic or mechanical including photocopying – without written permission from the DepEd Central Office.
First Edition, 2016.
+ sin
tan

2
Example 3.6.5. Prove: cos 2 = 2 tan .
Solution. 2
cos 2 = 1+ 2
cos
= 1 +2 tan
cos tan

=tan + cos tan


2 tan
sin
tan + cos cos
=
2 tan
=tan + sin 2
2 tan
We now derive the rst version of the half-angle formula for tangent.
A sin A
2
tan 2 = cos A
cos A2
2 2 sin A2 !
sin A 2 sin A
2
= 2

2
2 sin cos
A
2

= 2
2 sin 2

A A

1cos A
2 2
= A
sin 2 2
1 cos A
sin A

There is another version of the tangent half-angle formula, and we can


derive it from the rst version.
A 1 cos A
tan =
2 = 1 sin A sin A
1 + cos A
cos A 1 + cos A

2
0 cos A
(sin A)(1 + cos A)
2
sin A
(sin A)(1 + cos A) sin
A
1 + cos A

196
All rights reserved. No part of this material may be reproduced or transmitted in any form or by any means -
electronic or mechanical including photocopying – without written permission from the DepEd Central Office. First Edition, 2016.
Tangent Half-Angle Identities
A 1 cos A A sin A
tan = tan =
2 = sin A 2 1 + cos A
tan 2 cos A2 tan 2 = 1 + cos A
A
A sin 2 2 A 1 cos A

Example 3.6.6. Find the exact value of tan 12 .

Solution.
p3
1 cos 6 1 2
tan 12 = sin = 1
=2 p 3 2
6 2

Example 3.6.7. If sin = 2 , cot > 0, and 0 < 2 , nd sin , cos , and
tan . 5 2 2
2

Solution. Since sin < 0 and cot > 0, we conclude the < < 3 . It follows
2
that
3
< < ;
2 2 4
which means that lies in QII.
2
2
cos = p1 sin = s 1 5 =
5
2
2 p 21
v p
sin = 1 cos = 1 5
= 50 + 10 21
u
2 r 2
u 2
p 21 p 10

t
p
cos = 1 + cos = v 1+ 5 =
50 10 21
u
2 r
u2 2
p 21 p 10

p 21 p
1 cos 1 5 5+ 21
tan = = 2 = 2
2 sin 5 2

More Solved Examples


5
1. If cos = 13 with 0 < < , nd sin 2 and cos 2 .
197
All rights reserved. No part of this material may be reproduced or transmitted in any form or by any means -
electronic or mechanical including photocopying – without written permission from the DepEd Central Office. First Edition, 2016.
2 2
Solution: In this problem, we use the Pythagorean identity sin + cos = 1.
Because cos = 135 , we must have

2 2 25 144
sin = 1 cos =1
169 = 169 :
Moreover, since 0 < < , we take the square root of both sides of the above
equation to get
sin = 12 :
13
Now, using the double-angle identities we get

2 2
sin 2 = 2 sin cosand cos 2 = cos sin

2 = 25 144
169 169
1313
120
119
512 = 169 :
169

Derive an identity for cos 3x in terms of cos x.


Solution: We use the sum identity for cosine, the double-angle identities
for sine and cosine, and the Pythagorean identity.

cos 3x = cos(2x + x)
= cos 2x cos x sin 2x sin x
2
= (2 cos x 1) cos x (2 sin x cos x) sin x
3 2
= 2 cos x cos x 2 sin x cos x
3 2
= 2 cos x cos x 2(1 cos x) cos x
3
= 4 cos x 3 cos x:

Derive the identity for tan 3t in terms of tan t. Solution:


Using the sum identity for tangent, we obtain

tan 3t = tan(2t + t) = tan 2t + tan t :


1 tan 2t tan t
Now, using the tangent double-angle identity, we have
2 tan t
2 + tan t
tan 3t = 1tan t
:
2 tan t
1 2 tan t
1tan t

Upon simplifying the terms on the right side of the equation, we nally obtain
3
3 tan t tan t
2
tan 3t = 1 3 tan t :
198
All rights reserved. No part of this material may be reproduced or transmitted in any form or by any means -
electronic or mechanical including photocopying – without written permission from the DepEd Central Office. First Edition, 2016.
Find the exact value of sin 12 .
21 1cos y .
Solution: To nd the value of sin 12 , we use the identity sin 2 y= 2
With y = 6 , we obtain
p
1 1 cos 1 3 2 p 3
6 2
2 2
sin 12 = sin 2 6 = 2 = 2 = 4 :
Now, since 0 < 12 < , sin 12 must be positive, and so

2 p
sin 12 = p 2 3:
sec 1
2
5. Prove: sin 2 = 2 sec .
Solution:
sin22 = 1 2
cos
= 1 2 sec
cos sec

sec
cos sec
=
2 sec
1
= sec cos cos
2 sec
sec 1
= :
2 sec

6. Use the half-angle identity to nd the exact value of tan 75 .


p

1
5
0

1 =1cos 150 1+ 3
2
p
Solution: tan 75 = tan = =2+ 3.

1
2 sin 150 2
A ball is thrown following a projectile motion. It is known that the horizontal
distance (range) the ball can travel is given by
v2
0
R = g sin 2 ;
where R is the range (in feet), v 0 is the initial speed (in ft/s), is the angle of
2
elevation the ball is thrown, and g = 32 f t=s is the acceleration due to
gravity.

Express the new range in terms of the original range when an angle (0 <
45 ) is doubled?

If a ball travels a horizontal pdistance of 20 ft when kicked at an angle of with


initial speed of 20 2 ft/s, nd the horizontal distance it can travel when
you double . Hint: Use the result of item (a)

199
All rights reserved. No part of this material may be reproduced or transmitted in any form or by any means -
electronic or mechanical including photocopying – without written permission from the DepEd Central Office. First Edition, 2016.
Solution:
2
(a) Let R = v0 sin 2 be the original range. When the angle is doubled, the
g
2
0
new range will become R = vg0 sin 4 . Now, we solve sin 4 in terms of
the original range.
Note that sin 2 = gR . So, as a consequence of the fundamental identity,
2

v 0
we obtain s
1 v04 p

4 2 2
cos 2 = = v02 :

g2R2 v0 g R

Since sin 4 = 2 sin 2 cos 2 , it follows that


g g v02 p ! p
2 2 4 2 2
gR
v02

v v v g R
0 0 0 2R v04 g2R2 :
0 2
R = sin 4 = 2 v0

(b) Using the result in (a), if is doubled, then the new range is given by
p
2
p v0

2R v04 g2R2 640000 409600


800

40
R0 = = = 24:

Therefore, the new horizontal distance is 24 ft.

Supplementary Problems 3.6


1. Let be an angle in the rst quadrant and sin = 13 . Find
(a) sin 2 (c) tan 2 (e) csc 2
(b) cos 2 (d) sec 2 (f) cot 2

Find the approximate value of csc 46 and sec 46 , given that sin 23 0:3907:
3
If cos t = 4 , what is cos 2t?
Derive a formula for sin 4x in terms of sin x and cos x.

5. Let 4 < x < 0. Given that tan 2x = 2, solve for tan x.

Obtain an identity for tan 4 in terms of tan .


1
Solve for the exact value of cot 4 if tan = 2 .
2 2
Use half-angle identities to nd the exact value of (a) sin 15 and (b) cos 15 .
25 25
Use half-angle identities to nd the exact value of (a) sin 8 and (b) cos 8 .

200
All rights reserved. No part of this material may be reproduced or transmitted in any form or by any means -
electronic or mechanical including photocopying – without written permission from the DepEd Central Office. First Edition, 2016.
10. Find the exact value of cos 8 .
tan 1 y 1 sin y cos y 1
2
11. Prove that tan 1 y + 1 = sin y + cos y + 1 .
2
1
Verify that the following equation is an identity: cot 2 t = cot t(sec t + 1).
Use half-angle identities to nd the exact value of (a) cos 105 and (b) tan 22:5 .

Lesson 3.7. Inverse Trigonometric Functions

Learning Outcomes of the Lesson


At the end of the lesson, the student is able to:
graph the six basic inverse trigonometric functions;
illustrate the domain and range of the inverse trigonometric functions;
evaluate inverse trigonometric expressions; and
solve situational problems involving inverse trigonometric functions.

Lesson Outline
De nitions of the six inverse trigonometric functions
Graphs of inverse trigonometric functions
Domain and range of inverse trigonometric functions
Evaluation of inverse trigonometric expressions

Introduction
In the previous lessons on functions (algebraic and trigonometric), we com-
puted for the value of a function at a number in its domain. Now, given a value in
the range of the function, we reverse this process by nding a number in the
domain whose function value is the given one. Observe that, in this process, the
function involved may or may not give a unique number in the domain. For
2
example, each of the functions f(x) = x and g(x) = cos x do not give a unique
number in their respective domains for some values of each function. Given f(x)
= 1, the function gives x = 1. If g(x) = 1, then x = 2k , where k is an integer.
Because of this possibility, in order for the reverse process to produce a
function, we restrict this process to one-to-one functions or at least restrict the
domain of a non-one-to-one function to make it one-to-one so that the process

201
All rights reserved. No part of this material may be reproduced or transmitted in any form or by any means -
electronic or mechanical including photocopying – without written permission from the DepEd Central Office. First Edition, 2016.
works. Loosely speaking, a function that reverses what a given function f
1
does is called its inverse function, and is usually denoted by f .
More formally, two functions f and g are inverse functions if
g(f(x)) = x for any x in the domain of f,
and
f(g(x)) = x for any x in the domain of g.
1
We denote the inverse function of a function f by f . The graphs of a function
and its inverse function are symmetric with respect to the line y = x.
In this lesson, we rst restrict the domain of each trigonometric function
because each of them is not one-to-one. We then de ne each respective inverse
function and evaluate the values of each inverse trigonometric function.

3.7.1. Inverse Sine Function

All the trigonometric functions that we consider are periodic over their entire
domains. This means that all trigonometric functions are not one-to-one if we
consider their whole domains, which implies that they have no inverses over
those sets. But there is a way to make each of the trigonometric functions
one-to-one. This is done by restricting their respective domains. The
restrictions will give us well-de ned inverse trigonometric functions.
The domain of the sine function is the set R of real numbers, and its range is
the closed interval [1; 1]. As observed in the previous lessons, the sine function
is not one-to-one, and the rst step is to restrict its domain (by agreeing what the
convention is) with the following conditions: (1) the sine function is one-to-one in
that restricted domain, and (2) the range remains the same.

The inverse of the (restricted) sine function f(x) = sin x, where the
1 1

domain is restricted to the closed interval ; , is called the inverse


2 2
1 1
sine function or arcsine function, denoted by f (x) = sin x or
f (x) = arcsin x. Here, the domain of f (x) = arcsin x is [1; 1],
; Thus,
and its range is 2 2 . 1
y = sin x or y = arcsin x

if and only if
sin y = x;

where 1x 1 and 2 y2 .

1
Throughout the lesson, we interchangeably use sin x and arcsin x to mean
the inverse sine function.

202
All rights reserved. No part of this material may be reproduced or transmitted in any form or by any means -
electronic or mechanical including photocopying – without written permission from the DepEd Central Office. First Edition, 2016.
Example 3.7.1. Find the exact value of each expression.
1 1
(1) sin 2 (3) arcsin 0
1 1

(2) arcsin(1) 1 1 (4) sin 2 1


Solution. (1) Let = sin
1
. We get = . Thus, we have sin1
2 . This is equivalent to sin = 2 . This means that
1
=6.

we are looking for the number in the closed interval ; whose sine is
2 6 2

2 2

(2) arcsin(1) = 2 because sin 2 = 1 and 2 2 2; 2 .


(3) arcsin 0 = 0
1 1

(4) sin 2 = 6 1 2
emphasized in the last example, as long as 1 x 1, sin x is that
As 2 j j 1
such that sin y = x. If x > 1, then sin x is not de ned in
R. 2 2

number y ;
1
We can sometimes nd the exact value of sin x (that is, we can nd a value
in terms of ), but if no such special value exists, then we leave it in the form
1 1
sin x. For example, as shown above, sin 12 is equal to 6 . However, as studied
in Lesson 3.2, no special number satis es sin = 2 , so we leave sin1 2 as is.
3 3

Example1 3.7.2.1 Find the exact value of each expression.


(1) sin sin (3) arcsin(sin )
2 2

1
(2) arcsin sin 3 1 1 2
p3 1 (4) sin sin
Solution. (1) sin sin = sin 6 =2

(2) arcsin sin 3 = arcsin 2 = 3


(3) arcsin(sin ) = arcsin 0 = 0
1 1 1

(4) sin sin 2 = sin 6 =2 2


From the last example, we have the following observations:

1. sin(arcsin x) = x for any x 2 [1; 1]; and


2. arcsin(sin ) = if and only if 2 ’ ’ 2 ;
2 ;2 , and if 62
such that sin ’ = sin .
2 ; 2 , then

arcsin(sin ) = , where 2 2
1
To sketch the graph of y = sin x, Table 3.32 presents the tables of values
1 1
for y = sin x and y = sin x. Recall that the graphs of y = sin x and y = sin x
are symmetric with respect to the line y = x. This means that if a point (a; b)
1
is on y = sin x, then (b; a) is on y = sin x.

203
All rights reserved. No part of this material may be reproduced or transmitted in any form or by any means -
electronic or mechanical including photocopying – without written permission from the DepEd Central Office.
First Edition, 2016.
x 0
y = sin x 2 3 4 6 6 4 3 2
p3 p2 1 1 p2 p3
y 1 2 2 2
0 2 2 2
1

p p2 1 1 p2 p 3
x 1 3
0 1
1 2 2 2 2 2 2
y = sin x
y 2 3 4 6
0 6 4 3 2

Table 3.32

The graph (solid thick curve) of the restricted sine function y = sin x is
shown in Figure 3.33(a), while the graph of inverse sine function y = arcsin x
is shown in Figure 3.33(b).

1
(a) y = sin x (b) y = sin x

Figure 3.33

1
Example 3.7.3. Sketch the graph of y = sin (x + 1).

Solution 1. In this solution, we use translation of graphs.


1 1
Because y = sin (x + 1) is equivalent to y = sin [x (1)], the graph of y =
1 1 1
sin (x + 1) is 1-unit to the left of y = sin x. The graph below shows y = sin
1
(x + 1) (solid line) and y = sin x (dashed line).

204
All rights reserved. No part of this material may be reproduced or transmitted in any form or by any means -
electronic or mechanical including photocopying – without written permission from the DepEd Central Office. First Edition, 2016.
Solution 2. In this solution, we graph rst the corresponding sine function, and
then use the symmetry with respect to y = x to graph the inverse function.
1
y = sin (x + 1) () sin y = x + 1 () x = sin y 1
1
The graph below shows the process of graphing of y = sin (x + 1) from y =
sin x 1 with 2 x 2 , and then re ecting it with respect to y = x.

3.7.2. Inverse Cosine Function

The development of the other inverse trigonometric functions is similar to that


of the inverse sine function.

205
All rights reserved. No part of this material may be reproduced or transmitted in any form or by any means -
electronic or mechanical including photocopying – without written permission from the DepEd Central Office. First Edition, 2016.
1
y = cos x or y = arccos x
means
cos y = x;
where 1 x 1 and 0 y .

The graph (solid thick curve) of the restricted cosine function y = cos x is
shown in Figure 3.34(a), while the graph of inverse cosine function y =
arccos x is shown in Figure 3.34(b).

1
(a) y = cos x (b) y = cos x

Figure 3.34

Example 3.7.4. Find the exact value of each expression.


1 1 3
(1) cos 0 (4) cos cos 4
p
3
(2) arccos (5) arccos cos 6
7

2
p p
3 2

(3) cos cos1 2 (6) sin cos1 2


1
Solution. (1) cos 0 = 2 because cos 2 = 0 and 2 2 [0; ].
p
(2) arccos 3 = 5
2 6
p p p
3 3 3
(3) cos cos1 2 = 2 because 2 2 [1; 1]
3 3 3

1
(4) cos cos 4 = 4 because 4 2 [0; ].
206
All rights reserved. No part of this material may be reproduced or transmitted in any form or by any means -
electronic or mechanical including photocopying – without written permission from the DepEd Central Office. First Edition, 2016.
p
7
3 5

(5) arccos cos 6= arccos 2 = 6


pp
1
(6) sin cos 2 = 2 2
2 2
2 1
Example 3.7.5. Simplify: sin arcsin 3 + arccos 2 .

Solution. We know that arccos 12 = 3 . Using the Sine Sum Identity, we have sin arcsin 23 +

arccos 12
= sin arcsin 2 +
3 3
= 3 2
2 3 2
2
= sin arcsin 3
+ cos

cos 3 + cos arcsin 3 sin 3

2 1 2 p3
arcsin
= 1 + p 3 cos arcsin 2 :
3 2 3
2 2
We compute cos arcsin . Let = arcsin 3 . By de nition, sin = 23 , where
3
lies in QI. Using the Pythagorean identity, we have
2 p5
2
p
cos arcsin 3 = cos = 1 sin = 3:
Going back to the original computations above, we have
2 1 = 1 p3 2
sin arcsin + arccos =3+ cos
3 2 1 + p 3 p5

2 arcsin 3
3 2 3
p
2+ 15
= 6
: 2
1
Simplify: sin 2 cos 4

Example 3.7.6. 1 4 5 . 4
Solution. Let = cos 5 . Then cos = 5 . Because cos < 0 and range
of inverse cosine function is [0 ; ], we know that must be within the interval 3

2 ; . Using the Pythagorean Identity, we get sin = 5 .


Using the Sine Double-Angle Identity, we have
sin 2 cos1 4
5
= 2 sin cos
= sin 2
3 4
= 2524 : 2

=2 5 5

1 1
Example 3.7.7. Sketch the graph of y = 4 cos (2x).

Solution.
1 1
1 1
y = 4 cos (2x) () 4y = cos (2x) () x = 2 cos(4y)
207
All rights reserved. No part of this material may be reproduced or transmitted in any form or by any means -
electronic or mechanical including photocopying – without written permission from the DepEd Central Office. First Edition, 2016.
1
We graph rst y = 2 cos(4x). The domain of this graph comes from the restriction
of cosine as follows:
0 4x =) 0 x 4:

Then re ect this graph with respect to y = x, and we nally obtain the graph of
1 1
y = 4 cos (2x) (solid line).

In the last example, we may also use the following technique. In graphing
1 1 1
y = 4 cos (2x), the horizontal length of cos x is reduced to half, while the
vertical height is reduced to quarter. This comparison technique is shown in
1
the graph below with the graph of y = cos x in dashed line and the graph of
1 1
y = 4 cos (2x) in solid line.

3.7.3. Inverse Tangent Function and the Remaining Inverse


Trigonometric Functions

The inverse tangent function is similarly de ned as inverse sine and inverse
cosine functions.

208
All rights reserved. No part of this material may be reproduced or transmitted in any form or by any means -
electronic or mechanical including photocopying – without written permission from the DepEd Central Office. First Edition, 2016.
1
y = tan x or y = arctan x
means
tan y = x;
where x 2 R and 2< y< 2 .

The graph (solid thick curve) of the restricted function y = tan x is shown
in Figure 3.35(a), while the graph of inverse function y = arctan x is shown in
Figure 3.35(b).

1
(a) y = tan x (b) y = tan x

Figure 3.35

Example 3.7.8. Find the exact value of each expression.


1 1
(1) tan 1 (4) tan tan 6
1 7
(2) arctan p3 (5) tan tan 6
5 19
(3) tan tan1 2 (6) arctan tan 6

Solution. Note the range of arctan is the open interval 2; 2 .


1
(1) tan 1 = 4

(2) arctan 1
1 p 3 5= 3 5
(3) tan tan 2 = 2

(4) tan tan 6 = 6 because 6 2 2 ;2 .

209
All rights reserved. No part of this material may be reproduced or transmitted in any form or by any means -
electronic or mechanical including photocopying – without written permission from the DepEd Central Office.
First Edition, 2016.
7
.
tan 6 = tan 6
;
that 7 . Use the idea of reference angle, we know that

(5) Here, note 6 62 2 2


7

1 1
tan tan 6 = tan tan 6 =6
(6) Here, we cannot use the idea of reference angle, but the idea can help in
19
a way. The number (or angle) 6 is in QII, wherein tangent is negative,
and its reference angle is 6 .

19
arctan tan = arctan 6 6

= 2
6
Example 3.7.9. Find the exact value of each expression.
8 1 3 1

1 1
(1) sin 2 tan 3 1 8 (2) tan 8sin 5 tan 4
Solution. (1) Let = tan 3
. Then tan = 3
. Following the notations in
Lesson 3.2 and the de nition of inverse tangent function, we know that lies
2
p
in QIV, and x = 3 and y = 8. We get r= 3 + (8) 2 = 73.
(page 192) giv es

Applying the Sine Double-Angle Identity p

8
1
sin 2 tan = sin 2
3
= 2 sin cos
y x
=2 r r p
=2 p
73
73
8 3

48
= :
73

(2) Using the Tangent Di erence Identity, we obtain

3
tan sin 1 tan 1 1
5 4
tan
1
1 + tan sin 5 tan 4
1 1
tan sin 5 tan tan
3 1
4
= 1 3 1
= tan sin 5 4 :
1 3 1
1 + tan sin 5 4
1 3 1

1 3
We are left to compute tan sin 5 . We proceed as in (1) above. Let
13 3
= sin 5 . Then sin = 5 . From the de nition of inverse sine function and
210
All rights reserved. No part of this material may be reproduced or transmitted in any form or by any means -
electronic or mechanical including photocopying – without written permission from the DepEd Central Office. First Edition, 2016.
the notations used in Lesson 3.2, we know that lies in QI, and y = 3 and
r = 5. We get x = p 52 32 = 4, so that tan = y = 3 .
x 4

1 3 1
1 1
1
5 4 1 + tan sin1 5
3 4

3 1 tan sin 5 4

tan sin tan =


= 4 4
3 1

3 1
1+ 4 4

=8 2
19

?
Example 3.7.10. A student is
viewing a painting in a museum.
Standing 6 ft from the painting,
the eye level of the student is 5
ft above the ground. If the paint-
ing is 10 ft tall, and its base is 4
ft above the ground, nd the
viewing angle subtended by the
painting at the eyes of the stu-
dent.

Solution. Let be the viewing angle, and let = + as shown below. We


observe that
1 9
tan = and tan = :
6 6
Using the Tangent Sum Identity, we have
tan + tan
tan = tan( + )=
tan tan
1 9
6 + 6
= 1 9
1 6 6
20 :
=
9
20
1 9

Using a calculator, the viewing angle is = tan 65:8 . 2


We now de ne the remaining inverse trigonometric functions.

211
All rights reserved. No part of this material may be reproduced or transmitted in any form or by any means -
electronic or mechanical including photocopying – without written permission from the DepEd Central Office. First Edition, 2016.
De ne
1 1
cot x= 2 tan x:
1
It follows that the domain of y = cot x is R and its range is (0; ).

1
y = sec x or y = arcsec x
means
sec 3y = x;
where jxj 1 and y 2 0; [ ; .
2 2

De ne
1 1
csc x =2 sec x:
the domain of y = csc
1
x is (; 1] [1; ) and
This means that [ 1

its range is ; 2 [ 0; 2 .
The graphs of these last three inverse trigonometric functions are shown
in Figures 3.36, 3.37, and 3.38, respectively.

1
(a) y = cot x (b) y = cot x

Figure 3.36

212
All rights reserved. No part of this material may be reproduced or transmitted in any form or by any means -
electronic or mechanical including photocopying – without written permission from the DepEd Central Office. First Edition, 2016.
1
(a) y = sec x (b) y = sec x

Figure 3.37

1
(a) y = csc x (b) y = csc x

Figure 3.38

Observe that the process in getting the value of an inverse function is the
1
same to all inverse functions. That is, y = f (x) is the same as f(y) = x. We need
to remember the range of each inverse trigonometric function. Table 3.39
summarizes all the information about the six inverse trigonometric functions.

213
All rights reserved. No part of this material may be reproduced or transmitted in any form or by any means -
electronic or mechanical including photocopying – without written permission from the DepEd Central Office.
First Edition, 2016.
Function Domain Range Graph
Figure
1
sin x [1; 1] 2; 2 3.33(b)
1 Figure
cos x [1; 1] [0; ] 3.34(b)
Figure
1
tan x R 2; 2 3.35(b)
1 Figure
cot x R (0; ) 3.36(b)
3 Figure
1
sec x fx : jxj 1g 0; 2 [ ; 2 3.37(b)
Figure
1
csc x fx : jxj 1g ; 2 [ 0; 2 3.38(b)
Table 3.39

Example 3.7.11. Find the exact value of each expression.


1 1
(1) sec (2) (3) cot p 3
1 p
2 3 3 p
2 3

1 1
(2) csc 3 (4) sin sec 2 csc 3
4 4 4 3
1
Solution. (1) sec (2) = 3 because sec 3 = 2 and 3 2 ; 2
p
1 2 3 2
(2) csc 3 = 3

5
1
(3) cot p3 = 6
p
1 2 3 2 1 3
(4) From (2), we know that csc 3 = 3. Let = sec 2 . Then
3 inverse secant function and the notations
sec = 2 . From de ned range of
in Lesson 3.2, lies in QIII, and r = 3 and x = 2. Solving for y, we get
y= 32 p (2)
5. It follows that sin
2= = p 5 and cos = 2 .
now use the Sine Sum Identity. 3 3

We p
3 2p 3
1 1
sin sec 2 csc 3 !!

= sin 3
2
sin + 3

214
All rights reserved. No part of this material may be reproduced or transmitted in any form or by any means -
electronic or mechanical including photocopying – without written permission from the DepEd Central Office.
First Edition, 2016.
= sin cos 2 + cos sin 2
3 3
p 5 1 2 p
3

= 3 ! 2 + 3 2 !
p5 2p 3
= 2
6

More Solved Examples


1. Find the exact values of the following, if they exist.
p 1 1
2

1
(a) sin 2 (b) arcsin 2 (c) sin 2
1
Solution: Note that the range of f(x) = sin x is [ 2 ; 2 ]. Thus, if we let
1
y = sin x, then we are looking for y 2 [ 2 ; 2 ] such that sin y = x. Hence,
1 p 2 = ,
(a) sin 2 4
1
(b) arcsin 2
= 6, and
1
unde ned because sin y 1.

(c) sin 2 is
2. Find the exact value of each expression.
p
2 1 1 11

1
(a) sin sin 2 (b) cos arcsin 2 (c) sin sin 2
Solution:
1 p 2 p 2

(a) sin sin 2 1= sin( 4 ) = 2 p


(c) sin sin 2 2
3

(b) cos arcsin


1 11
2
= sin1
= cos( 6 ) =
(1) =
2

Answer the following.


1
0 What is the domain of y = sin 2x?
1
(b) What is the range of y = sin 2x?
1
(c) What is the xintercept of y = sin 2x?
Solution:
1 1
(a) Consider the function f( ) = sin . The domain of sin is [1; 1].
1
So, = 2x 2 [1; 1]. Therefore, the domain of sin 2x is [1=2; 1=2].

215
All rights reserved. No part of this material may be reproduced or transmitted in any form or by any means -
electronic or mechanical including photocopying – without written permission from the DepEd Central Office. First Edition, 2016.
1 [ =2; =2]
2 (0; 0)

From the concept of projectile motion, if an object is directed at an angle


v2

(with 2 [0; =2]), then the range will be R = g0 sin 2 (in feet) where v 0 (in f
2
t=s) is the initial speed and g = 32 f t=s is the acceleration due to gravity.
At what angle shall the object be directed so that the range will be 100 f t,
given that the initial speed is v0 = 80 f t=s?
Solution: From the formula of the range, we get
2
80 1
100 = 32 sin 2 =) 2 = sin 2
from 0 to (i.e. 0 2 ), this is equivalent to nding 2
Since must be 1 1 2
such that 2 = sin 2 . Hence,

2 = 6 =) = 12 :
Therefore, the object must be directed at an angle of 12 rad (or 15 ), to
have a projectile range of 100 f t.

5. Find the exact values of the following, if they exist.


p
1 2
(a) cos 2 (c) arccos(cos )
1

1
(b) cos cos 2 (d) arccos
Solution:
p
1 2
(a) cos 2 = 4
1
(b) cos cos 1
2
= cos 2 = 1
2
3

(c) arccos(cos ) = arccos(1) =


(d) Let y = arccos . Since cos y 1, we have y is unde ned because > 3.

1 p 3 1 1
6. Simplify: (a) cos cos 2 cos 3
p
1 3 11
Solution: We know that cos 2 = 6 . Let = cos 3 . Which is equivalent to
1
cos = 3 with 0 . Using the Cosine Di erence Identity, we have
p !
3 1
1 1
cos cos 2 cos 3 = cos 6 = cos 6 cos + sin 6 sin
p
31 1
2 3 + 2 sin :
216
All rights reserved. No part of this material may be reproduced or transmitted in any form or by any means -
electronic or mechanical including photocopying – without written permission from the DepEd Central Office. First Edition, 2016.
Now, we solve for sin using the Pythagorean Identity which gives
2 2
sin =1 (1=3) = 1 (1=9) = 8=9:
2p 2
Thus, sin = 3 because 2 [0; ]. Finally, we obtain
1 1
cos cos p cos 2
!=p +2
3
p 23 3 3

3 1 1 1 2 2
p p
= 3 + 2 2
6 6
p p

3+2 2
= :
6
2
1 2 1
7. Simplify: (a) cos 2 cos 5 ; (b) sin cos 5
1 2 2
Solution: Let = cos 5 . Which is equivalent to cos = 5 with 0
. Using the Double-Angle Identity for Cosine and one of the Fundamental
Idenity, we have
2 2 2 8 17
1 2
cos 2 cos 5 = cos(2 ) = 2 cos 1=2 5 1 = 25 1 = 25
and
p r
sin cos1 = sin = 1 cos2 =
p
5 1 25
=
5 :

2 4 21

Here, sin 0 because 2 [0; ].


1
8. Graph: y = 1 + cos x
Solution: The graph can be obtained by translating the graph of the
inverse cosine function one unit upward.

217
All rights reserved. No part of this material may be reproduced or transmitted in any form or by any means -
electronic or mechanical including photocopying – without written permission from the DepEd Central Office. First Edition, 2016.
9. Find the exact value of each expression.
4 1 4
(a) arctan(tan 3 ) (b) tan(tan 5 )

Solution:
p
(a) arctan(tan 4 ) = arctan 3= =3
3
1 4 4
(b) tan(tan 5 ) =5
1 7 1 1
10. Find the exact value of tan(tan 6 + tan 2 ).
Solution:
1 1 1
1
7 1
1 tan(tan 7
6
) + tan(tan 2
)
tan + tan 1 7 1 1
tan 6 2 =1 tan(tan 6 ) tan(tan 2 )
7
6 + 12
= 7 1
1 6 2
=4

11. Find the exact values of the following, if they exist.


1
p2 (c) cot
1 p 3 1
(a) sec cos(arccsc 2)
3
1 1 1
arccot (sin
20
)
(b) csc 1 (d) arcsec (cot( 4 )) 3

Solution:
1
(a) sec p2= 4
1
(b) csc 1= 2
p
1
(c) cot 3 =
3 3
(d) arcsec(cot( 4
)) = arcsec(1) =
(e) cos(arccsc(2)) = cos 6
= p3
2

(f) arccot(sin 20 ) = arccot p3 . Let = arccot p 3 . Then,


3 2 2

p3 2
cot = 2 =) tan = p3
2
1
=) = tan p3 ( 0:8571):
Here, we needed to use a calculator to solve for the approximate value,
2
since p3 is not a special value for tangent function.
218
All rights reserved. No part of this material may be reproduced or transmitted in any form or by any means -
electronic or mechanical including photocopying – without written permission from the DepEd Central Office. First Edition, 2016.
Supplementary Problems 3.7
1. Find the exact value of the following.
p 2)]
(a) sin[sin (1=2)]1 (e)cos[arccos(
1
(b) cos[cos ( p 2 =2)]
1
(c) tan[tan ( p 3)] (f)tan[arcsin(1=4)]
p 1 p
(d) sin[arctan( 3)] (g)cos[sin ( 3=2)]

2. Find the exact value of the following.


1 1 1
(a) sin [sin(25 =6)] (c) tan [tan(1)] (e) cos [sec(23 )]
(b) arccos[cos(23 =4)] (d) arcsin[cos(13 =4)] (f) arctan[sin( =12)]

Solve the exact value of the following.


1
0 sin[2 cos (4=5)]
1
1 cos[2 sin (5=13)]
1 1
2 sin(sin (3=5) + cos (5=13))
1 1
3 cos[sin (1=2) cos (8=17)]
1
Consider the function f(x) = tan (x + 1). Do the following.
0 Find the domain of f.
1 Find the range of f.
(c) Find the x and yintercept of f, if there are any.
2 Graph f.

Evaluate and simplify the following, if they exist.


1 1
(a) arcsec( p 2) (d) [sec (1)] [cos (1)]
1
(b) arccsc(2) (e) 2 cot p3 + 3 csc
1
2
p 1
(c) arccot 3 (f) csc 0
6. Evaluate and simplify the following, if they exist.
1 1
cos(sec 3 + tan 2) 2 sin
1
(1=2)
1
(a) cos(tan 2) (c) cos 2
(b) tan(2 arcsin(1=6)) (d) arcsec(sin(100 =3))

219
All rights reserved. No part of this material may be reproduced or transmitted in any form or by any means -
electronic or mechanical including photocopying – without written permission from the DepEd Central Office. First Edition, 2016.
7. A trough is in the shape of an inverted triangular prism whose cross section
has the shape of an inverted isosceles triangle (see Figure 3.40). If the length
p
of the base of the cross section is 2 3 m: and the length of the trough is p
3
3 m:, nd the size of the vertex angle so that the volume is 900 m .
Hint: V = bhl=2:

Figure 3.40

Lesson 3.8. Trigonometric Equations

Learning Outcomes of the Lesson


At the end of the lesson, the student is able to:
solve trigonometric equations; and
solve situational problems involving trigonometric equations.

Lesson Outline
De nition of a trigonometric equation
Solution to a trigonometric equation
Techniques of solving a trigonometric equation

Introduction
We have studied equations in Lesson 3.4. We di erentiated an identity from a
conditional equation. Recall that an identity is an equation that is true for all

220
All rights reserved. No part of this material may be reproduced or transmitted in any form or by any means -
electronic or mechanical including photocopying – without written permission from the DepEd Central Office. First Edition, 2016.
values of the variable in the domain of the equation, while a conditional
equation is an equation that is not an identity.
In this lesson, we mostly study conditional trigonometric equations. Though not explicitly, we
have started it in the preceding lesson. For example, the equa-
1
tion sin x = 1 has the unique solution x = sin 1 = in the closed interval
2 1 R 2 6

; . However, if we consider the entire domain (not the restricted domain)


2 2

of the sine function, which is the set of real numbers, there are solutions (other
than 6 ) of the equation sin x = 2 . This current lesson explores the techniques
of solving (conditional) trigonometric equations.
We divide the lesson into two groups of equations: the ones using a basic
way of solving, and those using more advanced techniques.

3.8.1. Solutions of a Trigonometric Equation

Any equation that involves trigonometric expressions is called a trigonometric


equation. Recall that a solution or a root of an equation is a number in the domain of
the equation that, when substituted to the variable, makes the equation true. The set
of all solutions of an equation is called the solution set of the equation.
Technically, the basic method to show that a particular number is a
solution of an equation is to substitute the number to the variable and see if
the equation becomes true. However, we may use our knowledge gained
from the previous lessons to do a quicker veri cation process by not doing
the manual substitution and checking. We use this technique in the example.
Example 3.8.1. Which numbers in the set ; ; ; ; ; 2 ; 3 ; 5 ; ;
solutions to the following equations? 0 64 3 2 3 4 6 2 are
1 2 2
(1) sin x = 2 (7) cos x = cos 2x + sin x
(2) tan x = 1 (8) sin x + cos 2x = 0
(3) 3 sec x = 2 p 3 (9) 2 sin x + tan x 2 cos x = 2
2 2
(4) p 3j cot xj = 1 (10) sin x + cos x = 2
2 2
(5) sec x tan x = 1 (11) sin 2x = sin x
(6) sin x + cos x = 0 (12) 2 tan x + 4 sin x = 2 + sec x

Solution. Note that the choices (except 2 ) are numbers within the interval [0; ].
To quickly determine which numbers among the choices are solutions to a par-
ticular equation, we use some distinctive properties of the possible solutions.

The sine function is positive on (0; ). From Lesson 3.2, we recall that 6 is an
obvious solution. We may imagine the graph of y = sin x. We may also
5
use the idea of reference angle. Thus, among the choices, only 6 and 6
1
are the only solutions of sin x = 2 .

221
All rights reserved. No part of this material may be reproduced or transmitted in any form or by any means -
electronic or mechanical including photocopying – without written permission from the DepEd Central Office. First Edition, 2016.
(2) Since tan x = 1 > 0, any solution of the equation among the choices must
be in the interval 0; (that is, in QI). Again, among the choices, the only
2

solution to tan x = 1 is 4 .
p
2 3
(3) Here, the given equation is equivalent to sec x = . Among the choices,
3
the only solution of the equation 3 sec x = 2 p 3 is 5
.
6
(4) Eliminating the absolute value sign, the given equation is equivalent to cot x =
p 3 or cot x = p 3 . Among the choices, the only solution of cot x = p 3 is ,
3 3
2
p 3 3
while the other equation has . Thus, the only solutions of 3 cot x =1
3
from the given set are and 2 . j j
3 3

The given equation is one of the Pythagorean Identities (page 175). It means
that all numbers in the domain of the equation are solutions. The domain
of the equation is R n fx : cos x = 0g. Thus, all except 2 are solutions of
2 2
sec x tan x = 1.
For the sum of sin x and cos x to be 0, they must have equal absolute values
3
but di erent signs. Among the choices, only 4 satis es these properties,
and it is the only solution of sin x + cos x = 0.

This equations is one of the Double-Angle Identities for Cosine. This means
that all numbers in the domain of the equation are its solutions. Because
the domain of the given equation is R, all numbers in the given set are
2 2
solutions of cos x = cos 2x + sin x.
We substitute each number in the choices to the expression on the left-side
of the equation, and select those numbers that give resulting values
equal to 1.

x = 0: sin 0 + cos 2(0) = 0 + 1 = 1


x = : sin 6 + cos 2( 6 ) = 12 + 1 = 1
6 2

x= 4
: sin 4
+ cos 2( 4 ) = p2
2
+0= p2
2

x= : sin + cos 2( ) = p3 1 = p 31
3 3 3 2 2 2
x= : sin + cos 2( ) = 1 1=0
2 2 2
x= 2
3
: sin 2
3
+ cos 2( 2
3
) = p23 1
2
= p 31
2

x= 3 : sin 3 + cos 2( 3 ) =p 2 + 0 = p2
4 4 4 2 2
5 5 5 1 1
x = 6 : sin 6 + cos 2( 6 ) = 2 + 2 =1
x = : sin + cos 2 = 0 + 1 = 1
x = 2 : sin 2 + cos 2(2 ) = 0 + 1 = 1

222
All rights reserved. No part of this material may be reproduced or transmitted in any form or by any means -
electronic or mechanical including photocopying – without written permission from the DepEd Central Office. First Edition, 2016.
From these values, the only solution of sin x + cos 2x = 0 among the
choices is 2 .
We again substitute the numbers in the given set one by one, and see which
resulting values are equal to 1.

x = 0: 2 sin 0 + tan 0 2 cos 0 = 2


32 p 3
x= 6
: 2 sin 6 + tan 6
2 cos 6 = 3
x= 4
: 2 sin 4 + tan 4
2 cos 4 = 1
p

x= 3: 2 sin 3 + tan 3 2 cos 3 = 2 3 1


x= : Since tan is unde ned, this value of x cannot be a solution of the
2 2

equation.
2 2 2 2
x= 3
: 2 sin 3
+ tan 3
2 cos 3
=1
3 3 3 3 p
x= 4 : 2 sin 4 + tan 4 2 cos 4 =2 2 1
p
x= 5 : 2 sin 5 + tan 5 2 cos 5 = 3+2 3
6 6 6 6 3
x = : 2 sin + tan 2 cos = 2
x = 2 : 2 sin 2 + tan 2 2 cos 2 = 2

Thus, the only solution of 2 sin x + tan x 2 cos x = 2 from the given set is .

This equation has no solution because one of the Pythagorean Identities says
2 2
sin x + cos x = 1.
We substitute each number in the given set to the expression of each side of
the equation, and see which resulting values are equal.

x = 0: sin 2(0) = 0; sin 0 = 0


x= : sin 2( ) = p 3; sin =1
6 6 2 6 2
x= : sin 2( ) = 1; sin = p2
4 4 4 2
p3 p 3
x= : sin 2( ) = ; sin =
3 3 2 3 2
x= : sin 2( 2 ) = 0; sin =1
2 2

x= 3 : sin 2( ) = 1; sin
3 3 =p 2
4 4 4 2
x= 5 : sin 2( ) = 5 p 3; sin =1
6 6 2 3 2
x= : sin 2 = 0; sin =0
x = 2 : sin 2(2 ) = 0; sin 2 =0

Thus, among the numbers in the given set, the solutions of sin 2x = sin x
are 0, 3 , , and 2 .

223
All rights reserved. No part of this material may be reproduced or transmitted in any form or by any means -
electronic or mechanical including photocopying – without written permission from the DepEd Central Office. First Edition, 2016.
(12) We employ the same technique used in the previous item.

x=0: 2 tan 0 + 4 sin 0 = 0


2 + sec 0 = 3
p
2 3+6
x=6: 2 tan 6 + 4 sin 6 = 3
p
= 2 3+6
2 + sec
6 3
p
x=4: 2 tan 4 + 4 sin 4 =2 2+2
p
2 + sec = 2+2
4
p
x=3: 2 tan 3 + 4 sin 3 =4 3
2 + sec 3 = 4
x= :
2 Both tan 2 and sec 2 are unde ned:
2 2
2 tan 3 + 4 sin 3 =0
2
2 + sec 3 =0
3 3 3 p
x= 4 : 2 tan 4 + 4 sin 4 =2 2 2
3
2 + sec 4 =2 p2
5 5 5 62 p 3
x= 6 : 2 tan 6 + 4 sin 6 = 3
5 62 p
2 + sec 6
= 3
3

x= : 2 tan + 4 sin = 0
2 + sec = 1
x = 2 : 2 tan 2 + 4 sin 2 = 0
2 + sec 2 = 3
After checking the equal values, the solutions of 2 tan x + 4 sin x = 2 + sec x
2 5
among the given choices are 6 , 3
, and 6
. 2

3.8.2. Equations with One Term

From the preceding discussion, you may observe that there may be more
solutions of a given equation outside the given set. We now nd all solutions
of a given equation.
We will start with a group of equations having straightforward techniques
in nding their solutions. These simple techniques involve at least one of the
following ideas:

224
All rights reserved. No part of this material may be reproduced or transmitted in any form or by any means -
electronic or mechanical including photocopying – without written permission from the DepEd Central Office. First Edition, 2016.
equivalent equations (that is, equations that have the same solutions as the
original equation);
periodicity of the trigonometric function involved;
inverse trigonometric function;

values of the trigonometric function involved on the interval [0; ] or [0; 2 ]


(depending on the periodicity of the function); and
Zero-Factor Law: ab = 0 if and only if a = 0 or b = 0.

To \solve an equation" means to nd all solutions of the equation. Here,


unless stated as angles measured in degrees, we mean solutions of the
equation that are real numbers (or equivalently, angles measured in radians).
Example 3.8.2. Solve the equation 2 cos x 1 = 0.

Solution. The given equation is equivalent to

1
cos x = 2 :

On the interval [0; 2 ], there are only two solutions of the last equation, and
5
these are x = 3 (this is in QI) and x = 3 (in QIV).
Because the period of cosine function is 2 , the complete solutions of the
equation are x = + k(2 ) and x = 5 + k(2 ) for all integers k. 2
3 3

In the preceding example, by saying that the \complete solutions are x = 3


5
+ k(2 ) and x = 3 + k(2 ) for all integers k," we mean that any integral value
of k will produce a solution to the given equation. For example, when
k = 3, x = +3(2 ) =
3
19
3
is a solution of the equation. When k = 2,
x= 5 + (2)(2 ) = 7 is another solution of 2 cos x 1=0. The family of
3 3
solutions x = + k(2 ) can be equivalently enumerated as x = 19 + 2k , while
3 3
5 7
the family x = 3 + k(2 ) can also be stated as x = 3 + 2k .
Example 3.8.3. Solve: (1 + cos )(tan 1)=0.

Solution. By the Zero-Factor Law, the given equation is equivalent to

1 + cos = 0 or tan 1=0


cos = 1 tan = 1

= + 2k ; k 2 Z =4 + k ; k 2 Z:
Therefore, the solutions of the equation are = + 2k and = 4
+ k for all
k 2 Z. 2
225
All rights reserved. No part of this material may be reproduced or transmitted in any form or by any means -
electronic or mechanical including photocopying – without written permission from the DepEd Central Office. First Edition, 2016.
Example 3.8.4. Find all values of x in the interval [2 ; 2 ] that satisfy the
equation (sin x 1)(sin x + 1) = 0.

Solution.
sin x 1=0 or sin x + 1 = 0
sin x = 1 sin x = 1
x= or 3 x= 3 or
2 2 2 2
3 3
Solutions: 2
; 2; 2 ; 2 2

Example 3.8.5. Solve: cos x = 0:1.

Solution. There is no special number whose cosine is 0:1. However, because


0:1 2 [1; 1], there is a number whose cosine is 0:1. In fact, in any one-period
interval, with cos x = 0:1 > 0, we expect two solutions: one in QI and another
in QIV. We use the inverse cosine function.
1
From Lesson 3.7, one particular solution of cos x = 0:1 in QI is x = cos
0:1. We can use this solution to get a particular solution in QIV, and this is x
1 1
= 2 cos 0:1, which is equivalent to x = cos 0:1.
From the above particular solutions, we can produce all solutions of cos x =
1 1
0:1, and these are x = cos 0:1+2k and x = cos 0:1+2k for all k 2 Z. 2
Example 3.8.6. Solve: 3 tan + 5 = 0.

Solution.
5
3 tan + 5 = 0 =) tan = 3
We expect only one solution in any one-period interval.
5 1 5

tan = 3 =)= tan 3 +k;k2Z 2


?
Example 3.8.7. The voltage V (in volts) coming from an electricity distribut-
ing company is uctuating according to the function V (t) = 200 + 170 sin(120
t) at time t in seconds.

Determine the rst time it takes to reach 300 volts.

For what values of t does the voltage reach its maximum value?

Solution. (1) We solve for the least positive value of t such that V (t) = 300.

200 + 170 sin(120 t) = 300


100

226
All rights reserved. No part of this material may be reproduced or transmitted in any form or by any means -
electronic or mechanical including photocopying – without written permission from the DepEd Central Office.
First Edition, 2016.
1 100
120 t = sin
170
1 100
sin 170
t= 120 0:00167 seconds

The maximum value of V (t) happens when and only when the maximum
value of sin(120 t) is reached. We know that the maximum value of
sin(120 t) is 1, and it follows that the maximum value of V (t) is 370 volts.
Thus, we need to solve for all values of t such that sin(120 t) = 1.

sin(120 t) = 1
120 t = + 2k ; k nonnegative integer
2
+ 2k
t =2
120
1 + 2k
2
t = 120 0:00417 + 0:017k
This means that the voltage is maximum when t 0:00417 + 0:017k for each
nonnegative integer k. 2

3.8.3. Equations with Two or More Terms

We will now consider a group of equations having multi-step techniques of


nding their solutions. Coupled with the straightforward techniques we learned
in the preceding discussion, these more advanced techniques involve
factoring of expres-sions and trigonometric identities. The primary goal is to
reduce a given equation into equivalent one-term equations.
Example 3.8.8. Solve: 2 cos x tan x = 2 cos x.

Solution.

2 cos x tan x = 2 cos x


2 cos x tan x 2 cos x = 0
(2 cos x)(tan x 1)=0

2 cos x = 0 or tan x 1=0


cos x = 0 tan x = 1
x= + 2k or x =4 + k ,
2
3
x= 2 + 2k , k2Z
k2Z
3
Solutions: 2 + 2k ; 2 + 2k ; 4 +k;k2Z 2
227
All rights reserved. No part of this material may be reproduced or transmitted in any form or by any means -
electronic or mechanical including photocopying – without written permission from the DepEd Central Office. First Edition, 2016.
Example 3.8.9. Solve for x 2 [0; 2 ): sin 2x = sin x.

Solution.

sin 2x = sin x
sin 2x sin x = 0
2 sin x cos x sin x = 0 Sine Double-Angle Identity
(sin x)(2 cos x 1)=0

sin x = 0 or 2 cos x 1=0


x = 0 or x = cos x = 1
2
x= or x = 5
3 3
Solutions: 0; ; ; 5 2
3 3

Tips in Solving Trigonometric Equations

If the equation contains only one trigonometric term, isolate that


term, and solve for the variable.

If the equation is quadratic in form, we may use factoring, nding


square roots, or the quadratic formula.

Rewrite the equation to have 0 on one side, and then factor (if
appropriate) the expression on the other side.

If the equation contains more than one trigonometric function, try


to express everything in terms of one trigonometric function.
Here, identities are useful.

If half or multiple angles are present, express them in terms of a


trigonometric expression of a single angle, except when all
angles involved have the same multiplicity wherein, in this
case, retain the angle. Half-angle and double-angle identities
are useful in simpli cation.

2
Example 3.8.10. Solve for x 2 [0; 2 ): 2 cos x = 1 + sin x.

Solution.
2
2 cos x = 1 + sin x
2
2(1 sin x) = 1 + sin x Pythagorean Identity

228
All rights reserved. No part of this material may be reproduced or transmitted in any form or by any means -
electronic or mechanical including photocopying – without written permission from the DepEd Central Office. First Edition, 2016.
2
2 sin x + sin x 1 = 0
(2 sin x 1)(sin x + 1) = 0 Factoring

2 sin x 1=0 or sin x + 1 = 0


sin x = 1 sin x = 1
2
x= or x = 5 x= 3
6 6 2

Solutions: 6 ; 56 ; 3
2 2
2
Example 3.8.11. Solve for x 2 [0; 2 ) in the equation 3 cos x + 2 sin x = 2.

Solution.
2
3 cos x + 2 sin x = 2
2
3(1 sin x) + 2 sin x = 2 Pythagorean Identity
(3 sin x + 1)(sin x 1)=0 Factoring

3 sin x + 1 = 0 or sin x 1=0


1
sin x = 3 sin x = 1
1 1
x = sin ( 3 )+2 x= 2
or
1
x= sin ( 13 )
1 1
Solutions: 2 sin ( 1 )+; + sin ( 1 ); 2
3 3 2

One part of the last solution needs further explanation. In the equation
sin x = 13 , we expect two solutions in the interval [0; 2 ): one in ( ; 3 ) (which 2

is QIII), and another in ( 23 ; 2 ) (which is QIV). Since no special number satis es


1 1
sin x = 3 , we use inverse sine function. Because the range of sin is [ 2 ; 2 ], we
1
know that 2
< sin ( ) < 0. From this value, to get the solution in (
1
3
3 ;2 ),
2
1 1
we simply add 2 to this value, resulting to x = sin ( 3 ) + 2 . On the other
1
hand, to get the solution in ( ; 3 ), we simply add sin ( 1 ) to , resulting to
1 2 3
x= sin ( 31 ).
2 2x
Example 3.8.12. Solve: sin x + 5 cos 2 = 2.

Solution.

sin2 x + 5 cos2 x2 = 2
2 1+cos x
sin x + 5 2 = 2 Cosine Half-Angle Identity
2
2 sin x + 5 cos x + 1 = 0

229
All rights reserved. No part of this material may be reproduced or transmitted in any form or by any means -
electronic or mechanical including photocopying – without written permission from the DepEd Central Office. First Edition, 2016.
2
2(1 cos x) + 5 cos x + 1 = 0 Pythagorean Identity
2
2 cos x 5 cos x 3=0
(2 cos x + 1)(cos x 3)=0

2 cos x + 1 = 0 or cos x 3 = 0
1
cos x = 2
cos x = 3
2
x= 3 + 2k or no solution
4
= 3 + 2k , k
2Z
2 4
Solutions: 3 + 2k ; 3 + 2k ; k 2 Z 2
Example 3.8.13. Solve for x 2 [0; 2 ) in the equation tan 2x 2 cos x = 0.
Solution.

tan 2x 2 cos x = 0
sin 2x
2 cos x = 0
cos 2x
sin 2x 2 cos x cos 2x = 0

Apply the Double-Angle Identities for Sine and Cosine, and then factor.
2
2 sin x cos x 2(cos x)(1 2 sin x) = 0
2
(2 cos x)(2 sin x + sin x 1)=0
(2 cos x)(2 sin x 1)(sin x + 1) = 0

2 cos x = 0 or 2 sin x 1=0 or sin x + 1 = 0


cos x = 0 sin x = 1 sin x = 1
2
x= or x = or x= 3
2 6 2
3
x= 2 x = 56
These values of x should be checked in the original equation because tan 2x may not be de ned.
Upon checking, this is not the case for each value of x obtained.
3
The solutions are 2
, 2
, 6 , 65 , and 3
2
. 2
?
Example 3.8.14. A weight is suspended from a spring and vibrating
vertically according to the equation
4 5

f(t) = 20 cos 5 t 6 ;
where f(t) centimeters is the directed distance of the weight from its central
position at t seconds, and the positive distance means above its central position.

230
All rights reserved. No part of this material may be reproduced or transmitted in any form or by any means -
electronic or mechanical including photocopying – without written permission from the DepEd Central Office. First Edition, 2016.
At what time is the displacement of the weight 5 cm below its central
position for the rst time?

For what values of t does the weight reach its farthest point below its
central position?

Solution. (1) We nd the least positive value of t such that f(t) = 5.


20 cos 5
4 t 6
5 =5
6
cos t = 4

5
4 5 1

this equation.
There are two families of solutions for
4 5 1 1
5 5
t 6 = cos )
4 + 2k , k 2 Z
t= + ( 4
1 1

cos +2k
6 4
5
In this family of solutions, the least positive value of t happens when
k = 0, and this is
1
5 cos 5
1
+ 2(0)
t= +
4
4
6 1:5589:
4 5 1 1
5 t 6 =2 cos 4 + 2k , k 2 Z
4 )+2k
1
t= 5 + 2 cos 1 (
6 4
5
Here, the least positive value of t happens when k = 1, and this is
1
t=
5
6 +
2 cos 1
5
4
+ 2(1) 0:1078:
4

Therefore, the rst time that the displacement of the weight is 5 cm below
its central position is at about 0:1078 seconds.

(2) The minimum value of f(t) happens when and only when the minimum value
of cos 45 t 56 is reached. The minimum value of cos 45 t 56 is 1, which implies that
the farthest point the weight can reach below its central position is 20 cm. Thus, we need to
solve for all values of t such that
4 5
t 1.
cos 5 6 4 5
= cos 5 t 6 =1
4 5 1
5 t 6 = cos (1) + 2k ; k 0
4 5
5 t 6 = + 2k
5 +2k 25 5
t= 6 + 4 = 12 + 2 k
5

Therefore, the weight reaches its farthest point (which is 20 cm) below its
25 5
central position at t = 12 + 2 k for every integer k 0. 2
231
All rights reserved. No part of this material may be reproduced or transmitted in any form or by any means -
electronic or mechanical including photocopying – without written permission from the DepEd Central Office. First Edition, 2016.
More Solved Examples

4
p !2

1. Give a particular solution of the following equation.


2 2
(a) sin x 1=0 (d) csc x csc x 2=0
p 2 2
(b) cot x = 3 (e) cos 2x = sin x
(c) sec 3x = 1 (f) 2 cos x 3 = 0
Solution:
2
(a) x = 2
is a solution because sin ( 2
) 1=(1) 2 1=0.
(b) Note that cos =p 3 and sin = 1 . Thus, cot = p 3. So, x = is a
6 2 6 2 6 6
solution.
(c) Since sec = 1 if and only if cos = 1, a particular solution of the
equation in 3x is , that is, 3x = . Hence, x = is a solution.
3
3 23 23
(d) Note that csc 2 = 1. So, csc 2 = 1. As a consequence, csc 2
3
csc 2 2 = 1 (1) 2 = 0.
(e) x = 2 is a solution.
0 Because cos x must not be more than 1, then the equation has no solution.
2 2
What is the solution set of the following trigonometric equation sin x + cos x
= 1?
Solution: The equation is the Pythagorean Identity, meaning any element of the domain of sin x
and cos x satis es the equation. The domain of both sin x and cos x is R. Therefore, the solution
set of this trigonometric equation is R.
One may try the numbers 6
; 0; and 4 for illustration.

(a) x = 6
2 2
2 2 2
sin x + cos x = sin = 2
6
+
+ cos2 2
2 6 = sin 6 !
=4+4=1:
+ cos 6
1 2 p3 1 3

(b) x = 0
2 2 2 2 2 2
sin x + cos x = sin 0 + cos 0 = 0 + 1 = 0 + 1 = 1:

(c) x = 4
sin2 x + cos2 x = sin2 4 + cos2

p !2
2 2 1
= + = + 1 = 1:
2 2 4 4

232
All rights reserved. No part of this material may be reproduced or transmitted in any form or by any means -
electronic or mechanical including photocopying – without written permission from the DepEd Central Office. First Edition, 2016.
2 2
Find the solution set of the trigonometric equation tan x + 1 = sec x:
Solution: Notice that this is a fundamental identity. Thus, the solution of
this equation is any number common to the domain of the tangent and
secant function. That is, the solution set is

3 5
fx 2 R j cos x 6= g0 = fx 2 R j x 6= 2 ; 2; 2; :::g
(2k+1)

= fx 2 R j x 6= 2 ; k 2 Zg:
p
4. Find all solutions of 3 tan x + 1 = 0.
1
Solution: The equation is equivalent to tan x = p 3. This is true only if
5
x = 6 + k where k 2 Z.
p
5. What are the solutions of 3 tan x + 1 = 0, where x 2 [0; 2 ].
Solution: The solutions are x = 5 and x = 11 .
6 6
2
6. Determine all solutions of 4 cos x 1=0.
Solution: Note that the equation is quadratic in form, so we can apply tech-niques in solving
quadratic equations. For this case, we factor the expression on the left and obtain, (2 cos x 1)(2
cos x + 1) = 0. Consequently, we have cos x = 1=2 or cos x = 1=2. The rst equation have solutions
of the form ( =3 + 2k ) or (5 =3 + 2k ) where k 2 Z, while the second equation have solutions of the
form (2 =3 + 2k ) or (4 =3 + 2k ). Combining the two solu-tions, one observes that the solution set
of the original equation may be given
by ;2 4 5 7
3 3 ; 3 ; 3 ; 3 ;::: :

We can write this in a more compact form as

k
3 : k 6= j;3 where j 2 Z :
2
7. Find the solutions of 4 cos x 1 = 0 within the closed interval [0; 2 ].
Solution: Similar to Example 6, the solution of the above equation is

k
3 : k 6= j;3 where j 2 Z :

Since we are to nd solutions in [0; 2 ], we take k = 1; 2; 4; and 5 to obtain


the solutions =3; 2 =3; 4 =3; and 5 =3.

233
All rights reserved. No part of this material may be reproduced or transmitted in any form or by any means -
electronic or mechanical including photocopying – without written permission from the DepEd Central Office.
First Edition, 2016.
2
8. If x 2 [0; 2 ), solve the equation 2 sin x = p 3 sin x.
2
Solution: First, we write the equation as 2 sin x p 3 sin x = 0. Then, we
factor out sin x and get
p p
sin x(2 sin x 3) = 0 =) sin x = 0 or sin x = 3=2:
The rst of these equations has solutions x = 0 and x = , while the second
has solutions x = =3 and 2 =3. The solutions of the original equation is the
2
union of the two, i.e., the solution set is f0; ; 3 ; 3 g.
2
9. Solve 2 cos x + 5 cos x3 = 0, where x 2 [0; 2 ).
Solution: By factoring the left hand side of the given equation, we get (2
cos x 1)(cos x + 3) = 0. This gives us two equations, namely

cos x = 1 and cos x = 3:


2
First, we remark that the second equation does not have a solution because
cos x should be more than or equal to -1. Hence, the solution of the rst
equation is the solution of the original equation. Thus, the solution set is
5
f3 ; 3 g.
10. Determine the solution set of the equation cos 2x = sin x on [0; 2 ).
Solution: Combining the equation cos 2x = sin x with the cosine double-
2
angle identity cos 2x = 1 2 sin x, we get
2
sin x = 1 2 sin x:

This is equivalent to

2
2 sin x + sin x 1 = 0 =) (2 sin x 1)(sin x + 1) = 0 =) sin

x = 1=2 and sin x = 1:

The solutions of the rst equation is x = =6 and x = 5 =6. The number x = 3


=2 is the solution of the second equation. Therefore, the solution set of the
5 3
original equation is f6 ; 6 ; 2 g.
Solve cos x = cos 2x, for x 2 [0; 2 ). Solution:
cos x = cos 2x
2
) cos x = 2 cos x 1
2
) 0 = 2 cos x cos x 1
) 0 = (2 cos x + 1)(cos x 1):
234
All rights reserved. No part of this material may be reproduced or transmitted in any form or by any means -
electronic or mechanical including photocopying – without written permission from the DepEd Central Office. First Edition, 2016.
The given trigonometric equation is equivalent to solving 2 cos x + 1 = 0 and
cos x = 1. For 2 cos x + 1 = 0 which is the same as cos x = 1=2, the solutions
in the given interval are x = 2 =3; 4 =3. For cos x = 1, the solution is x = 0.
2 4
Therefore, the solution set of the original equation is f0; 3 ; 3 g.
A lighthouse at sea level is 34 mi from a boat. It is known that the top of the
lighthouse is 42:5 mi from the boat and that x = r cos , where x is the
horizontal distance, r is the distance of the top of the lighthouse from the
boat, and is the angle of depression from the top of lighthouse. Find .
Solution:
x 34 4
x = r cos =) cos = r = 42 :5 = 5

1 4
=) = cos 5 0:6435 (or 36:87 ):
For this case, we used a calculator to nd the value of the unknown variable
4
since 5 is not a special value for cosine.
Three cities, A; B; and C, are positioned in a triangle as seen in the gure
below.

It is known that City A is 140pmi from City C, while City B is 210 mi from City
C. Cities A and B are 70 7 mi apart. Also, by the Cosine Law, we have
2 2 2
z =x +y 2xy cos
where x; y; and z are the respective distances of BC; AC, AB, and =
m\ACB. Find .
Solution: Substituting the corresponding values of x; y; and z, the problem
is now equivalent to solving the equation
34300 = 44100 + 19600 58800 cos

) 29400 = 58800 cos


1
) 2 = cos

) 3 = :
235
All rights reserved. No part of this material may be reproduced or transmitted in any form or by any means -
electronic or mechanical including photocopying – without written permission from the DepEd Central Office. First Edition, 2016.
Supplementary Problems 3.8
1. Find all solutions of the equation 2 cos x cos x sin x = 0.
2
2. Determine the solution set of the equation csc x + 1 = 0.
2
3. What are the solutions of sec x + sec x 2=0.
2
4. Find the solutions of the equation 4 sin x 1 = 0 on [0; 2 ).
p

5. Find the values of x 2 [0; 2 ) for which csc 2x = 2.


What is the solution set of the equation sin = csc ?
1
Solve t = sin (cos 2t).
2 2
Let x 2 [0; 2 ). Find the solutions of the equation cos 4x + sin 2x = 1.
If a projectile, such as a bullet, is red into the air with an initial velocity v at an
angle of elevation , then the height h of the projectile at time t is given by
2
h(t) = 16t + vt cos meters. If the initial velocity is 109 meters per second,
at what angle should the bullet be red so that its height is 45 meters above
the oor in 2 seconds.

In a baseball eld, a pitcher throws the ball at a speed of 60 km=h to the


catcher who is 100 m away. When the ball leaves a starting point at an
angle of elevation of , the horizontal distance the ball travels is determined
v
by d = 322 sin , where d is measured in meters and velocity in kilometers
per hour. At what angle of elevation (in degrees) is the ball thrown?

Lesson 3.9. Polar Coordinate System

Learning Outcomes of the Lesson


At the end of the lesson, the student is able to:
locate points in polar coordinate system;
convert the coordinates of a point from rectangular to polar system and vice
versa; and
solve situational problems involving polar coordinate system.

236
All rights reserved. No part of this material may be reproduced or transmitted in any form or by any means -
electronic or mechanical including photocopying – without written permission from the DepEd Central Office. First Edition, 2016.
Lesson Outline
Polar coordinate system: pole and polar axis
Polar coordinates of a point and its location
Conversion from polar to rectangular coordinates, and vice versa
Simple graphs and applications

Introduction
Two-dimensional coordinate systems are used to describe a point in a
plane. We previously used the Cartesian or rectangular coordinate system to
locate a point in the plane. That point is denoted by (x; y), where x is the
signed dis-tance of the point from the y-axis, and y is the signed distance of
the point from the x-axis. We sketched the graphs of equations (lines, circles,
parabolas, ellipses, and hyperbolas) and functions (polynomial, rational,
exponential, log-arithmic, trigonometric, and inverse trigonometric) in the
Cartesian coordinate plane. However, it is often convenient to locate a point
based on its distance from a xed point and its angle with respect to a xed
ray. Not all equations can be graphed easily using Cartesian coordinates. In
this lesson, we also use another coordinate system, which can be presented
in dartboard-like plane as shown below.

3.9.1. Polar Coordinates of a Point

We now introduce the polar coordinate system. It is composed of a xed point


called the pole (which is the origin in the Cartesian coordinate system) and a

237
All rights reserved. No part of this material may be reproduced or transmitted in any form or by any means -
electronic or mechanical including photocopying – without written permission from the DepEd Central Office. First Edition, 2016.
xed ray called the polar axis (which is the nonnegative x-axis).

In the polar coordinate system, a point is described by the ordered pair (r;
). The radial coordinate r refers to the directed distance of the point from the
pole. The angular coordinate refers to a directed angle (usually in radians)
from the polar axis to the segment joining the point and the pole.

Because a point in polar coordinate system is described by an order pair of


radial coordinate and angular coordinate, it will be more convenient to geomet-
rically present the system in a polar plane, which serves just like the Cartesian
plane. In the polar plane shown below, instead of rectangular grids in the Carte-
sian plane, we have concentric circles with common center at the pole to identify
easily the distance from the pole (radial coordinate) and angular rays emanating
from the pole to show the angles from the polar axis (angular coordinate).

238
All rights reserved. No part of this material may be reproduced or transmitted in any form or by any means -
electronic or mechanical including photocopying – without written permission from the DepEd Central Office. First Edition, 2016.
Example 3.9.1. Plot the following points in one polar plane: A(3; ), B(1; 56 ), 3

C(2; 7 ), D(4; 19 ), E(3; ), F (4; 7 ), G(2:5; 17 ), H(4; 17 ), and I(3; 5 ).


6 12 6 4 6 3

Solution.

As seen in the last example, unlike in Cartesian plane where a point has
a unique Cartesian coordinate representation, a point in polar plane have in
nitely many polar coordinate representations. For example, the coordinates
(3; 4) in the Cartesian plane refer to exactly one point in the plane, and this
particular point has no rectangular coordinate representations other than (3;
4). However, the coordinates (3; 3 ) in the polar plane also refer to exactly
one point, but this point has other polar coordinate representations. For
5 7 13 19
example, the polar coordinates (3; 3 ), (3; 3 ), (3; 3 ), and (3; 3 ) all refer
to the same point as that of (3; 3 ).

The polar coordinates (r; + 2k ), where k 2 Z, represent the same


point as that of (r; ).

In polar coordinate system, it is possible for the coordinates (r; ) to have a


negative value of r. In this case, the point is jrj units from the pole in the
opposite direction of the terminal side of , as shown in Figure 3.41.

239
All rights reserved. No part of this material may be reproduced or transmitted in any form or by any means -
electronic or mechanical including photocopying – without written permission from the DepEd Central Office. First Edition, 2016.
Figure 3.41

4 11
Example 3.9.2. Plot the following points in one polar plane: A(3; 3
), B(4; 6
),
7
C(2;), and D(3:5; 4 ).

Solution. As described above, a polar point with negative radial coordinate


lies on the opposite ray of the terminal side of .

240
All rights reserved. No part of this material may be reproduced or transmitted in any form or by any means -
electronic or mechanical including photocopying – without written permission from the DepEd Central Office. First Edition, 2016.
Points in Polar Coordinates

For any , the polar coordinates (0; ) represent the pole.


A point with polar coordinates (r; ) can also be represented by (r; +
2k ) or (r; + + 2k ) for any integer k.

3.9.2. From Polar to Rectangular, and Vice Versa

We now have two ways to describe points on a plane { whether to use the
Carte-sian coordinates (x; y) or the polar coordinates (r; ). We now derive the
conver-sion from one of these coordinate systems to the other.
We superimpose the Cartesian and polar planes, as shown in the
following diagram.

Figure 3.42

Suppose a point P is represented by the polar coordinates (r; ). From


Lesson 3.2 (in particular, the boxed de nition on page 139), we know that
x = r cos and y = r sin :

Conversion from Polar to Rectangular Coordinates


8
x = r cos
(r; ) ! ! (x; y)
y = r sin

Given one polar coordinate representation (r; ), there is only one


rectangular coordinate representation (x; y) corresponding to it.

241
All rights reserved. No part of this material may be reproduced or transmitted in any form or by any means -
electronic or mechanical including photocopying – without written permission from the DepEd Central Office. First Edition, 2016.
Example 3.9.3. Convert the polar coordinates (5; ) and (3; 6) to Cartesian
coordinates.

Solution.
8
x = 5 cos = 5
(5; )! ! (5; 0)
:
8 y = 5 sin = 03p 3
x = 3 cos 6 = 2
(3; 6) 3 ( 2 ; 2 ) 2
3p 3 3
! < y = 3 sin 6 = 2
:
!

As explained on page 239 (right after Example 3.9.1), we expect that there
are in nitely many polar coordinate representations that correspond to just one
given rectangular coordinate representation. Although we can actually determine
all of them, we only need to know one of them and we can choose r 0.
Suppose a point P is represented by the rectangular coordinates (x; y).
Re-ferring back to Figure 3.42, the equation of the circle is
p
2 2 2 2 2
x +y =r =) r= x +y :

We now determine . If x = y = 0, then r = 0 and the point is the pole. The


pole has coordinates (0; ), where is any real number.
3
If x = 0 and y 6= 0, then we may choose to be either 2 or 2 (or their
equivalents) depending on whether y > 0 or y < 0, respectively.
Now, suppose x 6= 0. From the boxed de nition again on page 139, we
know that
y
tan =x ;

where is an angle in standard position whose terminal side passes through


the point (x; y).

Conversion from Rectangular to Polar Coordinates


(x; y) = (0; 0) ! (r; ) = (0; ); 2R
8( 2
(0; y) (r; ) = 3

! y; ) if y > 0
y6=0 <( y ;2) if y < 0
: jj

(x; 0)x6=0
! (r; ) = 8 (x; 0)
< (x ; ) if x < 0
if x > 0

: jj

242
All rights reserved. No part of this material may be reproduced or transmitted in any form or by any means -
electronic or mechanical including photocopying – without written permission from the DepEd Central Office. First Edition, 2016.
(x; y) ! (r; )
x6=0; 6y=0
p

r = x2 + y 2
y
tan =
x

same quadrant as (x; y)

Given one rectangular coordinate representation (x; y), there


are many polar coordinate representations (r; ) corresponding to it.
The above computations just give one of them.

Example 3.9.4. Convert each Cartesian coordinates to polar coordinates (r; ),


where r 0.
(1) (4; 0) (4) (6; 2)
(2) (4; 4) (5) (3; 6)
(3) (3; p 3) (6) (12; 8)

Solution. (1) (4; 0) ! (4; )


(2) The point (4; 4) is in QI.
tan 2p 2 p 2 2 p
r= x+y= 4+4=4 2
= =

= y =4 = 1 ) x 4 4
p
(4; 4) ! 4 2; 4

(3) (3; p 3) in QIII


+ p
q (3)p 2
3
p3 p 2 7
r= ( 3) =2 3
tan = 3 = 3 =) = 6
7
p
(3; p 3) ! 2 3; 6
(4) (6; 2) in QIV
2 2 p
r= 6 + (2) = 2 10
= 2 = 1
3
= 1
3
p
= tan
6 )

tan
1
1
(6; 2) ! 2 p10; tan 3
(5) (3; 6) in QII
tan p 2 2 3
p = + tan
r= (3) +6=3 5
= 2 = )

6 1
= (2)

1
(3; 6) ! 3p 5 ; + tan (2)
243
All rights reserved. No part of this material may be reproduced or transmitted in any form or by any means -
electronic or mechanical including photocopying – without written permission from the DepEd Central Office. First Edition, 2016.
(6) (12; 8) in QIII
tan p 12 2 3 2 p 3
r= (12) + (8) =4 13
)

1
= 8
=2 = = + tan 2

p 1
(12; 8) ! 4 13; + tan 3 2
3.9.3. Basic Polar Graphs and Applications

From the preceding session, we learned how to convert polar coordinates of


a point to rectangular and vice versa using the following conversion formulas:
2 2 2 y
r = x + y ; tan = x ; x = r cos ; and y = r sin :
Because a graph is composed of points, we can identify the graphs of some
equa-tions in terms of r and .

Graph of a Polar Equation


The polar graph of an equation involving r and is the set of all
points with polar coordinates (r; ) that satisfy the equation.

As a quick illustration, the polar graph of the equation r = 2 2 sin consists


of all points (r; ) that satisfy the equation. Some of these points are (2; 0), (1;
3
6 ), (0; 2 ), (2; ), and (4; 2 ).
Example 3.9.5. Identify the polar graph of r = 2, and sketch its graph in the
polar plane.
2 2 2 2 2
Solution. Squaring the equation, we get r = 4. Because r = x + y , we have x
2
+ y = 4, which is a circle of radius 2 and with center at the origin. Therefore, the
graph of r = 2 is a circle of radius 2 with center at the pole, as shown below.

244
All rights reserved. No part of this material may be reproduced or transmitted in any form or by any means -
electronic or mechanical including photocopying – without written permission from the DepEd Central Office. First Edition, 2016.
2 2
In the previous example, instead of using the conversion formula r = x +
2
y , we may also identify the graph of r = 2 by observing that its graph
consists of points (2; ) for all . In other words, the graph consists of all points
with radial distance 2 from the pole as rotates around the polar plane.
Therefore, the graph of r = 2 is indeed a circle of radius 2 as shown.
5
Example 3.9.6. Identify and sketch the polar graph of = 4 .
5 5 2
Solution. The graph of 5 = 4 consists of all points (r; 4 ) for r 5 R. If
r > 0, then points (r; 4 ) determine a ray from the pole with angle 4 from
5
the polar axis. If r = 0, then (0; 4 ) is the pole. If r < 0, then the points
5
(r; 4 ) determine a ray in opposite direction to that of r > 0. Therefore, the
5 5
graph of = 4 is a line passing through the pole and with angle 4 with
respect to the polar axis, as shown below.

Example 3.9.7. Identify (and describe) the graph of the equation r = 4 sin .

Solution.

r = 4 sin
2
r = 4r sin
2 2
x + y = 4y
2 2
x +y 4y = 0
2 2
x + (y 2) = 4

Therefore, the graph of r = 4 sin is a circle of radius 2 and with center at (2; 2 ).

245
All rights reserved. No part of this material may be reproduced or transmitted in any form or by any means -
electronic or mechanical including photocopying – without written permission from the DepEd Central Office. First Edition, 2016.
?
Example 3.9.8. Sketch the graph of r = 2 2 sin .
Solution. We construct a table of values.

2 3 5
x 0 6 4 3 2 3 4 6

r 2 1 0:59 0:27 0 0:27 0:59 1 2


7 5 4 3 5 7 11
x 6 4 3 2 3 4 6
2
r 3 3:41 3:73 4 3:73 3:41 3 2

This heart-shaped curve is called a cardioid. 2

246
All rights reserved. No part of this material may be reproduced or transmitted in any form or by any means -
electronic or mechanical including photocopying – without written permission from the DepEd Central Office. First Edition, 2016.
?
Example 3.9.9. The sound-pickup capability of a certain brand of
microphone is described by the polar equation r = 4 cos , where jrj gives the
sensitivity of the microphone to a sound coming from an angle (in radians).

Identify and sketch the graph of the polar equation.


Sound coming from what angle 2 [0; ] is the microphone most sensitive to?
Least sensitive?

Solution. (1) r = 4 cos


2
r = 4r cos
2 2
x + y = 4x
2 2
x + 4x + y = 0
2 2
(x + 2) + y = 4
This is a circle of radius 2 and with center at (2; ).

(2) We construct a table of values.

2 3 5
x 0 6 4 3 2 3 4 6

r 4 3:46 2:83 2 0 2 2:83 3:46 4

From the table, the microphone is most sensitive to sounds coming from
angles = 0 and = , and least sensitive to sound coming from an angle
= . 2
2

247
All rights reserved. No part of this material may be reproduced or transmitted in any form or by any means -
electronic or mechanical including photocopying – without written permission from the DepEd Central Office. First Edition, 2016.
More Solved Examples
Locate in the polar plane the following polar points: M(1; =3); A(0; ); T ( ; 0),
and H(4; 5 =3).
Solution:

2. Locate in the polar plane the following polar points: W (1; 7 =4); X(2; =6); Y (4; 5 =6)
and Z(3; 11 =3).

Solution:

248
All rights reserved. No part of this material may be reproduced or transmitted in any form or by any means -
electronic or mechanical including photocopying – without written permission from the DepEd Central Office. First Edition, 2016.
Convert the following polar points to Cartesian coordinates.

0 (5; 5 =4) to Cartesian coordinates


1 (2; 3 =4) to Cartesian coordinates
2 (; )
3 (0; 10)

Solution: (a) Using the conversion formulas with r = 5 and = 5 =4, we get p

x = r cos = 5 cos(5 =4) = 5 2=2

and
y = r sin = 5 sin(5 =4) = 5 p 2=2:
Therefore, (5; 5 =4) in Cartesian coordinate is(5 p 2=2; 5 p 2=2).
(b) Using the conversion formulas with r = 2 and = 5 = 3 =4, we get

x = r cos = 2 cos(3 =4) = 2 p 2=2

and
y = r sin = 2 sin(3 =4) = 2 p 2=2:
p p
Therefore, (2; 3 =4) in Cartesian coordinate is (2 2=2; 2 2=2).
Notice here that is used in two di erent ways. First is , with numerical
value approximately equal to 3:14, is used as a radius and second, as an
angle equivalent to 180 . That is, the point is in the negative x-axis units
away from the origin. Hence, the Cartesian coordinate of ( ; ) is ( ; 0).
(d) Since the radius is 0, then the polar point (0; 10) is the origin with
Cartesian coordinate (0; 0).

Convert the following Cartesian points to polar coordinates.


(a) (5; 5) (c) (5 p 3; 15)
(b) (3; p 3) (d) (8; 0)

Solution: (a) The point (x; y) = (5; 5) is in the fourth quadrant. Using the
conversion formulas, we get

p 2 2
p 2
p
r= x +y = 5 + (5) 2 =5 2
and
1 1
= tan (y=x) = tan (1) = =4:
Therefore, (5; 5) in polar coordinate is (5 p 2; =4).

249
All rights reserved. No part of this material may be reproduced or transmitted in any form or by any means -
electronic or mechanical including photocopying – without written permission from the DepEd Central Office.
First Edition, 2016.
(b) Similarly, we use the conversion formulas to get

q p 2
p p
r= (3) 2 + ( 3) = 12 = 2 3
and
1 p
= tan [ 3=(3)] = =6:
Note that the point is in the second quadrant so we must use pi + . There-
p p 6

fore, (3; 3) in polar coordinate is (2 3; 5 =6).


(c) The point is in the third quadrant.
q 2
p p
r= (5 p 3) + (15) 2 = 300 = 10 3
and p
tan = 15=(5 3)) =4 =3:
p
Therefore, (5 p 3; 15) in polar coordinate is (10 3 ; 4 =3).
(d) Using the conversion formula, one can show that the point (8; 0) in
polar coordinate is also (8; 0).
Identify (and describe) the graph of the equation r = 4 sin . Using a graphing
software, graph the following equations.
(a) r = 2 sin (c) = 2r
(b) r = 5 (d) r = 2 2 cos

Solution: (a) r = 2 sin is a circle with radius 1 centered at (1; 2 ).


r = 5 is a standard circle with radius 5.
Notice that as increases, the r also increases. The graph of = 2r is a spiral
rotating counter-clockwise from the pole.
(d) The graph of r = 2 2 cos is a cardioid.

(a) (b)

250
All rights reserved. No part of this material may be reproduced or transmitted in any form or by any means -
electronic or mechanical including photocopying – without written permission from the DepEd Central Office. First Edition, 2016.
(c) (d)
A boy is ying a kite with an angle of elevation of 60 from where he stands.
What is the direct distance of the kite from him, if the the kite is 6 ft above
the ground?
Solution: The problem can be illustrated as follows:

Here, r (in ft) is the distance of the kite from the boy and is the angle of depression. To solve for
r, we apply the formula y = r sin . Thus,
p p p
r = y= sin = 6= sin(60 ) = 6=( 3=2) = 12= 3 = 4 3:
Therefore, the kite is 4 p 3 ft away from the boy.

251
All rights reserved. No part of this material may be reproduced or transmitted in any form or by any means -
electronic or mechanical including photocopying – without written permission from the DepEd Central Office. First Edition, 2016.
Supplementary Problems 3.9
1. Give two more pairs of coordinates that describe the same point.

(a) (13; =3) (b) (0; 0) (c) (15; 15 =4)

2. Locate the following points in the polar coordinate plane:


(a) P (3; )
0 Q(3; 7 =4)
1 R(5=2; 5 =2)
2 S(8; 23 =6)
Transform the following to Cartesian coordinates:
0 (3; )
1 (3; 7 =4)
2 (5=2; 5 =2)
3 (8; 23 =6)
Transform the following to polar coordinates:
0 (9; 40)
1 (15; 20)
2 (5=2; 5 =2)
3 (14; 14)
Consider the equation in polar form r = 4 cos 2 .
0 Complete the table

0 =6 =4 =3 =2 2=3 3=4 5=6 7=6 5=4 4=3 3=2


r

1 Plot the points obtained in part (a) in a polar coordinate system.

A helicopter is hovering 800 feet above a road. A truck driver observes the
helicopter at a horizontal distance of 600 feet. Find the angle of elevation
of the helicopter from the truck driver.

252
All rights reserved. No part of this material may be reproduced or transmitted in any form or by any means -
electronic or mechanical including photocopying – without written permission from the DepEd Central Office. First Edition, 2016.
Topic Test 3 for Unit 3

1. Let be an angle in QIII such that cos = 12 . Find the values of the six
13
trigonometric functions of 2 .
2
2. Prove that cot(2x) = cot x 1 .
2 cot x
3. Using half-angle identities to nd the exact values of the following.

(a) tan 15 (b) tan 7:5


4. Find the exact value of the following.
103 40

1
(a) tan cot 6 (b) cos sin1 41
5. Let y 2 [0; 2 ). Find the solutions of the equation
1 2
sin (cos y cos y 1) = =2:

Let 2 [0; 2 ]. Find all the solutions of the equation


2
4 cos sin = 3 sin :

Let r = 2 2 sin . Complete the table and plot the points (r; ) in the same polar
coordinates.

0 =6 =4 =3 =2 2 =3 3 =4 5 =6 7 =6 5 =4 4 =3 3 =2
r

8. Transform the following points from Cartesian to polar coordinates.

(a) (42; 56) (c) (0; 7) (e) (2;2)


(b) (100; 100) (d) (7; 0) (f) (5; 12)

9. Transform the following points from polar to Cartesian coordinates.

(a) (3; =3) (c) (1; ) (e) (2 ; 2 )


(b) (45; 7 =4) (d) (5; 0) (f) (9; 17 =6)

253
All rights reserved. No part of this material may be reproduced or transmitted in any form or by any means -
electronic or mechanical including photocopying – without written permission from the DepEd Central Office. First Edition, 2016.
Topic Test 4 for Unit 3
nd 7
1. Let be an angle in the 2 quadrant such that cos = 25 . Find the
following.

(a) cos(2 ) (b) sin(2 ) (c) tan(2 )

2. Given that cos 48 0:6691. Find the approximate value of the following.
2 2 2
(a) cos 24 (b) sin 24 (c) tan 24
3. Using half-angle identities to nd the exact values of the following.

(a) tan( =12) (b) tan( =24)

1 1 1 3 .
4. Find the exact value of cos cos + cos
7 5

5. Let x 2 [0; 2 ). Find the solutions of the equation


2 p p p
4 sin x + (2 3 2 2) sin x 6=0:

6. Let 2 [0; 2 ]. Find all the solutions of the equation


2
2 sin (2 ) sin(2 ) 1=0:

Let r = 2 + 2 cos . Complete the table and plot the points (r; ) in the same
polar coordinates.

0 =6 =4 =3 =2 2 =3 3 =4 5 =6 7 =6 5 =4 4 =3 3 =2
r

8. Transform the following points from Cartesian to polar coordinates.

(a) (21; 28) (c) (0; 5) (e) ( ; )


(b) (100; 100) (d) (5; 0) (f) (15; 8)

9. Transform the following points from polar to Cartesian coordinates.

(a) (4; =6) (c) (1; ) (e) ( ; )


(b) (100; 5 =4) (d) (5; 0) (f) (15; 8 =3)

254
All rights reserved. No part of this material may be reproduced or transmitted in any form or by any means -
electronic or mechanical including photocopying – without written permission from the DepEd Central Office. First Edition, 2016.
4

Answers to

Odd-Numbered Exercises
in Supplementary Problems
and
All Exercises in Topic Tests

All rights reserved. No part of this material may be reproduced or transmitted in any form or by any means -
electronic or mechanical including photocopying – without written permission from the DepEd Central Office. First Edition, 2016.
Supplementary Problems 1.1 (page 17)

1
1. center (0; 0), r = 2
3. center 4; 3 , r = 1
4

5. center (7; 6), r = 11

256
All rights reserved. No part of this material may be reproduced or transmitted in any form or by any means -
electronic or mechanical including photocopying – without written permission from the DepEd Central Office. First Edition, 2016.
7. center (2; 4), r = 5
3
5 7 7
9. center ; ,r=
2 2 4

2 2 2 2
11. (x 17) + (y 5) = 144 19. (x + 10) + (y 7) = 36
2 2 2 2
13. (x 15) + (y + 7) = 49 21. (x + 2) + (y 3) = 12
2 2 2 2
15. (x 15) + (y + 7) = 9 23. (x 2:5) + (y 0:5) = 14:5
2 2 2 2
17. (x + 2) + (y 3:5) = 31:25 25. (x + 5) + (y + 1) = 8

Set up a Cartesian coordinate system by assigning C as the origin. Then the


2 2
circle on the left end has radius 100 and has equation x + y = 10000. A
radius of the circle on the right end can be drawn from C to the upper right
corner of the gure; this radius has length (by the Pythagorean theorem)

257
All rights reserved. No part of this material may be reproduced or transmitted in any form or by any means -
electronic or mechanical including photocopying – without written permission from the DepEd Central Office. First Edition, 2016.
p p
2
3002 + 1002 = 100000. Then the circle on the right end has equation x 2 +
y = 100000. We want the length of the segment at y = 50. In this case, the
left endpoint has x = p 10000 502 = p 7500 and the right endpoint has
p p p
x=
p
100000
p
2
50 = p 97500. Then the total length is 97500 ( 7500)

= 50 3+50 39 m 398:85 m.

Supplementary Problems 1.2 (page 31)

1. vertex (0; 0), focus (9; 0), directrix x = 9, axis y = 0

3. vertex ( 1; 7), focus (2; 7), directrix x = 0, axis y = 7

258
All rights reserved. No part of this material may be reproduced or transmitted in any form or by any means -
electronic or mechanical including photocopying – without written permission from the DepEd Central Office. First Edition, 2016.
5. vertex (3; 2), focus 3; 3 , directrix y = 11 , axis x = 3
2 2

2 2
7. (y 11) = 36(x 7) 13. (y 8) = 8(x + 3)
2
9. (x + 10) = 34(y 3) 15. 4:17 cm
2
11. (y 9) = 80(x 4) 17. 3:75 cm

Supplementary Problems 1.3 (page 45)

center: (0; 0)
foci: F1(2; 0), F2(2; 0)
p p
vertices: V1(2 2; 0); V2(2 2; 0)
covertices: W1(0; 2); W 2(0; 2)

3. center: (1; 1)
foci: F1 (1 p 3; 1),

F2 (1 + p 3; 1)
vertices: V1(1; 1); V2(3; 1)
covertices: W1(1; 0); W2(1; 2)

259
All rights reserved. No part of this material may be reproduced or transmitted in any form or by any means -
electronic or mechanical including photocopying – without written permission from the DepEd Central Office. First Edition, 2016.
center: (7; 5)
foci: F1(1; 5),
F2(13; 5)
vertices: V1(3;
5); V 2(17; 5)
covertices: W1(7; 13); W 2(7; 3)

2 2
7. (x 2) + (y 8) =1
49 16

9. The center is (9; 10) and c = 12. We see that the given point (9; 15) is p
2 2
a covertex, so b = 5. Then a = 5 + 12 = 13. Therefore, the equation is
0 (y 10)2 = 1.
16925
(x+9)2
Since the major axis is vertical, the center has the same x coordinate as the
focus and the same y coordinate as the covertex; that is, the center is (9; 10).
2 (x + 9)2
Then c = 5; b = 10, and a = 125. Therefore, the equation is + 100
2
(y 10) = 1.
125
13. Recall that the unit is 100 km. The vertices of the ellipse are at (3633; 0) and
(4055; 0). Then the center of the ellipse is at (211; 0). Then a =
3844
2 (x + 211)2
and c = 211. It follows that b = 14731815. The equation is + 14776336
2
y
= 1.
14731815
Set up a coordinate system with the center of the ellipse at the origin. Then a = 60 and b = 20. We
want the length of the segment with endpoints (on the
2
ellipse) having x = 45 (or 45). The y coordinates are given by 45 2 + y = 1,
60
2 2
20
2
45
q 2
or y = 20 1 60
2
13:23. Hence, the desired width is 26:46 ft.
260
All rights reserved. No part of this material may be reproduced or transmitted in any form or by any means -
electronic or mechanical including photocopying – without written permission from the DepEd Central Office. First Edition, 2016.
Supplementary Problems 1.4 (page 59)

1. center: (0; 0)
p p
foci: F1( 181; 0), F2( 181; 0)
vertices: V1(10; 0), V2(10; 0)
9
asymptotes: y = 10 x

3. center: (0; 5)
p
foci: F1( p 19; 5), F2( 19; 5)
p
vertices: V1( p 15; 5), V2( 15; 5)
2

asymptotes: y 5 =p 15 x

5. center: (3; 3)
foci: F1(3; 3 p 15),
F2(3; 3+ p 15)
vertices: V1(3; 3 p 6),
p
V2(3;
3 +p 6 6)
asymptotes: y + 3 = 3 (x + 3)

261
All rights reserved. No part of this material may be reproduced or transmitted in any form or by any means -
electronic or mechanical including photocopying – without written permission from the DepEd Central Office.
First Edition, 2016.
2 2 2 2
7. y (x + 7) = 1 9. (x + 10) (y + 4) = 1
144 145 81 256

11. The intersection (4; 8) of the two asymptotes is the center of the hyperbola.
Then the hyperbola is vertical and c = 13. Since the slopes of the asymptotes
5 a 5
are 12 , we have b = 12 .
2 2 p 2
Since c = 13, we have a + b = 169 and b = 169 a . It follows that
2
a 5 (y 8) (x + 4)2

p 169 a2 = 12 =) a = 5 and b = 12 =) 25 144 = 1:

The midpoint (9; 1) of the two given corners is the center of the hyperbola. Since the transverse axis is
horizontal, a = 7 and b = 2. Therefore, the
(x 9)2 (y 1)2
equation is = 1.
49 4

Supplementary Problems 1.5 (page 66)

1. pair of intersecting lines 5. parabola

3. parabola 7. empty set


2 2
9. The standard equation of the ellipse is (x 5) 2) = 1; so its foci
+ (y
36 100
are (5; 10) and (5; 6) while its vertices are (5 ; 12) and (5; 8). The equations
2 2 2 2
of the circles are (x 5) + (y 10) = 4, (x 5) + (y 10) = 324,
2 2 2 2
(x 5) + (y + 6) = 4, and (x 5) + (y + 6) = 324.
2 2
11. The standard equation of the hyperbola is (y + 5) (x + 9) = 1. Its auxil-
25 25
iary rectangle has corners (14; 0); (4; 0); (4; 10); (14; 10). The equa-
2 2
tion of the circle is (x + 9) + (y + 5) = 50.
13. The equation simpli es to
r+2
2 2
(x + 7) + (y 3) = r 1:
Its graph
r+2
(a) is a circle if r 1 > 0; that is, when r 2 (; 2) [ (1; +1).
r+2
(b) is a point if = 0; that is, when r = 2.
r 1
r+2
(c) is the empty set if r 1 < 0; that is, when r 2 (2; 1).
262
All rights reserved. No part of this material may be reproduced or transmitted in any form or by any means -
electronic or mechanical including photocopying – without written permission from the DepEd Central Office. First Edition, 2016.
Supplementary Problems 1.6 (page 77)

1. (a) (1; 6)

; ;
(b) 3 20 ; 3 20
4 9 4 89

(c) (1; 6); (1; 2)

263
All rights reserved. No part of this material may be reproduced or transmitted in any form or by any means -
electronic or mechanical including photocopying – without written permission from the DepEd Central Office.
First Edition, 2016.
p
(d) 1; 2 ; (1 p 15; 1); (1 + 15; 1)
3

(e) No solution

Let (x; y) be the ordered pair that satis es the conditions. The resulting system of equations is
2
8 x = 2y2 + 8
> x2 + y2 = 5
1
<
> 16

1 1 :1 1 1 1 1 1
Solving yields 2 ; 4 , 2 ;4 , 2 ; 4 , and 2 ; 4 .

264
All rights reserved. No part of this material may be reproduced or transmitted in any form or by any means -
electronic or mechanical including photocopying – without written permission from the DepEd Central Office. First Edition, 2016.
5. We have the system
8
2 2
x + (y 6) = 36
2
x = 4ky;
where the rst equation is a circle above the x-axis, tangent to the x-axis at x =
0, and the second equation is a parabola facing up/down, depending on
.
2
Substituting the second equation in the rst equation yields y +(4k 12)y = 0.
Note that y = 0 is already a root.
We now consider two cases.
If k > 0, the system might have one or two solutions. To ensure that the
solution is unique, we set the discriminant to be nonpositive: 4k 12 0 ) k 3.

If k 0, the system will always have a unique solution.


Thus k 2 (; 0] [ (3; +1).

Topic Test 1 for Unit 1 (page 78)

2 2
1. (a) Since the coe cients of x and y are equal, the graph is a circle, a
point, or the empty set. Completing the squares, we see that the equation
is equivalent to
1 2 3 2

x 2 + y+ 2 = 4:
Hence, the graph is a circle with center (0:5; 1:5) and radius 2.
By inspection, the graph is a parabola. Completing the squares, we see
2
that the equation is equivalent to (x + 2) = 14(y + 4). Hence, the
graph has vertex at (2; 4) and is opening upward.
2 2
Since the coe cients of x and y are of opposite signs, the graph is a
hyperbola or a pair of intersecting lines. Completing the squares, we
see that the equation is equivalent to
2 2
(x 7) (y + 3)
= 1:
4 3
Hence, the graph is a horizontal hyperbola with center at (7; 3).
2 2
Since the coe cients of x and y have the same sign and are unequal,
the graph is an ellipse, a point, or the empty set. Completing the
squares, we see that the equation is equivalent to
2 2
(x 8) y
+ = 0:
2 7
Hence, the graph is the point (8; 0).

265
All rights reserved. No part of this material may be reproduced or transmitted in any form or by any means -
electronic or mechanical including photocopying – without written permission from the DepEd Central Office. First Edition, 2016.
x2 y2
(a) The equation is equivalent to + = 1. This is a vertical ellipse. 7 25

center: (0; 0)

foci: F1(0; 3 p 2), F2(0; 3 p 2)


vertices: V1(0; 5), V2(0; 5)
p
covertices: W1( p 7; 0); W2( 7; 0)

2 2
(b) The equation is equivalent to (y + 4) (x 1) = 1. This is a vertical
64 36
hyperbola.

center: C(1; 4)
foci: F1 (1; 14), F2 (1; 6)
vertices: F1 (1; 12); F 2(1; 6)
4
asymptotes: y + 4 = 3 (x 1)

(a) The parabola opens to the right and has focal distance c = 6. Its equation
2
is (y 3) = 24(x + 1).
0 The intersection (2; 5) of the two asymptotes is the center of the
hyperbola. Then the hyperbola is horizontal and a = 5. Using the slopes
of the asymptotes, we have ab = 124 . It follows that b = 12 and the
2 2
(x + 2) (y + 5)
equation is = 1.
25 144
2 2
4. Multiplying the rst equation by 2, we get 2(x 1) + 2(y + 1) = 10. By
subtracting the second equation from this new equation, we get the
2
equation 2(y + 1) + 8 = 10 y. This has solutions y = 0 and y = 5=2.
When y = 0, the corresponding x values are 3 and 1. When y = 5=2,
p 11
the corresponding x values are 2 + 1. Therefore, the solutions are (1; 0),
+ 1; 2 .
p
5
(3; 0),2

11

266
All rights reserved. No part of this material may be reproduced or transmitted in any form or by any means -
electronic or mechanical including photocopying – without written permission from the DepEd Central Office. First Edition, 2016.
5. Set up a coordinate system by making the center of the door’s base the origin. Then the ellipse
has center (0; 2) with a = 1=2 and b = 0:3; then its is equation
2
(y2)
x
2 + 2
2
= 1.
0:5 0:3
To determine if the cabinet can be pushed through the doorway, we determine the height of the
doorway when x = 0:25 (or 0:25). We solve for y from the
2
equation 0:25
2
+(y2) 2
2 = 1. Solving for the y coordinate, we see that the height
0:5 0:3

is 2:2598 m. Hence, the cabinet cannot be pushed through the doorway.


6. Let (x; y) be the coordinates of the point. This point satis es

2 2
p x + (y + 1) = 2jx 3j:
Manipulating this equation gives us
2 2 2
x + (y + 1) = 4(x 6y + 9)
2 2
3(x 8x + 16) + (y + 1) = 36 48
2 2
3(x 4) + (y + 1) = 12
2 2
(x 4) (y + 1)
= 1:
4 12
Therefore, the point traces a horizontal hyperbola with center at (4; 1).

Topic Test 2 for Unit 1 (page 79)

1. (a) By inspection, the graph is a parabola. Completing the squares, we


2
see that the equation is equivalent to (y 5) = 8(x 5). Hence, the graph
has vertex at (5; 5) and is opening to the left.
2 2
Since the coe cients of x and y are equal, the graph is a circle, a point,
or the empty set. Completing the squares, we see that the equation is
2 2
equivalent to (x + 5) + (y + 9) = 4. Hence, the graph is the empty
set.
Since the coe cients of x2 and y2 have the same sign and are unequal, the graph is an
ellipse, a point, or the empty set. Completing the squares,
2 2
(x + 2) (y 1)
we see that the equation is equivalent to + = 1. Hence,
4 9
the graph is a vertical ellipse with center (2; 1).
2 2
Since the coe cients of x and y are of opposite signs, the graph is a
hyperbola or a pair of intersecting lines. Completing the squares, we
2 2
see that the equation is equivalent to (y 4) (x 6) = 0. Hence,
11 17
the graph is a pair of intersecting lines given by the equations y 4=
11
17 (x 6).

267
All rights reserved. No part of this material may be reproduced or transmitted in any form or by any means -
electronic or mechanical including photocopying – without written permission from the DepEd Central Office.
First Edition, 2016.
2 2
2. (a) The equation is equivalent tox y = 1. This is a horizontal hyperbola.
64 64

center: (0; 0)
foci: F1(8 p 2; 0),
p
F2(8 2; 0)
vertices: V1(8; 0), V2(8; 0)

asymptotes: y = x

2 2
(b) The equation is equivalent to(x + 3) + (y 2) = 1. This is a horizontal
49 4
ellipse.

center: C(3; 2)

foci: F1(3 3 p 5; 2), F2(3 + 3 p 5; 2)


vertices: F1(10; 2); F2(4; 2)
covertices: W1(3; 0); W2(3; 4)

3. (a) The parabola opens downward and has focal distance c = 5. Its equation
2
is (x 7) = 20(y + 7).
(b) Since the ellipse has vertical or horizontal major axis, the center is at
either (1; 12) or (5; 3). Since the major axis is longer than the minor
axis, the center must be at (5; 3). Then the ellipse is vertical with a = 9
and b = 4. Its equation is
2 2
(x + 5) (y 3)
+ = 1:
16 81

268
All rights reserved. No part of this material may be reproduced or transmitted in any form or by any means -
electronic or mechanical including photocopying – without written permission from the DepEd Central Office. First Edition, 2016.
Completing the squares, we see that the rst equation is equivalent to 9(x +
2 2
3) = 4y + 36. On the other hand, the second equation is equivalent to
2
9(x + 3) = 36y + 36. Subtracting the second equation from the rst, we get
4y2 36y = 0, which has solutions y = 0 and y = 9.
When y = 0, the corresponding x values are 5 and 1. When y = 9, the
corresponding x values are 32 p 10. Therefore, the solutions are (5; 0),
p
(1; 0), 32 10 ; 9 .
Set up a coordinate system so that the opening of the hose (the parabola’s vertex) is at (0; 3) and
that the water ows towards the positive x-axis. Then the x-axis (y = 0) corresponds to the ground;
it follows the parabola passes
2
through the point (2; 0). Hence, the equation of the parabola is x = 43 (y 3).
If Nikko stands on a 1.5-ft stool and the vertex remains at (0; 3), the line
y = 1:5 will correspond to the ground. Hence, the water will strike the
4 p
ground when y = 1:5. This gives x= 3 (1:5 3) = 6. Therefore, the
p q
water will travel 6 2 ft further before striking the ground.
6. Let (x; y) be the coordinates of the point. This point satis es
2
p
(x 2)2 + y 2 = 3 jy 5j:
Manipulating this equation gives us
4 2
2 2
(x 2) + y = 9 (y 10y + 25)
2 2
9(x 2) + 5(y + 8y) = 100
2 2
9(x 2) + 5(y + 4) = 100 + 80
2 2
(x 2) +(y + 4) = 1:
20 36
Therefore, the point traces a vertical ellipse with center at (2; 4).

Supplementary Problems 2.1 (page 85)

1. a3 = a1 + (3 1)d = 35; a10 = a1 + (10 1)d = 77 ) d = 6; a1 = 23 ) a5 = 47.


n (2(17) + (n 1)3)
2
3. sn = 2 = 30705 ) 3n + 31n 61410 = 0. Using the
quadratic formula and noting that n must be a whole number, we have n =
138.
3
a1 a1 (1 r )
3
5. We have s = 108 = 1 r and s3 = 112 = 1 r = 108 (1 r ) ) r =
1
27 ) a1 = 144.
269
All rights reserved. No part of this material may be reproduced or transmitted in any form or by any means -
electronic or mechanical including photocopying – without written permission from the DepEd Central Office. First Edition, 2016.
Note that 0:123123 : : : = 0:123 + 0:000123 + 0:000000123 + : : : = 0:123 +
0:123(0:001) + 0:123(0:001)2 + : : :, which is an in nite geometric series with
r = 0:001. Thus, 0:123 = 0:123 = 41 .
1 0:001 333
4 (2a1 + (4 1)d)
We have s4 = = 80 ) 2a1 + 3d = 40. Since the sum of the 2
rst two numbers are one-third of the sum of the last two numbers, we have
1
3 (a1 + a2) = a3 + a4 ) 4a1 + 14d = 0. Combining yields d = 10, and thus
a1 = 5; a2 = 15; a3 = 25; a4 = 35.
11. Note that this is a geometric series with common ratio 2n 1. Thus, the sum
will have a nite value if j2n 1j < 1 ) 1 < 2n 1 < 1 ) 0 < n < 1.
Thus, n 2 (0; 1).

Supplementary Problems 2.2 (page 95)

i=3
10 i p3 3 2=2
3 2 +:::+
10
2 = 26
p

X
p
1. (a) 3

0 x2ix2 x4 x6 x 8 x 10
= + + + + 5
2i 2 4 8 16 32
i=1
5
Xi
2 3
(c) (1) i xi1 = x x +x x4
=2

150 (4i + 2) = 4 150 i+ 150


X
150(151)
3. (a) 2=4 + 2(150) = 45; 600

X X i
2
i=1 i=1 =1
120 120
2
(b) i(i 5) = (i 5i) 1(1 5) 2(2 5) = 120(121)(2(120) + 1) +10 =
Xi X 6
=3 i=1
583; 230
(c)
130 (2i3)(2i+3) = 130 (4i29) = 4
130 i2
130 9=
130(131)(2(130) + 1) +
Xi X X X 6
=1 i=1 i=1 i=1
9(130) = 741; 975
200 200 200
Xi X X 200 s
2 2
5. s = (i 1) i = (1 2i) ) i = 2
=1 i=1 i=1

270
All rights reserved. No part of this material may be reproduced or transmitted in any form or by any means -
electronic or mechanical including photocopying – without written permission from the DepEd Central Office. First Edition, 2016.
Supplementary Problems 2.3 (page 108)

Part 1.
1 1+2
1
2 =2 2
The formula is true for n = 1.

Part 2.
k i k+2
Xi
Assume: i =2 k .
=1 2 2
k+1 i k+3
Xi
i .
To show: 2 =2 2k+1
=1

k+1
Xi k i k+1
Xi
2i = i + k+1
2
i=1 =1 2
k+2 k+3
k
=2 2 + 2k+1
k+3

=2 2k +1 :

Part 1.
1(1!) = (1 + 1)! 1
The formula is true for n = 1.

Part 2.
k
X

Assume: i(i!) = (k + 1)! 1.


i=1
k+1
X

To show: i(i!) = (k + 2)! 1.


i=1

k+1 k
X X
i(i!) = i(i!) + (k + 1)[(k + 1)!]
i=1 i=1
(k + 1)! 1 + (k + 1)[(k + 1)!]
(k + 2)(k + 1)! 1
(k + 2)! 1:

271
All rights reserved. No part of this material may be reproduced or transmitted in any form or by any means -
electronic or mechanical including photocopying – without written permission from the DepEd Central Office. First Edition, 2016.
5. Part 1.
1 1 =1 = 1
2 2 2(1)
The formula is true for n = 2.
Part 2. .
Assume: P = 1 2 1 3 1 k 1 1 k =2k
1 1 1 1 1
To show: 1 2 1 k 1 k + 1 = 2(k + 1).
1 1 1 1

1 2 1 k+1 =P 1 k+1
1 1 1

1 k
= 2k k + 1
1
= 2(k + 1) :

Part 1.
3(1)+1 3(1)+1
4 +2 + 1 = 273 = 7(39)
The number is divisible by 7 for n = 1.

Part 2.
3k+1 3k+1
Assume: 4 +2 + 1 is divisible by 21.
3(k+1)+1 3(k+1)+1
Prove: 4 +2 + 1 is divisible by 21.
43(k+1)+1+23(k+1)+1+1 = 64 43k+1+8 23k+1+1 = 56 43k+1+8 43k+1 + 23k+1 + 1
7

Part 1.
2(1)+1 1+2 1+2 2(1)+1
5 2 +3 2 = 1216 = 19(64)
The number is divisible by 19 for n = 1.

Part 2.
2k+1 k+2 k+2 2k+1
Assume: 5 2 +3 2 is divisible by 19.
Prove: 52(k+1)+1 2(k+1)+2 + 3(k+1)+2 22(k+1)+1 is divisible by 19.
52(k+1)+1 2(k+1)+2 + 3(k+1)+2 22(k+1)+1 = 50 52k+1 2k+2 + 12 3k+2 22k+1 =
12 (52n+1 2n+2 + 3n+2 22n+1) + 38 52n+1 2n+2

272
All rights reserved. No part of this material may be reproduced or transmitted in any form or by any means -
electronic or mechanical including photocopying – without written permission from the DepEd Central Office. First Edition, 2016.
Part 1.
10 1 5 1+2
3 + 3 + 4 = 1029 = 3(343)
The number is divisible by 3 for n = 1.

Part 2.
k 5 k+2 10
Assume: 3 + 3 + 4 is divisible by 3.
10k+1 5
Prove: + + 4k+3 is divisible by 3.
3 3
k k
10k+1 5 10 5 10 5 5
k+3 n+2
3 +3 +4 = 10 3+ 3 +4 4k+2 = 10 3 +3 + 4k+2 64 93
1
13. Part 1. 1 2 1 =1
Part 2
k
X
1 1
i

Assume: i
3
2 k
=
1
k+1
X
1 1
i
3
Prove: =1 i 2 k+1
3
k+1 1 1 1 (k + 1) k
Xi 2
2 + 3
=2 )
3

i 3
k (k + 1) (k + 1) 3
. Note that 0 < (k + 1)2
=1
2 3 (k + 1) k (k + 1) 1
(k + 1) < (k + 1) k, thus 2 3 <2 3 =2 .
(k + 1) (k + 1) k+1

Supplementary Problems 2.4 (page 119)


5 5 4 3 2 2 3 4 5
1. (a) (2x 3y) = 32x 240x y + 720x y 1080x y + 810xy 243y
px 2 4 32 8 1 8
2 8 3 2
(b) 3 x = 3 x11=2 + 16x + 3x + 81 x 27 x1=2
p 4 3=2 2 p
(c) (1 + x ) = 4x + x + 6x + 4 x+1
4
X 10

3. Approximating yields (2:1)


10
k=0 k 210k (0:1)k = 1667:904, which has
an approximate error of 0:08.
1
9 19
X

k 19
5. In sigma notation we have k=0 k (3) = (1 3) = (2) 19 .

273
All rights reserved. No part of this material may be reproduced or transmitted in any form or by any means -
electronic or mechanical including photocopying – without written permission from the DepEd Central Office. First Edition, 2016.
Topic Test 1 for Unit 2 (page 121)

1. (a) G, r = 3=2 (b) O (c) O

2. First, a3 = a2 + 5 = a1 + 10. Also, a3 + 1 = a2 + 2 . Thus, a1 = 5,


a2 = 10, and a3 = 15. a2 + 2 a1 + 4
50 3 2
X
2i + 9i + 13i + 6 (2i + 3)(i + 1)(i + 2)
i

3. We have 2
i + 3i + 2 = (i + 1)(i + 2) = 2i + 3.
=1
50 3 2
2i + 9i + 13i + 6 50
Xi X
Thus, = (2i + 3) = 2700.

2
=1 i + 3i + 2 i=1

8 1 k

4. (a) k x162k 2 =) 16 2k = 8 =) k = 4
8 1 4 35
2819 =) 4 x8 2 = 8 x8
(b) k = 19 =) 19 (n3) (3m )19 = 19 319n27m19
28 28

5. For n = k + 1:
1 1 1
1 3 +3 5 + + (2(k + 1) 1)(2(k + 1) + 1)
2
k 1 2k + 3k + 1 k+1
= 2k + 1 + (2k + 1)(2k + 3) = (2k + 1) (2k + 3) = 2k + 3
6. a1 = 10; 000, r = 1:04, 60 20=40
40
1 (1:04)
s40 = 10; 000 1 (1:04) 499; 675:83 pesos

Topic Test 2 for Unit 2 (page 122)

1. (a) G, r = 4=5 (b) O (c) A, d = 5=2

2. We have a1 + a2 = 2a1 + d = 9 and a1 + a2 + a3 = 3a1 + 2d = 9 yielding


a1 = 9; d = 9. Using sn = 126, we get n = 7.
50 50
Xi X
2
3. (a) (2i + 1)(i 3) = 2i 5i 3 = 79; 472
=1 i=1

(b) i=1
30 r
i2 2i + 1 =
4 i=1
30 i 1
2
435 = 2

X X

274
All rights reserved. No part of this material may be reproduced or transmitted in any form or by any means -
electronic or mechanical including photocopying – without written permission from the DepEd Central Office.
First Edition, 2016.
8 1 k 8 8
8k
4. k(x3) x = kx244k =) 24 4k = 0 =) k = 6 =) 6 =28
5. (a) For n = k + 1:

1 + 4 + 7 + + (3(k + 1) 2)
k(3k 1) 3k2 + 5k + 2 (k + 1)(3k + 2)
= + (3k + 1) = =
2 2 2
(n+1) (n+1)1 n n1 n
k + 1: 3 +7 + 8 = 7 (3 + 7 + 8) 4 3
(b) For n = n 6 8,

where 4 3 is divisible by 12 for n 1, and 6 8 = 48 = 12(4).


Supplementary Problems 3.1 (page 133)

1. 6 rev = 6 rev 360 = 432


55 1 rev
6 6 24

3. 216 = 216 180 = 5 rad; s = 4 5 = 5 cm


5. 2110 5(360 ) = 310
2
9 1 9 7 189
7
;r= cm; A = = 2
7. = 6 2 2 2 6 16 cm
s p
5 2(15) 6 6
2
9. =150 = ; A = 15 in ; r = 5 = in
6 6 =p
11. r = 6 in; s = 6 in; =r = 6 = 1 rad; 1 rad = 1 57:30
s 6 180

13. 8 cm
3
2
15. =20 =
v
9 rad; A = 800 cm
r= =p = cm; s = 9 = 3 cm
p p p
u 2(800) 120 120 120 40

u 9

3
17. r = 6 cm; = 54 = 10 2 2
Area of shaded region = 2 area of sector AOE = 2 2 (6) 10 = 5 cm
1 3 54

1 2 1
A =A A = 2
p p
19. segment sector triangle 2 3 (6) 2 (3)(6 3) = (12 9 3) cm2
2
7
5
All rights reserved. No part of this material may be reproduced or transmitted in any form or by any means -
electronic or mechanical including photocopying – without written permission from the DepEd Central Office. First Edition, 2016.
Supplementary Problems 3.2 (page 143)

1. 33 is coterminal with 33 8 = , and 33 terminates in QI.


4 4 4 4
The secant function is positive in QI and QIV. The cotangent function is
negative in QII and QIV. Therefore, the angle is in QIV.

p 1
5 5 3
!
5. 6 is in QII. The reference angle is 6 , and therefore P 6 = 2 ;2 .
2 p 13
7. tan = 3 ; cos > 0 =) sec = 3
p 13 2 p
sec + tan 3 17 4 13
3
sec tan =p = 9
2
13
+
3 3
p r 2 2p 3
9. csc = 2, cos < 0; r = 2; y = 1; x = 3; sec = x = p 3 = 3

11. csc = 4 and not in QIII =) in QIV


4
csc = 1 =) r = 4; y = 1
p
p
p
= x 15 r 4 p 15 y 1

2
x (4) (1) 2 = 15, is in Quadrant IV, x = p 15

cos = r = 4 sec = x = 15 sin = r = 4


p

csc = r =4 tan = y= 15 cot = x = p 15


y x 15 y
x p5 p 2
r
2p p y 2p 5

13. x = 2; y =4 =) r = (2) + (4) = 2 5

cos = r = 5 sec = x = 5 sin = r = 5


csc = r = p 52 tan = y = 2 cot = x = 1

y xy 2

p p
x ;y ;r = (2)2 + (6) 2 =2 10; sec = 10, csc = p 10

15. =2 2 =6 p 10 80 3
sec csc2 = 10 =
99
2 p 33 11
17. cos = sin 3 = 2 and 2 < <2 =) = 6
2
7
6
All rights reserved. No part of this material may be reproduced or transmitted in any form or by any means -
electronic or mechanical including photocopying – without written permission from the DepEd Central Office.
First Edition, 2016.
19. f(x) = sin 2x + cos 2x + sec 2x + csc 2x + tan 2x + cot 2x
p p
7 2 2

p p
f8 = 2 + 2+ 2 2 1 1=2
Supplementary Problems 3.3 (page 170)

1. P = 2
4 8
4
=8 7. domain = R; range = 3 ;3
1

9. y = 3 sec 2(x ) 3
3. 4 = 2 =) k = 8
k
3 8 2 13 3

5. y = 3 sin 4 9 + 3 5= 2 11. Asymptotes: x = 2 + 2k ; k 2 Z


13. (a) P = 8 , phase shift = 4 , domain = R, range = [3; 1]

(b) P = , phase shift = 6, domain = xjx 6= 3 + k ; k 2 Z , range = R

277
All rights reserved. No part of this material may be reproduced or transmitted in any form or by any means -
electronic or mechanical including photocopying – without written permission from the DepEd Central Office. First Edition, 2016.
2k
(c) P = 4
, phase shift = 2 , domain = xjx 6= 2 + 3 ; k 2 Z , range =
3
3 1

; 2 [ 2 ;1

k
6 , domain = xjx 6= 12 + 2 ; k 2 Z , range =
P = , phase shift = (; 1]
[ [3; 1)

1
15. y = 8 cos (t 10 ); at t = 10, y 4:32 (that is, the mass is located about
10

278
All rights reserved. No part of this material may be reproduced or transmitted in any form or by any means -
electronic or mechanical including photocopying – without written permission from the DepEd Central Office. First Edition, 2016.
Supplementary Problems 3.4 (page 179)

sin x
tan x sin x cos x sin x sin x 1
1 = sec x
1. sin x = sin x sin x = cos x sin x 1
2 2 2
cos Asin A + sin A cos A sin A + 1
3. sin A + = = =1
1 + sin A 1 + sin A 1 + sin A
1 +1 cos x + sin x
csc x + sec x
= sin x cos x = sin x cos x = cos x + sin x
5. 2 2
cot x + tan x cos x sin x cos x + sin x
sin x +cos x sin x cos x
sin x sin x sin x + sin x cos x
cos x
tan x + sin x cos x
7. = =
csc x + cot x 1 + cos x 1 + cos x
sin x sin x sin x
sin x(1 + cos x) 2 2
cos x sin x 1 cos x
cos
= 1 + cos x = x =cos x
sin x
cos = cot cot a
2 2 2 2
9. sin cos = sin sin csc = 1 + cot =1 + a
1 + 11 + sin a
csc a + 1 sin a sin a 1 + sin a
11.csc a 1 = 1 1 = 1 sin a =1 sin a
sin a sin a
2 2
cos a cos a cos a 1 sin a
13. sec a + tan a = 1 + sin a = 1 + sin a = 1 + sin a = 1 sin a
cos a cos a
1 1 2
15. + = 1 + cos a + 1 cos a = 2 = 2 = 2 csc a
2
1 cos a 1 + cos a (1 cos a)(1 + cos a) 1 cos a sin2 a
sin sin
tan cos cos
2 2 = 2 2
17. 1 tan = sin cos sin
1 cos2 sin 2

2
sin cos
2 2
=cos cos sin
sin cos sin cos
2 2 2
= cos (1 cos ) = 2 cos 1

279
All rights reserved. No part of this material may be reproduced or transmitted in any form or by any means -
electronic or mechanical including photocopying – without written permission from the DepEd Central Office.
First Edition, 2016.
cos sin
19. cotsin sec = sin cos
sec csc 1 1
cos sin
2 2
cos sin
2 2
= sin cos cos sin = cos sin

Supplementary Problems 3.5 (page 188)

1
2 p3 1
1. cos = sin 3 = 2 and in QIV =) = 6
sin +k B
2
3. +k
tan A = tan 2 B = cos 2 +k B
+
k B +
k B
sin cos cos sin
= 2 2

cos 2
+k cos B + sin 2
+ k sin B
sin 2
+k cos B
= sin 2
+k sin B

= cot B
5. sin 105 cos 15 = sin(90 + 15 ) cos 15 = cos 15 cos 15 = 0
p
7. cot = 7; csc = 10; and and are acute
p p p p
7 2 2 10 3 2
=) cos = 10 ; sin = 10 ; sin = 10 ; cos = 10
cos( + ) = cos cos sin sin
p p p
7 2 3 2 p2 p 10 2 5

= 10 ! 10 ! 10 ! 10 ! = 5
2
3 sin x = 2 =) sin x = 3
sin(x ) + sin(x + )
0 sin x coscos x sin + sin x cos + cos x sin
= 2 sin x cos = 2 3 (1) = 3
2 4

4 3
11. sin A = 5 and A in QII =) cos A = 5
4 3
cos B = 5 and B in QIV =) sin B = 5
(a) sin(A B) = 5 5 35 35

4 4 7
= 25

280
All rights reserved. No part of this material may be reproduced or transmitted in any form or by any means -
electronic or mechanical including photocopying – without written permission from the DepEd Central Office. First Edition, 2016.
(b) cos(A B) = 3 4 + 4 3 = 24
5 5 5 5 25
7
25 = 7
(c) tan(A B) =
24 24
25
cos(A B) < 0 and sin(A B) > 0 =) A B in QII
13. Given: sin = 4 and cos = 5
5 13
sin( + ) + sin( ) = sin cos + cos sin + sin coscos sin
= 2 sin cos = 2 5 13 = 13
4 5 8

p 1
15. csc A = 17; A in QI =) tan A = 4
p 34 3
csc B = 3 ; B in QI =) tan B = 5
1 +3
tan(A + B) = 4 5 =1 = A+B =45
)
3
1
5

17.
tan + tan 23 = tan 9 +
36 = tan 4=1

9 36 23 3
1 tan tan 23

9 36
19. sin 2 = sin( + ) = sin cos + cos sin = 2 sin cos

Topic Test 1 for Unit 3 (page 190)


5
1. r = 6 cm, = 37:5 = 37:5 = rad
180 24
(a) s = 6 5 = 5 cm
24 4

1 2 5 15 2
(b) A = 2 (6) 24 = 4 cm
p
2 p
2. x = 1, y = 2, r= (1) + (2) 2= 5
p p p
2 5 5 10 3 5
sin + cos + tan = + +2=
5 5 5

281
All rights reserved. No part of this material may be reproduced or transmitted in any form or by any means -
electronic or mechanical including photocopying – without written permission from the DepEd Central Office. First Edition, 2016.
12 5 ; tan A = 12
3. sin A = 13 , A is in QII =) cos A = 13 5
5 3 4 3
cos B = 3 , B is in QIV =) sin B = 5 ; cos B = 5 ; tan A = 4
(a) cos(A B) = cos A cos B + sin A sin B
= 13 5 + 13 5 = 65
5 4 12 3 56
(b) tan(A B) = tan A + tan B = 5+ 4 = 33
12 3
1 + tan A tan B 1+ 5 4 56

12 3

tan 57 + tan 78
4. = tan(57 + 78) = tan 135 = 1
1 tan 57 tan 78
cos x tan x + sin x sin x
p 2 p 2
5. tan x = cos x + tan x = 2 cos x = 2 1 sin x = 2 1 a
6 6 2 3 2 3
6. cos x + sin x = (cos x) + (sin x)
= (cos2 x + sin2 x)(cos4 x cos2 x sin2 x + sin4 x)
4 2 2 4
= cos x cos x sin x + sin x
4 2 2 2 2
= cos x cos x(1 cos x) + (1 cos x)
4 2 4 2 4
= cos x cos x + cos x + 1 2 cos x + cos x
4 2
= 3 cos x 3 cos x 1
Connect the three diagonals of the hexagon. In doing this, the hexagon is p !
1 3
divided into 6 equilateral triangles. Hence, B ; . Same coordinates
2 2
for C, E and F , except that they will just vary in signs depending on the
quadrant.
8. y = 2 sin 2 + 3 1 =) y = 2 sin 2 x + 3 1
x 1 2

P = 4 , Phase Shift = 2 , Amplitude = 2, Range = [3; 1]


3

282
All rights reserved. No part of this material may be reproduced or transmitted in any form or by any means -
electronic or mechanical including photocopying – without written permission from the DepEd Central Office. First Edition, 2016.
Topic Test 2 for Unit 3 (page 191)

1. A = 2
sector 3 cm , = 30 = 30 180= 6 rad
1
2
3 = 2 6 r =) r = 2 cm =) 2 6 = 3 cm
2 2
2. x = 8, y = 6, r= (8) + (6) =10 2
sin2 + 2 sin cos + cos2

(sin + cos ) = = 1 + 2 sin cos = 1 + 2


p 10 10 = 25
6 8 1

8
3. sin A =
17
1
5 8 7

sin 2 A + cos 2 A = cos A + sin A = 17 + 17 = 17


4. sin 160 cos 35 sin 70 cos 55
= sin 20 cos 35 cos 20 sin 35
p2 p6
= sin(20 35) = sin(45 30) =
4
p
7 tan 4 + tan 3 1+ 3
5. tan + = p =2 p3
12 = tan 4 3 = 1 tan 4 tan 3 1 3
3 4 4
6. cos A = 5 , A is in QIII =) sin A = 5 ; tan A = 3
2
4 24 7
tan B = 7 , B is in QIII =) sin B = 25 ; cos B = 25 ,
(a) sin(A + B) = sin A cos B + cos A sin B
= 5 25 + 5 25 =5
4 7 3 24 4
1
(b) cot(A + B) = tan A tan B = 1 3 7 = 3
4 24
tan A + tan B 3 + 7 4

4 24
2 2
tan x tan x
tan x
3 2 2
7. tan x + tan x =tan x(1 + tan x) = sec x = sin x cos x

2
8
3
All rights reserved. No part of this material may be reproduced or transmitted in any form or by any means -
electronic or mechanical including photocopying – without written permission from the DepEd Central Office.
First Edition, 2016.
sin x
8. = sin x cos x
sec x
1
sin x cos x =
3
1 2 1
2
2 2 sin x cos x + cos x = 9
2
(sin x cos x) = 3 =) sin x
1 8 4
=) 1 2 sin x cos x = 9 =) 2 sin x cos x = 9 =) sin x cos x = 9
y x = tan 1 x 6 +2
9. = tan 18 3 +2 3
P = 3 , phase shift = 6

Supplementary Problems 3.6 (page 200)


p p p
1. (a) sin 2 = 4 2 (c) tan 2 = 4 2 (e) csc 2 = 9 2
8
9 7

(b) cos 2 = 7 (d) sec 2 = 9 (f) cot 2 = 7p 2


9 7 8
3. cos(2t) = 1 7. cot 4 = 1=(tan 4 ) = 7=24
8
p p
2 5 2+ 2 25 2
5. tan x = 1 5

2 9. sin 8 = 4 and cos 8 = 284


1 cos y 1 cos y sin y
tan 1 y 1 1
11. 2 =
sin y sin y
tan 1 y+1 sin y = sin y + 1 + cos y
2
1 + cos y + 1 1 + cos y
2
1 cos y sin y sin y cos y
= 2
sin y + sin y + sin y cos y
2
sin y sin y sin y cos y
= 2
sin y + sin y + sin y cos y
= sin y 1 cos y :
sin y + 1 + cos y
p 2 4
All rights reserved. No part of this material may be reproduced or transmitted in any form or by any means -
electronic or mechanical including photocopying – without written permission from the DepEd Central Office.
First Edition, 2016.
p 2+p p
13. (a) cos 105 = 2
3
(b) tan 22:5 = 2 1

Supplementary Problems 3.7 (page 219)


1
1. (a) sin[sin (1=2)] = 1=2
1
(b) cos[cos ( p 2=2)] = p 2=2
1
(c) tan[tan ( p 3)] = p3
p p
(d) sin[arctan( 3)] = 3=2
p
cos[arccos( 2)] does not exist p
tan[arcsin(1=4)] = 15=15
1 p
cos(sin 3=2) = 1=2
1
3. (a) sin[2 cos (4=5)] = 24=25
1
cos[2 sin (5=13)] = 119=169
1 1
sin[sin (3=5) + cos (5=13)] = 33=65
p
1 1
(d) cos[sin (1=2) cos (8=17)] = (15 + 8 3)=34
p
5. (a) arcsec( 2) = 3 =4
arccsc(2) = =6 p
(c) arccot 3 = =6
1 1 2
(d) [sec (1)] [cos (1)]==
1 1
(e) 2 cot p 3 + 3 csc 2=2( =6)+3( =6)=5 =6
1
0 csc 0 does not exist
Vertex angle should be =3.

Supplementary Problems 3.8 (page 236)

1. Solution set: f =2; 3 =2; 5 =2; 7 =2; :::g = f(2k + 1) =2 j k 2 Zg

3. Solution set: f2k =3 j k 2 Zg

5. Solution set: f =8; 3 =8; 9 =8; 11 =8g

7. Solution set: f =2; =6g


The bullet should be red with an angle of = 60 .

285
All rights reserved. No part of this material may be reproduced or transmitted in any form or by any means -
electronic or mechanical including photocopying – without written permission from the DepEd Central Office. First Edition, 2016.
Supplementary Problems 3.9 (page 252)

1. (a) (13; 7 =3), (13; 13 =3) (c) (15; 7 =4), (15; 23 =4)
(b) (0; 2 ), (0; =4)
3. (a) (3; 0) (c) (0; 5=2)
p p p
(b) (3 2=2; 3 2=2) (d) (4 3; 4)

5. (a) r = 4 cos 2

0 =6 =4 =3 =2 2 =3 3 =4 5 =6 7 =6 5 =4 4 =3 3 =2
r 4 2 0 2 4 2 0 2 4 2 0 2 4

(b)

Topic Test 3 for Unit 3 (page 253)


1. (a) cos(2 ) = 119 (c) tan(2 ) = 120
csc(2 ) = 169
169 119 120
120 169 119
(b) sin(2 ) = 169 (d) sec(2 ) = 119 cot(2 ) = 120

2. Hint: Use the double-angle identity for tangent tan(2x) = 2 tan x . 2

1 tan x
p p
4 6 2
3. (a) tan 15 = 2 p3 (b) tan 7:5 = p6 p2
1
103= (b) cos sin 40 9
1
4. (a) tan cot 6 3 41 = 41
5. Solution Set = 0; ; 3
2 2

286
All rights reserved. No part of this material may be reproduced or transmitted in any form or by any means -
electronic or mechanical including photocopying – without written permission from the DepEd Central Office.
First Edition, 2016.
; 7
6. Solution Set = 0;;2; 6 56 ; 6; 6
11

7. r = 2 2 sin

0 =6 =4 =3 =2 2 =3 3 =4
r 2 3 2 p2 2 p 3 4 2 p3 2 p2
5 =6 7 =6 5 =4 4 =3 3 =2
r 3 2 1 2+ p2 2+ p 3 0

1
8. (a) (r; ) = (70; tan 4 ) (d) (r; ) = (7; 0)
3
p p
(b) (r; ) = (100 2; 4 ) (e) (r; ) = (2 2; 4 )
1 12
(c) (r; ) = (7; 2) (f) (r; ) = (13; tan 5)

9. (a) (x; y) = ( 3
2
; 3p23 ) (d) (x; y) = (5; 0)
(b) (x; y) = ( 2 ; 2 ) (e) (x; y) = (2 ; 0)
p p
45 2 45 2

(c) (x; y) = (1; 0) (f) (x; y) = ( 9p 3 ;9 )


2 2

287
All rights reserved. No part of this material may be reproduced or transmitted in any form or by any means -
electronic or mechanical including photocopying – without written permission from the DepEd Central Office. First Edition, 2016.
Topic Test 4 for Unit 3 (page 254)
1. (a) cos(2 ) = 527 (b) sin(2 ) = 336 (c) tan(2 ) = 336
625 625 527
2 2 2
2. (a) cos 24 0:8346 (b) sin 24 0:1655 (c) tan 24 0:1983
p p
2
3. (a) tan( =12) = 2 p3 (b) tan( =24) = 4 p6 6 p2
4. cos cos1 7 + cos1 5 = 35
p
1 3 3 16 3

5. Solution set = ; 3 ;4 ; 5
4 4 3 3
6. Solution set = 58 ; 78 ; 8 ; 8;4; 4
13 15 5

7. r = 2 + 2 cos
0 =6 =4 =3 =2 2 =3 3 =4
p p
r 4 2+ 3 2+ 2 3 2 1 2 p2
5 =6 7 =6 5 =4 4 =3 3 =2
r 2 p3 0 2 p3 2 p2 1 2

288
All rights reserved. No part of this material may be reproduced or transmitted in any form or by any means -
electronic or mechanical including photocopying – without written permission from the DepEd Central Office. First Edition, 2016.
1
8. (a) (r; ) = (35; tan ( 43 )) (d) (r; ) = (5; )
p 5 p
(e) (r; ) = ( 2; )
(b) (r; ) = (100 2; 4 ) 4
1 8
(c) (r; ) = (5; 2) (f) (r; ) = (17; tan ( 15 ))
p
9. (a) (x; y) = (2 3; 2) (d) (x; y) = (5; 0)
(b) (x; y) = (50 p 2; 50 p 2) (e) (x; y) = ( ; 0)
p
(f) (x; y) = ( 15 ; 15 3 )
(c) (x; y) = (1; 0)
2 2

289
All rights reserved. No part of this material may be reproduced or transmitted in any form or by any means -
electronic or mechanical including photocopying – without written permission from the DepEd Central Office. First Edition, 2016.
References

R.N. Aufmann, V.C. Barker, and R.D. Nation, College Trigonometry, Houghton
Mi in Company, 2008.

E.A. Cabral, M.L.A.N. De Las Pe~nas, E.P. De Lara-Tuprio, F.F. Francisco,


I.J.L. Garces, R.M. Marcelo, and J.F. Sarmiento, Precalculus, Ateneo de
Manila University Press, 2010.

R. Larson, Precalculus with Limits, Brooks/Cole, Cengage Learning, 2014.

L. Leithold, College Algebra and Trigonometry, Addison Wesley Longman


Inc., 1989, reprinted by Pearson Education Asia Pte. Ltd., 2002.

M.L. Lial, J. Hornsby, and D.I. Schneider, College Algebra and Trigonometry and
Precalculus, Addison-Wesley Educational Publisher, Inc., 2001.

J. Stewart, L. Redlin, and S. Watson, Precalculus: Mathematics for Calculus,


Brooks/Cole, Cengage Learning, 2012.

M. Sullivan, Algebra & Trigonometry, Pearson Education, Inc., 2012.

C. Young, Algebra and Trigonometry, John Wiley & Sons, Inc., 2013.

290
All rights reserved. No part of this material may be reproduced or transmitted in any form or by any means -
electronic or mechanical including photocopying – without written permission from the DepEd Central Office. First Edition, 2016.

Das könnte Ihnen auch gefallen